From f588b148aef8c3e05f61d1c0c245a0c4bede1693 Mon Sep 17 00:00:00 2001 From: aaronshaw Date: Tue, 20 Oct 2020 12:54:20 -0500 Subject: [PATCH] initial commit of categorical data analysis examples --- r_tutorials/w06-R_tutorial.html | 1690 +++++++++++++++++++++++++++++++ r_tutorials/w06-R_tutorial.pdf | Bin 0 -> 241614 bytes r_tutorials/w06-R_tutorial.rmd | 114 +++ 3 files changed, 1804 insertions(+) create mode 100644 r_tutorials/w06-R_tutorial.html create mode 100644 r_tutorials/w06-R_tutorial.pdf create mode 100644 r_tutorials/w06-R_tutorial.rmd diff --git a/r_tutorials/w06-R_tutorial.html b/r_tutorials/w06-R_tutorial.html new file mode 100644 index 0000000..2d62246 --- /dev/null +++ b/r_tutorials/w06-R_tutorial.html @@ -0,0 +1,1690 @@ + + + + + + + + + + + + + + + +Week 6 R Lecture + + + + + + + + + + + + + + + + + + + + + + + + + + + + + + + + + + + + + + + + + + + + + + + + +
+ + + +
+
+
+
+
+ +
+ + + + + + + +
+

Analyzing categorical data in R

+

The goal of this script is to help you think about analyzing categorical data, including proportions, tables, chi-squared tests, and simulation.

+
+

Estimating proportions

+

If a survey of 50 randomly sampled Chicagoans found that 45% of them thought that Giordano’s made the best deep dish pizza, what would be the 95% confidence interval for the true proportion of Chicagoans who prefer Giordano’s?

+

Can we reject the hypothesis that 50% of Chicagoans prefer Giordano’s?

+
est = .45
+sample_size = 50
+SE = sqrt(est*(1-est)/sample_size)
+
+conf_int = c(est - 1.96 * SE, est + 1.96 * SE)
+conf_int
+
## [1] 0.3121018 0.5878982
+

What if we had the same result but had sampled 500 people?

+
est = .45
+sample_size = 500
+SE = sqrt(est*(1-est)/sample_size)
+
+conf_int = c(est - 1.96 * SE, est + 1.96 * SE)
+conf_int
+
## [1] 0.4063928 0.4936072
+
+
+

Tabular Data

+

The Iris dataset is composed of measurements of flower dimensions. It comes packaged with R and is often used in examples. Here we make a table of how often each species in the dataset has a sepal width greater than 3.

+
data(iris)
+table(iris$Species, iris$Sepal.Width > 3)
+
##             
+##              FALSE TRUE
+##   setosa         8   42
+##   versicolor    42    8
+##   virginica     33   17
+

As described in the most recent OpenIntro reading, the chi-squared (\(\chi^2\)) test is a test of whether the frequencies we see in a table differ from what we would expect if there was no difference between the groups. Base-R provides a chisq.test() function to conduct these tests. Here’s an example of the test evaluating whether the frequency of Iris sepal widths greater than 3 differs by species (the null hypothesis is that there is no difference in the frequencies and the groups are drawn from the same population). It’s always good to examine the contingency table before conducting the test!

+
table(iris$Species, iris$Sepal.Width > 3)
+
##             
+##              FALSE TRUE
+##   setosa         8   42
+##   versicolor    42    8
+##   virginica     33   17
+
chisq.test(table(iris$Species, iris$Sepal.Width > 3))
+
## 
+##  Pearson's Chi-squared test
+## 
+## data:  table(iris$Species, iris$Sepal.Width > 3)
+## X-squared = 50.225, df = 2, p-value = 1.241e-11
+

While we know little about the dataset, sample, or measurement, the incredibly low p-value means that the observed frequencies likely did not come from the same distribution and that the frequency of sepal widths more/less than 3 differs by species.

+

Note that you can create tables to conduct \(\chi^2\) tests by hand. The chisq.test() function will need the object to be of “class” table or matrix in order to run the test.

+
+
+
+

BONUS: Using simulation to test hypotheses and calculate “exact” p-values

+

Section 6.1.4 of OpenIntro mentions a simulation-based approach to inference when assumptions of \(\chi^2\) tests aren’t met. You possess all the tools to implement something like this in R, so let’s do it. Basically, the idea is that we use simulated data to construct a null distribution of outcomes and then compare our observed test statistic against the null to estimate an “exact” p-value corresponding to the proportion of samples drawn from the null would produce statistics at least as large as our observed data.

+

The earlier (3rd) edition of Chapter 6 of the OpenIntro textbook uses an example of a medical practitioner who has 3 complications out of 62 procedures, while the typical rate is 10%. We can use that information to create an example here based on the proportions. The null hypothesis is that this practitioner’s true rate is also 10%, so we’re trying to figure out how rare it would be to have 3 or fewer complications, if the true rate is 10%.

+
# We write a function that we are going to replicate
+simulation <- function(rate = .1, n = 62){
+  # Draw n random numbers from a uniform distribution from 0 to 1
+  draws = runif(n)
+  # If rate = .4, on average, .4 of the draws will be less than .4
+  # So, we consider those draws where the value is less than `rate` as complications
+  complication_count = sum(draws < rate)
+  # Then, we return the total count
+  return(complication_count)
+}
+
+# The replicate function runs a function many times
+simulated_complications <- replicate(5000, simulation())
+

We can look at our simulated complications

+
hist(simulated_complications)
+

+

And determine how many of them are as extreme or more extreme than the value we saw. This is the “exact” p-value.

+
sum(simulated_complications <= 3)/length(simulated_complications)
+
## [1] 0.1324
+
+ + + +
+
+ +
+ + + + + + + + + + + + + + + + diff --git a/r_tutorials/w06-R_tutorial.pdf b/r_tutorials/w06-R_tutorial.pdf new file mode 100644 index 0000000000000000000000000000000000000000..c3825ce852c24d2f7e259b2c5f9d41382983e2eb GIT binary patch literal 241614 zcmd42W0WV&vM>B^+n%=y}gU6DABs0Q|!A|SeO`CyTNeqgPzSB`6 z{U9b8LVbl+tGU3_ofW_{1~*@f;s*>CoUjBb;!T)tRK^|R4atCtn?|jtn+jcT96u>B zo+8$HkKUNs1iPJCnLIQSH*i8i;?SNP#H>=%oD$3>Ja7zZ64OCMhT2{oM5L9Ag9cRz zpP~fA*bh$XN`s_HM7pVriAe(mDn#HPQCP&NAxzGJeT%VlUgbi0`%Ri6HNB3lM1Nq= z0!lW1ZF*u-eA;iK-v3s&^j1%nd?>E?H70y{#HzGs%c;e;{k!t)(?6bsQ){vw+W9lo z)vRhE_As1v6tus=>uWc%TleZSg2hd}M!TEFO9SkkZCX@KF=TcLZptCoBb7%sMcT$v za~~nik&u(W??)AQ7630Y4n!>XNsz@tuTL-wZW{rWr)=kuj>n)YyA$r&{l3xG+c&_nv$S>d-FmYFaz!Ku14IN>b`A zD+z^_VeUXUI?h80Z5;d<(y&)5>%gUXU^Tqsd<>0d=VlMG6l=@wkV0>ynHG1c+o=`f zLWiIkb5do$HaU?rtu>XZf9NM?mbzMNqziR*m_aeQEJ*~1!^h!ZtxwAs1?QF<7SXB$ z*>y^Zvt)z?7!!7rKn{{!Y{^?xlFeEY%^*jyuN7u$Ic#!OQ?4{H0wJBsqUUe`cds*+ zj|%KCv>Wwt!;NM3WGdr~H044eP9qe!b`V>7Ql&7ZZBB4m#9}s$P1tXDPD*ViY4088 z&p_G8`X2~cxZ}AX{_ZTzmLyw{Q6=W}^g&W^(#CWH`K)@7wsuCAsro7l>{BU&r@uJ- ziH7|dCT99g47I?yTFvh0#N}zi(j4r2jJTR3SGKy-FAxo%5N>03ceq(h1web!}&@NxE z8^$QK!+Pt)9jWg4iPC8dpZ_cLJpqjSPi4kd@fv*&00n02ibpSk{hcs#h@O;L;zHb& zz%Kdue)Y7rsWe$3Lv))2aY%7K;6Mf4m_R<_x@$Ln;cZ_X@A6J)yKhV zI#*iiiMFUK?ygWBd$6kE4OSuGA>C^2^X>dO)>{ftuXZ}zYOqb170M*|T(j@jF-*}DsQ^ZswPJ>F@o2U}Q2f5wiJ$~ZrvaU+^J0U5!vtE6bp}doS7_Q7 zCA;7oE-DJm8k60Z-HOxfUM~zD`W;1jC2h5VlqQZ32(uy4mU zXHDD9CC%;Xt#;~G8<^_#fOAfY)&*(uR)Z2_iX|}BEQv|v+t=#TFM`F)Wh@jO)2R!6 z#@S1KM;Zc{6Bz`}I8(Id&Y)|;<>p>t7@Xw$8Ny0xB~8Pi=F8f)K;y@XddEobzV?n? zKXEWkiRH-tf?DVSXg2u{o*K{ga8mcB-%x9QM(M9l_wp=F{#W+12W;zugpIkZA1Nr$ zIp?0?c|Z-PWDelvs5tYI-4#g02)2y@D+oOE9diqesENGRXloytGDKwe7iQY)&=*{x zGH=a0SqG$D2DbEvyGx@%;K@#X?F%c>k2D3a6E7bk1D$a89Il-(Uu**%_&DVh8 z!B(i&(&Wh>a;`Ns7O=@XzjWZFw^$^Nuc1W5;GKPbz!^$|AvpUypM?^=@T%oeJwO}V z82z_t`b+&|))?vj)4nm%)6xGw+c%B#*!5PV@0@OdLG3z*S`{IMh5Z>kJ9Ecn)Ise` zt2lJm&%CBnX3ypWu{(~)ulwv>SYz??O(WK4;0#*IjM!bfmrOEN`04<*mm~3uZuxle zs=(8hHt4G#9vjy@96W%Vj2dB&GKTzf4$Cz5xz|vvs*`kHAATY*v-tE*~4llMbd@o1) zytvb7^-aS~9UFYZoUO3e`&4Ht5cCZCdb7l*f&oy)LamgvtVcRLex}r=UUUfTbeQF+ zUjy{FbZ2do{b_5*&(^<*%RdHGIU8zAxN|3)7Iuf};!Na$ZrtcVOjKKnIT$7iQ%;W0|-X;Or278nC`aUeb*nw=a)xoK^)g$Cpk-m(|XOxqP>%VQNHM=@@} z$5P2=awHXj zI8oR&5@nXOGE15wJrr3R+EDGI?#tu1JC>WQtF4SNYO|&EIg8!+j9RZwmmSFM5M`%K zexb-iOQa3~N|~T&n)x0p(aXc^%-4hsX?I2OGDsg+Bf2)%x*id z44zVxuBCI~U%zhJlo*>;U??x^Rd(fCmm8y28b}q=AI_<$=7lhf$KcS!l{RI!*ghNY zPy43?G|vdKbT+B9hcN&S!@dye49w4{#DEMaTr)r-%7D>|18jx2raQ&}*Wx_IaO7m8 zKWOE~pXFpA&d|IOEI~@GezMkEOh^UUK9>j-9_SIviN|8(gz;feD41Q?-}g(I5dK!N zo2{~s?%QPYkvt=lyzodtC&{=Ly0h+>cFA(oP zC0x`)&+dPb;Q`K6O!0H0L{FXtKkh!w~~2JB(P= zb+ZOsgBpcpnK3)$&_vfIPnl_+*an(H4;vUqnJzC(4;h3ljy-e``^?_6?^)Q;p7w3F z;Je@mC_ByYN6l1Hc0U%ZlO=~B5KW)68`I5!R!O`?i&t*(Lg1 z#E+xKl(zam&BR)qc#wHBr)0`aqKZ6-Aya&VukgcV$?hT|OwT`|f8^EMK9m1aP+wM6 z86Tw*V2EYeXtR8`+Vs%+cL|KWmaIF#0hMgWEe(gFR;nmGVm^*WgsP?tvCdvb0-*s>(NA?M=0X zC47tsPX63>W{U)hNH>8m7HOulf{YMeIz;5S4lF1i*|*;Lo3>eI!}7w1ConZaEOJi} zE(hQj?8|CAF93RQ=cfgK4a%hPC}HP9w2Zl>A?fxyPggn!Ut`g`p)KJM8O@p2-=uc^ z9`Nckwy(T*zfSP!1&O`Kzedlg^pt!c6R3L)pe;dcHR`{_1GGGzn>)&bFkIy7^ReybIAk}?+3)! zfS`B<{HEH1nf6YH8m+q(g{cp*uu0;eQAW59?wgY+*bhX>>a|I0zl7GkG+jsBNKX z0cVYPfQ&4fJ-iFHdl@qZDHVgN_CYv_KoJG|B7xX`n^wcG4!XjwmO3U_$pAmQ%7*~% ziXO-a#G($d0rnkgAr}_2R;YJ1>3;@8!aKmAqX$-LrD+z4B1#iMuGq34JK6@wL9BUI zcXpI;VMwsQmTMtBnd(4EQbIPimmOuOKQ}o&BmMMGs9jY>Srl3ojlx8W%TBq15-~y? zfrQ;}AfJXQ_`K+_)$M)@7-b#*~8eMhHKa?zP@VfMjFteLPp6~+4y~@x{@A4`UVAD)`!0@aW@%}vo zbaqEiO(h@MB}9EiA&&s?Z?DyK3p^{rUBWEIJ}aQH<(m!uyDUD=io`{hZ{`z-aYLdpe#LUIe%P zJDk6O^U(q$M4Pi(#T>)OGS#w?hVm|@hV{4#4@8oc*WwiCQb44#ey4eCw|%`Z>t#() zyM~>`W=5qnJA2=s(uU?g(T5rb z$WbCW4F?-v1+JL_c>Ne=*VYBo9iBj$P$y zxpOVH^8~$dNX2vS7C<wnxuDvv!e${XC<)t+ zSQhLWyi?KTMF2i|wS;EVN778{Cj_(3C`ODdLami#Gq`HIWR&eH4yMCw%F`^m6V+J> zy_PCCt2E#?N!e=yGb!1WnTqCwL=@RjDQMA^!mIz<@X=-OcMq7~RPn`?O{i)=ax)R^ z!aJNMtXG#^@mePEK4<>l+w_N~IglKH?Jiyi1QrV)XC?9yUf9 zfwnod_k>GcVnX3e3VFfAMA`1k+D}dPt#|n0*WTw_Mrsjgxfd@F4hLOD)95jsH~E+F z%%C$TpuMrwX2^5lPQB{GT?0O9%O4r?&BLn5%4~=0fXgi`)Ut*RwOQ6=D`_lNn1&#s zVq+BNoAQKZ8mm?{_hWY&zipcrI$&L-@2%BN5--Ma6x)6DFQibRgIr~ktK>vRN-s|> zeUDiyZ>!zrtb1zq?*u$&;UcJm9iBfA^v!cruTJEzOj?C+%pZlW4RIyP!?0t{QmE-V zi_o$APG+R>RN_HBJ+=-h^Nj284xRPWc;3%3wAit*&)!F-te!lf7}bRbvw7gaGsr}6 zA_8rRq<711=zITi}QN zY;PyF%TsIG#JG%n=2Pqrc-63VBfO$J##pke5L6k9bl z{Y<~sUl(pXlLxc;$W5s~kJQLCya_|_#3%=6ehA%0<(0~6FTPDQV;zc49FOD~n49;F0Sk$Hqq_gXL z_mw{OpUwso{eL@3XxW5mC2fUHpK4dn%V)<_^|WX7s-HrBDcy-Y^!2! zKtS1}XO$T;i}){`mu;+Jneh89n8w;LNu24Z;qKQvSbo;9JYEbVwK{nzNG9z}Y$yEl z!Q#>{O}W-Yf{3EtpgsXKJ1~t@7}068&?*rT!-T@2g7d+9NuZnw6y%c)dZ@=xBIpqw zb?Ow=5Td|nBjl>;coVi%)L%q(DeM~7Gvf#R8BjkHe9uC^j?F&Xt1Dev>Y}%u*W2Nu zq!n@22|AjhaY6g0Lk)Z(yE_P)TdX<7(3Iai9$9w`h_$lJsQd)Didy)MV72vzAbb>^ zp5UorKL$>+kR?3c1BGVcQ&O&(6N5E;YnE_9A*J2>6yzs zbrS`==y~L=_3qJxL*Ax}CatGBhLds&U&=e(v%AR3i^x0lf)_FQeYmX28sx|i?kJ!K z{4D&fYi-Ru`aa|mT9QHHtm-=GGlGt>yRy*GFS3UU?K&kCqd38gEqdj$2B#ySE>kz< z?lpWYi3_QQ^+(s~thHAgPk2QKzoJPHH7%54LCqecgQ`Ndx4UC*DrIXx|FW9vVs~yb zv(?ShIJRC_!F|WbC1OOCbpkVv!3eW9*$=(|1V*S)6&+y%Ah?(f8`I z`H+)B_x*Z-qh-!Q=nzUzE9wi6Zr|!U?H4T$MoyKDEUbX}$dwldeNQ0VL@pD(7pM4J zIN=z|VB8FQiNGtQm8&&tDzQC+3%i4C`Bzb6xEGXOddMh@1o%hmM`h88Ib6*7WW*c{ z)9EhaATlB&*kcNIBL&biezGKLNka6cmEsjes-x7;^)=(U+54SGiF9$ES8eZ0RWq#O zZ^{h2u}Q2LJzke+YU?=LfD{kJ$d%7LCh~_5gmQrPr`=(*CE>UT$F#$EOL-FfT9|eCk<5*-N18(`!N5aG{hef6rlpp*w({gr{lpRn*J|WcrgXSx zFl2S_8f(D~ScjmL`ZE2i`;ny$*XFVfOSk6BKNi1UU4;90?=JXZ;V-S7=dDdktI9aP zzH3&Tf~z4MxeJLJyXhXeAp4gk-`2kU+!sc|NoNIz_3TgwT7Rhl$8Vt_kvGsz>^tiI zr&3^K`yWa{$=%MFfL2Y`z{1$j37S^X*}&;vEh4rK)_?WG-|}A^^w6|&O2TS1f1~fl zGWyoW1mrZdu5>Ka3c60tPPPu_`c}Foe;+y0nwXn9I~Y^{6P$OWb+)nj%WFh!Xy$BV zNzL*%$Zuz4^6S65lG8UemT|T=_#4dsckkjhCbmN6hE4=bOn--{08J|(VCzPp`A@`z ziGiMgfsIKUnpVNs(bn0)(AbgSpA5l&kNjV>xHUZi6Z5~`fGvEy6&+#QI-- zi7*l{{UcNg|H4?<%}G?z>8}9%!}bs8g0@z+4vKdAhW~1zbyN@)AYl3@&GRq*|1mAh z?Ck&Rykr$8OUq*O!-ibHQ+mOMX4DAT-^+Z zhJJ_}^~m}2^!w5tlE!}r4=;oAHOh4I^=rM9p6V%DpcoZ)`O{!MOSbXD#|6Tu*6)u| zCV4?@4`D$a-FnaC$-TvQUPmKicT!GOiH|SSvmV5tN9wkW-|o z6vvVDgjCVRU4y4AZy{?AorxUP-S!i1-eL%ys^~=Y89lDGW+r0ik##8J$EaR}#wRE0 z$R)U@hmf8%O;{#W*LZ88zl*m~V=kGR1u1>q30J?*uGT|h;-tKP5Xa7xE-1&e;rKTf z#rmxQyc>_q@cVrB%yq~N=M9hZY~-PbvSj^!L)s$e!BSBHS(|$-*IQB`iR*P-LyD0+ zB|@iGwc9V}F0!?j#1NulaCfaf*A@60aXp0orp}+=yhIMfbpm9HsdB@p9vec@G%9lA zoxkFYkS(jt@)kDIEdpnkVwee61FD3vEvPoj;Mou|p(T&awC$WQouZ3L%d?_J3+-u# zOmO8EYe*Z!OB702iX<6}hDv}c#g7<}vB-+bzC;H-_T>KbA7{|J*w9|C4jH`A4K_g^XRy z4UPYbFf^?U0ptJGC=AT(|GQ4f%1&`rQd~t3o$h4qluJO2NSd)UQveo&>lBNpp`eFC zl<0{A5U~g;*$K(Vmq0R*@Awr8#@7)=1Ov&}or7WtE!f+kVyB23Rbg%aalf9-+{S?j8?@MMs+IK>)$t06?q&J9Z5X0_ZOy!0EtV zD^XVIC@&bD?eGMz?-cXAQN4ig+@v;jB6@&aTGa8~%c>R@#wZ)NY8GjW>}Xmulk^mw z#1xTw0E0@R;~kZz*HgMN0{cBf5Es_C2AQNZ z4C|+sVn}o%O62bJwDfW0-~=5!Yc%m5{YpBPI?k>Gz-0PE9o5|$tDcB+g^it!&YIe~ zi+sUX`y(^q*(F6M$oC3M2>+%3N!2GE4p3WS+_d@@YHf}N&^`x}>-Y?v)T0GN)OR_< zd)wo+d+8_)>`|FgR&H!)Ndo0s{-Qm(e#?dT71Z^bR_NNiKGb)+Ph?AQq8HR*wK4H1 z5gHho+OvCwJ<1Jz!H4*A#zRN?An6LDO%fIwFOedUN@>y@K?ws(7YN~d$<25{=>IBO zZA%GXdqJ8>Q3Ea}{K4mg2Oy_FB7pKVqMW(*h8YqCd7X@$O;5{ypAPu)R&AB~8bK%V zjWHAzVrWH{pSsBS&;Vv400{Geobh3wfS~k%HuoU40uXjUfceS8K<)MDcLGr3fP@Gz zQ2mvKvO&fKVf2pPFtFj%1hn)d+kto?UIoear|uxBLG${TBKaXA zaTG+-A))t)2*kUQKp_!U#vvC0oAa&3;T7SxMTo_hj%cES3k%cBNhQII;iCob6{O^= zqpFOXY6KO#wsSQbpqPMrXps5r2)BWXlb^D<{}PZ~~;9@yT{zytFQ zqv&HWeWh@IqBQ_FU^QUR$FK)u4Mpjf)T#aESi(7vfE#qEHL#*@hW14CMD_&ZjPVX6 z-gB~RX4BQ;k_n?6P~Oq7BePSrLutkOgS>@!9fdG>e=~k-{37Co-~-7UlS@dB5(QNq zBql(iPo1AIBf=)xD(S{gg8UdJGhkQX*BYWG&LzYpi$mH)DlcY2!t%?XgwUGsk}R8O zlcW8XD;D3S+{&1g&GAY`3YSrPGkPMh4|o5q2eZLH7oS$UCq@QBia2U#9iT+Fgn-a5fs$Bg4#WhJnO9&aoKF;6lPGpU+g&Ew2uEzK?6&5zAh%yASx zDrHt*=4V%Q3cd$D6~QYHON-_;2p$(r8HPajC&o{e89mUKI59haI+M0PwgGSv9=l9?+nCx6x2zkw&gWR*8+7+}Eq?_D76y*wPwLOj_R!DP zE!~1g4nZ4Qn`FBRaJsWsxo{jkEUoscUwEy&Vqla=W=KYk7rGDKzuf;gj&3_^bI>~1 zF1CSf5o+nQQE=6Ejqwb82SM;g$PrB(4k|4-tvKeNHq}|uA*z9|S++8Ll6d-MJFq_8 zG2l59x`%S8cX;{p#%0P8UCt^MfWx#pZaKCUr zbFUQRjcTLKj;og^pKbSzXkXYpl7?zYEvxJ01$mG(i&PYOHk>}3I{cbFKMQ>7Va~6( zNwT!n>WbzHB?rP5!ZzW0jP_WTr0r)i8MlOs^i~d6`d*Gg{GCz{x)MxI>09YiahD8? z^m^{oFYce6+#sp(Y4%EDNF`A33{8yp451AZ4G|4R<5{VfS}df{$jruqtC95`gG{T_ z3w{Ga^*Rh0`zeZv_di)Vp2cz;a_DUJ+E(wR$I0%z@4I#UHF~vYHS;uUoAsQEkH_AW zXDW&-Jyg$YXf(LHhHNMOn}V9mEq%!wX{czu+GtF)W#d(5GYaY+ea8r}OMAk~yOKMGk!FZ@c-plI&@ow z>I*MG$(1?E&EiRRK07@@}Tz`h3CbzV-M~2Q!5c zqhq9f(SBf8?%3m#w-51-&jF4L&KZgm+D^-a`c~o;eaqt_eA zX}D2G#%uij;PdB%@Kt{>XiDgwcwKb(r#Bfbug;6xhxk%rdg&l9<+s-R#KEd-x7!B; z9U3R|m)66_d`+fqS@+8K+?DaQ)5^oADprk&PU;_dU&kHlbL;PpG*=YHw(GiUO5QY| zlUK}#+pE$j*<2rx=b-!d|MV%@{^v&YzYfIF6VS6UG5>q-$w0tB&&I&^?i! z1_pL^hX2QdbB*rCN}22J&i1a9cpFWv9fJ0*&jh`j#jPEl`Eptj9f}*jt~9iG+n>3g zFC2$H-=m`rCcT`x9JwuxN%9HgmGlu=z zpGXBBya^S+qOjYKXa;LQ&+6dgilw2Yww|@nT^0$9!;MW{T`k~i;boxbrgSdX3Q5E- zDn1-;W^#21TF=N9I!-^y6qFpWxfQJfu*l=?S(%s{S;7G@L6l{f11~5IBr5|F#&Q=? zmCZ1>VJchM*Z|pIp^;?~6&WPld{S!s$~bx@G6-=>it?ZL6&5DnQ`-S}d3kSF-}A@Z zH?r-2S}STRQVQ5$=5sv(wE(TjtgNc9#!p(Cr0WU5cga(Z#T97ddnJ(H8ZxNEj=q6| zqoZM?ojo``a5fbtuGRNmsfGC=tQ`o)DqyQd#-!QA7i!EDqRYqPOUeDCX7=fXI zwVu@-0Tjz$D+?&ZcXcb*q(sU0?*XN-b~g6UQ^IfB)cNn`R}5#Qo%ih1HIG-!w$JAX ztkn)S&o9Bu??2Wkd7TZ(j3ckt5hegfO3en|i|p^;>B*nEEbjVX=1}%%%HVC!y$z;u z5M7mq(~u4jpJ<<@Ikjmx2!4_2AU%DP;Cepz3ih6g@_E^toR+39ym#U1o#&vy9PV_V z%S!7jAe$GRU+We&#ncDMc|D&Fj9yy;*y|*G=@nf$2adaDYs%ex^wI zYJ4*P@|~daow!DRabpMI0**wQr%ZTMj!dpG+-OaSRT%*zqk|K}=eaT8LN`=d07Xia znfJJEThy2C3d!|M5TFg;%|BY!hz~rl3*Wv^ zE545jc6Ck4boGpdU$Xq)oMR(XV@vzI`(Kno{TB$zXWzuDW9ofln-fa2&_)KYEfv1& z#rz`Ig0xn)`j$^!M84?@Ul+vd&;H)LPQSmh)B)h=EHU4Aozn&3(VgiHjDGvqObxDZ zUy;`xH@+*tX+pt+N`q*pUzS>5(8w*UKp7gVngF#`c7TYCj0oP%#5H5j*cAMYfjhpj zBI9#+2x!lM83Jf$DF8={j|bqGhw_mde6Ryd7ye2A8Qlsbwf7y_x&n}@_Wb$?&9BMC(iTk zcoHA`^1%a8PX5;G$8~RbgOxuV3qG3k)4r=7zHyp-_0i%`W`?Bt#~wZnkMIfaLFFJsK-AjNId&<2Jd7UmI!J%< zJbta2UlZTbejJYgrKPuJruWcJf=IgsRw-f&j(ZQnlv3o8w_?ds)PXS>?-8all=$rU zqK>Z|ezQ2~#M-=4hwgjh*6Qvfww?W%4eQI^)pq=2%rqWn_B-n;FUC*S? zv0BhUTd>!(Pb-E;z<;cxC=*rFsTU?VT@N3bUa7 zCz2}06e{p7uN%{Br>1k6bCnzDA4q&jC8e;y?*-&ru*6 zomrjlxR1R-Ca33&-#GOg#vuRi+HMexEh;dlhB@?1tY}zK+`cqXHy14Z3u-+ZxtqS=Aq2PS(H9k0HZgbGvw*GY_6rP}f;ZX)v*0EqoO-NC z-8`A!r(9RT>jU#JTH_`++iFltHnS%*9__A^Aw&@lDjV4_cMgbv3y6;O&;taP^2c!h zV-lzNwGmL5)ZdTySa^zr4bG3Qx*&5HO7EUo_x8CuWB7E*!+l*c$Adf7i+yPI9Jhdqeak2TK+7J zjQ}Ajx?(#OjMN6`IY88vACS6)GuekwsumGEH^{GAnd^1Y1);w$d_ZrqN?R}9+R<5r z9xq&`Qg{_3#I@tfNJsgTYPv{qC!KMgM;Li&^MXejJ5p;2NRi_rLvRaL=I~%aFD19Z zbHEacnN@0OPlDxjTbVSX_8t%n1)osFAI~L1Gw1A_GZMeqg9%>4hLH_kZp1XQ)MnL^ zqhy@u8Q5$8oIs4nmkHIqFB0hqH}N!3NBDdSAyBrJl(m=LJWP9}X7YXG$46Ksp&$X> zH_>_%REC3y$Y`}Y3+PWooUPGrhFCG3rwE*uSu zzM9cYT0x4bLhMDOtf;=jECZd90L3sYv5>Ple8<@!*ZrNJKr}Sf0-GV8Xrq%4h1}2( zNiuxY*|!-s}C|7>X%iSty|HNH~wEU`1-bPd-a z8Rj0|5CW=#NF(;6w107tyeo24$Gf8sMf=oVr+53JYn2L}8TlGE#EF7ARN7w7d>|PXp?>J4 zntDfEIJ}|f%_X9zM=*xjiHHyLxT93E-0mWlMMAusWB%4Cw8!xaw#x~g>75MIt;YWL zA=+#e+-F*=t>IXA_g`$&tGZ`D=tsC(#V z6VrZ5?9#J=|E`LEb*8bnvX#HW`nloC=(3pk(_i6NrIi1!w^vFWyR<%jxEA_!NJGm) zRe_Eeq<1+X4lBM?5$YS)MJIm$9$j+xi@m?ee7H`O+#xTXStZjgV(PjisYY9lQlrih zY5zQ-SoD~fkfyf=C-rBLg}hFDT3t@X`L?N*>3v2=mQj{ACw{y{o0PNH5a|#|>V^gH zVvlkHi_PrPY73mw$044lq-+puq@eI;P=d+f_}M36w1O|SN5aNdb9`6Dxkl&fFbG|6 zU}BUS%=l@$!B$vZuYp}hm(O?BD{%_Q-Adk)mz2s#(m0hoFc1WVcGR8`hmZMs)wE*C zQ=wmzTt2aD?%7sg>Hu`Re(y0je##}LQCPEiSe zTZLq7nfpag;vSTM6eUi7#}#crS?Ugb#oeA~LJ|i2+(wUcdOk<|xDg;q;q%b_3=A^W zz^3LwO+>tnsnJ;oIXX@rgAU}DRv`AoWFR(iKhw4AAj_^!(gQoneP2OWc4mB9Y-Z1G zoFcY3EUN=U!J#-v5S44JbghTN3gX8SU-)9I=Vi~6sml(zckwO5(n;o(Lk~R(_vc2! zj$JoN&UG`(ot_sRk|5HpRycs=C8IB0aWwLd>nU^ACH^?LvSfovE>-@Z)T`0jGyv%m z(0o#XD0ZrFe>bZzWu;VWwupEJ&6H{>!8 zM$X|-3D~IGh+jp9&Cek&tDiRJ`|Ogpz~+XZUH;=3vD)Fv~d#8Dj)PUI@1IN?07 z!hdMRXYo8(l7@%kawjpzt=vHSbwb&P=}ZZB+XxJQPpE(V8CSE}Q>uAT`hM*iKE-0Z zt>^L#j(=I4ry_v*%r8<(x+&q{3Xy$jV5HRl&a|-@Bk!ViK4S0v+?hJ zc8`e`I|umDwYkyowEBzraOZ3Z5%g{sD#m=^`q~K)s zQ%YF2mP|0CfO{>GKm(>iYJu+aQQJd0BE})kCZR^x_?ck)TAr-0VAfyI_cY1nX7;DD zaY7u&uBQ_3j|Rdn%ALAmwoLf~DuVW_BVyQ8BA2&?x{X89VY%EBg(7B^f+IcqDxM}} z;Uwp=TSK$(#VSMUq-cf*a%UlV2Na;^$FguS?Q*C!uRif84}EWe@2ZNWQAVFvImsOh=eJRUC^-4`-N6d~&( z-jLbqpBtTE_>H%d>1EJ;MU9t1j&J$vKWX7ph$- zi6b_CZ3jfDsP>$x;v%{6Jf7X)&!TwGe49ozCL9>TW-JBZsCw`LRy?ir4A{>ds@WwY*N$G76B+n50|rp()3y z_gd!9l8U>EwvUVMZh`CLJ8pD!G+LTnMYu#jRwx50d@C1w&nhu`-(Z1ZJH$!3@8=p4O#czo_&5=5JI@XQS)c@>_=N7IPE_#{)&?PPP0;q_v)=% zQ3DF`y1U<#DO4jhis$Gk;9L*nl8qr? z&o;~m(8pnI=@=%~xEo7@1(l)*1~(kKP~`bEYn9Ra>VzV*r0`(dSfiM>a$tdv8vha~ z&pl>pAM=4nS5T^e^pcA9t0fny!>>h>2795c8a*^(3Gub4(152FQigmx3jE4|+&=Wo z&q$k0U5nB_1r+D&Yhfpj9?}b8AN7XF^wGRx zWA<v`r|NEe(3)BG2DZ$;*qcsh6(JIwdayvMl5-SCHY@;GcNFoF z2-S??wiX#S%7V(2ryPd2_?UlTJnw-u%>78a0#@ICg;~US5GJZd*tU8K=aCK6pa)FBJ^}(k#O*QX2D@9xE|50lq%0OYxg{9Pwd5q-G zbx_|np+GXdUKV??BQzqxNjD&9nS(HbmdWJA!%p^FgZ+Ryk1m%+@MC!E1RhIL`3ewxGYh`? z1<|j?DJ+)F^u=WnI3#qB(^M#z&hBa8Lkw6mn7&Z}qps{2)5^dyxVc_)%OnRJp7!Uk zmcEAD=c$3!CF~GB1I@~T%jstPjR@-^v%=m~&jsVnFGm(-)*tjkw>|@{7qvV+*1**q z$-352pQCRcpwozc;%R6;Jubo7w)uo(ncl3rlLsD29DAj za4-s|Qrl3NSbXiBOc<(3Ms%$)TJPXlobF}61Y3#AigT3e*iuDI-PUQz+c*{0;^(Gv zQb=llekLC@a}26eqZP1oL59UkD2&`Q(CRfJQbmh*49-_*O|u~5TXPv~P+lpSjrB#{ zhnOR#{&AI})|{qBX!&Nj85$crs$IubVWnV$Wy4#4$qP2&eG2`Q|X<5a=X{#J##S?k3J(B-a#1hz+)}Vd9WZwcX!l0#C-)rGOkSe zM(4@6wXnYvw^_^>TFuo!xiZ(1`m--bKnRA@`yH8;a4BVgMbe1b#HU=P9n5keVusW4 zT5!)l)a#&>BcsMOqY=t2#n7DUJY2h2oAlg^;Bcumg{&1i&L4InnY2&_L&hF>WsBcx zvtlKqh8!7gI6m7W2{BH_3sRLL$hAN$*;NkTpcN1!2O?(MOo!oIeepv83UPcwtF zG%NqmogfTQPHjTS|M~GeBGujr*b6nve_)4mP zbkb6suL5ofRGswr>UTixU_!x;^0f3bz&L#Hd+Eo1j|1ZT&xYnXK`aRSTHbl#&26`u zP8grnYe;@uU~&qvv!~fynUxz1jz6b}aDDCjPn91>n8q3L>C>!@NFY=VTjBK+rekJF zO_!z9>i1&ZMN-wDTc$ zT^BU692%46A%TN@2`(wp4EXQ;4zQzf!#Q&c+0JG0Ih{m{uM%L1!$1#*y&|fc{gPM~ ze8xvlb3<7#u5U|&U&vsj_)_-Y-s2bT%xH^b%4_VmR%olm|DjBX?t`93)`+G|b(t2v z&?~thoL-k<&b+mPTkWOfkI3OZ%#rYg1YMnY96oTXtjX8#G>R+QuOD06DSK-c(uta8 z3xA@94buco6l>wD2aji+5rjTZI;jLlGmMHoY#n_WNxFXvX#<^>%IuUrd%dOdkm+sg zBsRHj(OpLGA38n51u13p(IzP9A9yyZ0eiZbgGlYxe_{pput`aKnyT~1uru+uwSCw< zWAC0NrM%2&5@s`C-E17{Iki+`Ii1LJ#+8cvV)l>biQ6~$ho^^tYiiem(ib)+s_5hg za3CLN?wKSjll`Oy`&-O5fN-M;c6@UW1CwKc2dh6?#99{N7>xvvGJ_C;Gb0}&Tnp2f z%Q49h?>E#i`uB`A8HWX&lCk3&_dREXz8>mteF&g^$};1R3?Kqv#bjiBzuX>~wBlr5>j-=hL&CuZ3O=0gO z&K{Ii+FCZM4^8T+DdcNzfQu@0T-TBl;U|U)8jjV6u!gg#17F|pT?y{9lN!C+;&x6| z6;`BDcM#yBA)b91Lc*V3{U-)fOLmY;QoIYJEvG-m?*kl+qP}nwr$(CZQHhO+qP|g*SCM# zv`Ldb^mXQalG*3%wHB3i(8L1DfkFwbE#U38L7lM+iaHm}V^R(dfX{ek7~9{`9DBoC z#9T>MKy^HFC?M22{c3#2uH(GcSrBCXHf1*`G6jQ>6nOJu$Gdw)_&OmtAxDNt0=pX; zBQ`_Ahs!f5>C?zb`GqNm-w3`yuCvwH2vArWq`!YZAf%7ybOph;Ik@T9AQ>5~%gd7X z!#I^{88(~w{xuO@zuG%V}NFNJtMJp zr!bT(2vEd0{JMl%Ubjm#_9)oewVUH91$*8~E_I=8gB-7hDK#$7P69cR8-ZXogQLP1}-ST5#$R zJE~tiS&yEth|H8w>aXg@;5R0_eh6%UH4U!vp{haTqYK2RLCY4Vxun8SN#>Kfm$8aD zE~O||*s-?U)Wt6xSbHE6+Z2@f18y#Y%VG=Xr*P;}Vj=NWbKtl^4`qiV!yo(h_0}j6 z7;T3(?`5*vK71+7rRGnKr6OOw!DyNlR?ifZc0JkhFH#=oWma|?zGgc1S(yk1l;)BnaTuhfJYQB#RKqt=rFI`+-&vyTxg@=cFL=)P<&)23Kozr%Wgj9z zV5?D=3Kx8pDTpSPX28X8fUoCT_LP24n;w3CO7Fhdd)VSs(z}9D6-<|+X2^z@mat$l zswDnQ%ZD<@^(7}K8MXLK-1$i<$6ybp^;c}{O8jGNGxEC9qQ<>p1@TY5k0_qppU>%L z;l}Mg{3MX&f@`^iYPZ`#G5*VoEWIdd8+rqM|*uf?GH9_I-O@A6IYXwFvA z7;9iwfBSi8A%J}Fm~~C=gKRJ`jPz$L9Wt2g7`V1c${ACZ2kIw1(M0GnU`+MQTD>54 zxTzHL#v(d&mqYwYwx`CIq+g?G8PI)2c24{~pSLcQH z<2J=r)nzNr(35G^NFo}2!gso<$zH;tA~756WDfU+P3e5jKH}(rVG%RX zkC1l+PKW~*ybY6Y{TzOJ6KM+hOl$~0Q;1W;Kugr08sJAnQy5EQ6$%5|Er#F81+tjX&-L$PEZI6fV&N_GjkKWnBiC$c#fm5-C#OP+kWTWiCY=Gz#QDcR$ zCmeAa@!^)_XKg4qom;Mqo*({FZfPXl$061)fa<1Gy^U|!Uqoq9YN~?K*1yN$QJSp7 zRXTE8g)e`GZW#Pqc+F6~jxKdy323ba>5^N#8!8JDm_m3ufeN;4mfovtz%Zw_XtHU& zxDqp_j+(L2I$F;_?EbMr6CT#@4XI6$;7nJ5q>D}UTGU)v#W`)m+;RCX8Q)1%{VY>| zf$rMRpPyVlH6K1D@}p$8fA%l z3VE?HM|m3P)dRyAy^wVBsDs1HkVjVcEB|atEriOWZ z(>*BUu#FzSHnug+aP6uFM=Pr3V8q;Y?GGlA3^Oh&%n zRgVv>7<|*oaOt;G!#;aiWd~j4nG2p%B5&+C-g0(0DvuECQE&ni3H9002USgfyPcr* z&GWpWUeyx-ae|>+V@I()0Oi`(Z%Gi1YQrOFv5AK0)UZG-yN8l~k0WbP>^Z}7^}ops z5%cSd#S}^8@$?()p~P`*yAyINcv)f;nSvBZH-S`ndKIMq#X^v*mBGOdUXBut9$@t8 z(YCyC5abm-5j>1_1DBW&;DE7?Z}3*UF>QuXrja}(u=h8}P6Ac?ME{o1mmYiW5t<4N zLRHXmG^~~Pla9IouGFLXxtSkUtX|$s`lY2~Y39BZnD+@Acvq33&qjjT18}0R#aOJt z>wjrLHZCSa6pBLLE-e#z_w|)X6trg8w;fP86+YMMpsESH=$;1qgm7Y9hxX-x+b0>DO17|_fB3DqGgpM9x z?H4B~4@T|8)f4@)U#A9CwACA&&%4}Y;0FWf#+iIK=JR*n;$gk|fWmO4%bl4*j?n~@ z`LcF7tn1H9L3Giif0&B^Z*ND$(g#G(sGLtvNX5lc>h2o*?-)ivT_X+j?vri|=X`VG zQMc+5NPyaZoz1wiu*GDE5^Z{;=mN^1b1(N@){7bA`E?R~NY}W>L9AIehh);>ku`A> zgA?iuMZOgdX;RdaC5qT?0x$KQT{o4-TDySe+&t|cHnXjX1Qm@X;9uvV`<7awlpGfI zqAZ+KD0v?H8fFT5K>wq^t`N;{HpssPK??pB{VpGmg#lt9)%mebe2&)9;-k>=*8BPxS1xmXH1t!z^h zFFdLr1f4Y&WKUaUETXeZwnZP0p#6bg@4uiXoGv?y$bxg#qlAwTox89o*Dq~ES>}S6 zH=1Ej!`Hc0ICmsMz5MK?^6#MZk}oax8uJ_O+_Iug17pqeh)9bnig z^IETPed9TcMxj70r$)xYe?4S=9rm(PA=)OMpkg&ch~=(|)k^5)8)wA&AZ($KxiI-F z%E347Pi=Di1Ef*l|1~-Vdz*~z$B+CchB)3oU$?+aJNU=c$UKpECOyeG99}m8EbL=6 z!r5FoxQ3mSg8l#zi%AL$4__&v>nc}ICB;<0sTRDm5hs-eUg<7W1=4(5KciwIeoAq+ z&P#|sl?@h%zn7Y9Q)`zTUg(x5>b4I!dzL908-&k`Q<{CEZ{KYv*f(sYIYeF?b!wGi z@laEWDW{m&;Bhwfsvm%o<=?59NapSoly`>}{JJw+!CwMb!n@!*L;Lukp`|6(zoUjiFY9~>w z7kMt2?FnFJYK4NgV2%^^)s4giuQo(cJKjGSLTpzw zb)=2X7lHzhfZu@C^(m%NM=2OeyPjd#iRjtcxdE@%&bY{ej09{O(C7I0*?P_M8iuqiV2%W$pZbg6C_GXhdkGr4J^}+Rp=nE^_ISu_ZhX@e(Po<`< zZl)m}v+ZdpY+xf#ZJVB!0T08S&{~R#kGR0{JHo%FK%}aBGq>%6`b{G0u!cOPtTj|q zU!`!OwJ={*xcTzl?(t{?@s#}8l+$GM@t!bKg(BbLTp2z z&vH&!jQAUf)zwsB{mVK&jHYKj)jJ|;`w79ZY+1_CD5#hd#VZGYJQlFrWRw)Q(^(w8 z#7e_|cMS6wjP&I;c|iTcF+mklw7BRrxk(I`UWvp~d)(lS=0Cn$aF0B5b-Y$NiqQ^t zFaM1uc3;i4+iPmX_34Y$xz;>+u>Jeos(4(JiL<_@h8%aj$zXE_DoXJa8lmTofGWo> z0=c%Gm@Idc6aAKV`x}>+5=yKWL2X5^<+Ysb8|G5OWRY;-4F=6++a4Y^8iYG zY`_1G1UEXlka4>`SE}|73Q|&=-n1hgf7;CqARwfTeCyu3F!)pI*_mv)1{WQYtUh=3 zf=3gn`=kK)tS4r)d@2_lKC=B3$zNASr~-7Hctd*};gnv?fyP*|J{H{vmxs8Odhn{~ z2ufmxN<2$wORMd4n^Hc@7QN;QGc>x+6-qR=IYx0?zAt^9@cOr| zSme_+x~)Fm%zKnvr{5=3!(|X)Nd^^WjZh^zpU-gO(V4|Gr$1|tbOr2FuX1cH&xmi)E zKuwTuQ#ZsFSg_lz3cYDU&?V;S69IoHn{3fdOxdg4egNPv1_ z=f1@hm7q}?(8;3Ji1<8^JdwG>`gsDoOWUSUGx^hkNzevlwZeTEIL^$Tju z(P(GwLnABZQ1^@C66yp9eRf=vQs6oUYTA%Eq(lf$3N+c7RK&Rp#1$KQ?CCngK5Om{ zH~#fGSbDoLF)z%T$a2tW^fO*wn)!{&yMLcb$KOn)D(!$5z|B07q&j#vDT_-f#!ima z>5m1-*u3b!V84dgv87X8AOQPh^IGyDMe{;5-_)Lp-th172t3vcJCGnQabf~iDAnRy z!-v%8y3P>f+3r)mp)C2zPFF3|4*X;yZrX@FOkXrC$HngfPU!7A!!EPN9siZtEL zu}j!h)>LuTaF3Aqi>0+3Y(^|RWmp0^03 zF8Y>-%I1~(>eZuez0?F1{Y2JxgM!@bvPLg5Rv={#(V`!^0@i5`qeAb$?eM~1F#}9H zb*1gWbna>^`s@f=-#m@+d|S2lG8k}bQgaH6F#bSmt1Nk1Oyl*Kr;ZyH2pL~UE`72?!9BvTj4N8?dI1Y69Vw+Vvg7-GUb%1+`{ z5Jk+q|1io9S_%z(_Rh&N?hu>NRRX@7K`GplsKE82vlRBKB{w?we?>lg-=nbl=fnS| z#OZo#q+nf%{~N2|(afK3z_MiKdWfl}(Q$hPG2d|-^0Ay~`MPvu)+4D#cc|Iwz0uW2 z5Qluiw`bnEab*E9^x$<3wKIZQ!aI-{Z`-9zps&%KfsE1Ht86{w6~4=-CD-l?Eubg;;}u_DHuTolc@B0MA`b4y zmwDvnPDBN}ZB*a6f}86<8gN1_(aA2-?~oVI{2`!mjXnB93Y{hVE@8aJ(`VGbUg%aXn+$>t!c zn@Y2bWB*qOOZr_FPwgHwgq)s=yqqtehh{}_z2Kx;a9a#Gqc4DEhM&(V+LpJg46V70 za2G?%w$!Sz3r6Ta@hOmjIm$Q~>n}O(u}s`5F%fOs+s;$(s_(8?%`0SOF%UQ-jF9`_ zjoFX=BPuOR&ix8bNDcmElFLm;y259AsC8)v?0Ht1Tn=h?`b+a~gAB9NzM<_;tBaD^ zKH!!PXvI3-I{1KUk}GCwa}V}Ao~N=hTpmN7-&$qBUJn-5v&+G^=NW4g5F_2cO!BalA%1^&z zQMHpuEo%tzZ$|2%KI}&di=`dFkZhM%V`?7?b6l8c-ScWeY+a@jd%%`cj)?h*OM=+J znv8v*3vG5L{M}Je^CKh%)I7ULc*;Hf1}UgCqn4OpD9FLX!KuA~`A}5Bu*$x4 z7Q*4c@DmurmF0M&IaL|;;YT%BrEAJa9Kn}p(QPgxX-8QcMfw@bbJs}^mHFZiiFz6< zY};wJY`QfTsZoO_1pb81LRqjc*h6i&Zm`g);mbEu;Jas%{ zHeTOvn5D60p5?oDX7NxabAW)25m0d5#L_0)1P#WN0LyU|Flb|`o4tCTVkTtVAbpot zAjKCqfU0KMQji%H+ht2!#xx-5=XIAQ8HnK0yl^w0cMkjDJI`RW z43M;smEngdzj+4sME#^nFzaQVP@E^Gdlb}(Z)IU7hV*rO-=hwxG`98?e*cFA@#Y&4kh6xb%!cb;Hl+50$F{0`xe`A8;Nb2P1XOgI2sG7jPQZ z@s`^DI{CiD!UVdyyB(g7cn5ApQt6>LGW4-8>3Q%Gb50&eN6iE@_3=`Stud|Emf& za5Mw8kT$EDssw&id#YwT9fZMd9r`cN+a=M!5Y1u9X)DCGwf(rlD6-`1ivI$T}N z#fuliP+AD5AVJZr^X~$)kwra-_%tJ>v?;0yKUT{pV{N^jx18Ii9WJKGwia&WvhO8W zwyona3;h{WQ@e}pY>`3&A#|D$UqYPt81c5+8f@Ty)o;I9?0jUg<3viXQit}W@$y(r z4BZQ68=Dev?sX9mDW7T%kGnxZ-)?n$L%N{)eAU$|2o8g$rnRg2GklA?ZAn+&KUN1CY1vH0fP$*M9mtqO-NQjo~uc3F2XAHF&VByK2Z%6^8W zfvJUg)>vz9F%lL$l5;FU!JC5g_jv7yFcfhT=q=H(Qo*hw@SN?TP_xoRv+gAIlH7Hd zy-QM5DoreK|9CpmT`7Cu{UfY%e<05gUnIT;E1afc0y6LGiaHUN<|8U-+f3fgSk=UV zSaP5l6P~_A+{O89-CPeu6Vl0NdKVwe1*0MpQ3LEeUW2NLS9?|1#|H@!kpOsboR7EU z0slpr#V%6@51gnLMDbxbuOPl1uoJBph z7JJsBIvXCc8cZ>>tT>MpvIJ^^^q-2L_Kaai$G&UUtLNeHgVB)14;~gQtgQQuB7iFx z+!0-RD?)lEBuB^e?9%|1s~J03eT&>LwS~!vxv8RxB74}OLW9#5USj@B@W_K!V_*hyS;k!(M4{(9$|+rw%(S`$iM8}vTy2MKGyg& zK|;Wj`nDENDaL=-4P{Yq`C52C#%tTam6=yh(&t?(FtJSBv z{}Q~B7VBWT-7n2Oj}e=@V%AG3XD6$eueLdSj*XC#p}klnmhMSLy3;oJj1}lqGs7V7 z6W2ARcUy;F(TM@CFnW64<>|%la;M4F4n_N#*30(LlX7hW3bA7Rrnrz=AS|`Hhb^;a zM4%g>p*JSPPG0ZX`d@|7hRPJ^$gXhzT?x#MU)Fo}YLJ$GE9f_d70r8(Re`WpQDj!g zApSvLVPC4^&}un*I}a^fGQKJ3-pn)8ezB+!o|1Sp<$XvdMW9IT@tz#9dg%`=6`wjT zEon>N2bwj8bpTSS2EJ&fX8B|DcVf0(2G$H0BL0oDV;YX7=`SEmzf|{Rn2)iLVa>0Y z68h!6+UXwjaz+Qe!qp=zO4>U4p$73ApEG0GcdRNp6`eBKh5IF}<``RqyhYUNiD+!> zsflZJYly5RuRKmj62Tr;x2dZoE(HUs;h;11VA<*!x7tIBXs5zL;&h z1?H9Zl5c5i08L9YY0x7YcsAq-)wnUMt{V@H<` z8G{-3t7oAfPr2M>7ZuICQ_jWSA_5#KF{%g((WJ#3asCJ7e_K5(XBbqD9{KPXW0->p zeIO_O40Gk3Q!J{@C|&LBdS`r&Q?yd<6j_J~g&=)+r>#O41U?5``(D)^f3HJ4Idwq} zqFF5qVzS9N(xpJc>{Y$)Kza~Pt(3u18>XGUGA8B%9>%NnAl4d)O75aaHLh#d&NUtHj>AG(}~@2T@0QryJ1=eB5B%p99so`T5T$Ry*qzZ_?*It2eN)575NvjAYnxEY z_R4gLM52wf$vu=ZeyXIAjW?qhil}i`qSf+P<3Y&y-*?PLVlOCuM zzhpELRhOgtS&%$t$&mrNEn7L+Xl@&dBIHiGAe}(b$RP*{m;!JsVvqb7#a$DfNSwEUzg&fq$$NU@j zuFlIv4@zhxhsdwxA&%7NW}{ZZeO_dLXM4o(;Q0h>PDamhO4*6iSyb+;{H!miA zHGEw>X+$jeEyV4L+JahDp)K74p>UwH;}Q zON!;H(!Mt2m%ud{Y5aJb1Wy0KIN)0Rm_<5;x1qD>LtC#4{%Ojk+Fjz}KWC2p36&&u zv&{3+H^t=HOp<97dD+R`0}dJjf!D@C0hF_(0-uC5Drlv!#GwmNcbp_2ZIBF1Gwx+E z$AeDek`r4}NrC+8@Ojd`zgUo^LupAgiL`g?3iM@l`xMjXsSMr9-xhN`FXF%CYjtOS z4pKdg-~<(eGG`-5+CMna+=V~rFbvOgVvN6MGP*g4?oD+bKYALXX-e?ch0tq8>FE8g zvt)1v9J-kxVIGxE#5Hu1D;xdrS@!1JOoQj^44-4Gcd~+^OO71s=#Jx1&!xas6iQ(d z&l0rTMAy~H#H=z!d$|9YpK3g8u~vV4_F-61On1b}a#M|@IpKO6){>DLN)}Ioxl6=B zew_vbxIfxH%w>DB+0$ba_4OAzl;;J-$am zKIoEZx%J#m^NX*ZFJ*KN+O}&aVT~n%2{5_e1ONUIWdP-HD*%Y>zgbheo*;O0{|3Hl zTc8dZDxUzTXQPmnWeAP)^FJ1watby$06@SNCr@7(8mYn&XYKk^l$+j|j;&Ceo&Wgj zl$B{GP@&sGhfqK=C{2LeVaLg69Pj$`t_HaiRw0`-l&j(Y`@46C!8xRbT$yWe+~y|_ z`{)lT(1~Va++xmqRe$@6CLRExP`Qb`S`#+jZf^8I#)t54p9aB`ih>f5k zR0yr^e+CgZFt~!H(o4MIK`X^eIF~Y^bijIgwI8R?f*1bSfOt;4XN|>P_=Y{teOvrM zgq)+s7Q|06MB%(47Ui{iTSJvL1>O{XLqYe9{>2z|q#S^|L#kY+`k4gt8e?!(h#O|f z1KJh;?1}FS9o@!{rnk65+L@^AGJ+rV?-yN<7ax_!M3O(rfGMOKaCnck56w*7vUu6J zU`M*8=wa3cMCp2xn{g0|m@^t79J!8EjVB{hz82e{S>Shy`%xhYPlqeOL@?I*1Zwa6%*~onnq9BcO@O?KeYu0| z?|X2omIdK^7&cP4j~NG9&VOap8Y&iAPZg^`g;sIuYNw$KW}%sSZ= zKWb+Z7b7MaWY;r--v!E%)@d)^6Pg%*|R$i7LE66%cXa}R7GxbL}sFcs%iVAN^4S6T&}n%E)OUV?5FsN}cm5-b)zp3n)Ky_J9l+1gBOGb!7pk=mDjYU!p(y zshHPdwK1g+2Q_~6ZtVOrfWz^0b1e;4IDCQqo>wR$(W1hul=cE#)jWYmq=ncd)ZILhYp>?7T5*#Kd)t=I1l-Q3d_N;)ubI4!IcEFm1YfIlG$ z*W${T41y&GdaIsrG}%GB9z6rX4C)1}rHFGp%_)8NnoxZzUYU~LgH$x`h*QyYF0sFAc%DVodl7xBXjdeAF`(PUAD+%wI~O=?ZFyhPd7BZA=kw zr48nQ-x5q%$UhE*?bF1Cb3-==V`YQ~XOJ<2(iTLC)i# z>~iGJE(Od0mmddwC;P!JY3Ks=AeY=+WPf*8n!``5RD|pJyCP=;T)dzR>8N9@ zW8r6C;93c?-%rJj&zt*ja&A|Z0X$F&l~5xhYJ1tZlR;SUJjh+B04Bv9bUm+W#tJU! zF?TPM`032#us5_AfnT%;EscCu9){@{SZR|o&zum+OF!f(sH(=ai;MO&)mj!x3eWul z)_d=AHMp-8%F_U0k(!m@eym#lY?q<7RQ6ymYKI+2h#`Mw1Ma(O{`EPpM55!YTQ5xM z6P}sl(3H#HoAN-fqo!m;Y4CDNR&AVu#Ml~iQx4r|-9s>8?HQ%VCyzuZ7v;le`5_zA zXoduBwVB&ve|9wgnlGBkdZdt?9w_qg3QFT_Mz9y|2o11gSpPDYOW6De6ar^a^OnfT zhq00J;V|puBDClc@5e_Z`U-fg1#y;xn>wEZK*CW>KClc|1xqK%12}#$!^==8<^M4k z686PudhZvool#^aKfQURhdf=>`S#~Km>^ja06wTNn?w;OX6|i&b^;~;_ ze|b<@RWQfpO+x1zMeg%^E6X%BPkrOj^zbt}WH(sN>SvTPeH#HIP&?NK&PCiCOdNPE z8uFWV27t(epm1cWA4%SI`Lxw(l_qO{0iubFZV|)uA<^PnAO;R6cG3^cA)3q8jgEO= zB`I8}R39v*|8G?=sBK>|R z>jX#EmbE%7X`VpJ?u5X{UP_<4RI{FsfI|*iW+zpEM?>A^fmtXED zt|{#p@`~!;;^|Flm+f|O^7dy8Px2;+8dIPdRQK4K&+sDc18~ViG($iQ9v7c$VnhBt zUIbuM_9#p0kbE65WfU|0e5(SD*uE5EO{pvqwg{0V(~=%s8%MF{0xaI@MTPa{4$d9_ zG;n9ZNp~Nk2SlN_q|f$5Ju43qs)lf+8l|{QaF8b;UiX&Z z?jjH^3K?p^nZ@hI&Xxq-{iDS)hu-3B@Zk&XgG~%K5ag94@F1*aB>Ntwm<@2|QSz+; zDj|d^P2)P{vwl%1j%C?u;wIOLTQw?^pd<*NBa@*y2J7ylnMZ2LEqt=|}2P@ZF zJ=ozjxq9h7uTfG{vG^w|+5%G1Ks83g$m!N?FT`z64|k15JeyxAd~UY*9do5tFUKhI zjH}JprbofQ4)2;AR{XO&f(qyLUc7m9En^L$5@%9_1vzaPyKiIa`Oj^ErH{#{@h3Gw z9|M`u!j7j~ucu1nFPbwW!BGU7y5UUfyT|-?fPGketLAhf)Tx7BL{p;YT%|fot^tp3 z)Mxdi9g|L$-P>mrUtAgbadMV_gfQbV@}I9zRA!s0`5EV3#4E1r8FPQoMBfC;{Ozkr zM^TX7iyR+Jlm^s42qxvKr@Mg*NR7c>GGa4s7>vX!J%FM z@h}$4!rE+y;k(sKA`BQSK{&x&X#HCa1f5Cn4#vkS`1Fc40a3&(99bP=tPdPp@%MzZ zph}%21JLc?YgKAb1DKpj7W#|=vHKKa2P*aC0Qm6Ux5VeuSGo(pq@6j#eJ@*eQNP!6qmlYgK4#3dBzCPT`TA7Fme4t@T-yv|Am5AXpuwy^ zbeTAB=2WdDeOrGgz!FGnbsRSAFygQKDiOjpf%9C09W3zY)hPc7bCW*+bAHF)cwRPO@41S4RK@*h>F-Gz4Xv?=`6h1(Q8bMjq? z*K;_yx&bf~3jDnR;`{eihw9km_nOIowy52;E+?)k&}iqxQSAZAwqZbv3vmG z20N_(qUB&HKx&)FlS7zD(=U&vjdky%*AZig{e*iivw-bj-Ua$>LqFa=jIptJ86Z*D!h`n*#v}L(gOJA^RMj=>yG92>cB`IQjbSND zd2S^XcWWQ^E$ZVOhplGIL|5AKu0uoCY9iw2cTYUHjI(Sj@s>vW^MevS+N-7xmmP`1 zz*@t@uV*Tiw6scke-<;s)|y%f-v`vU5gtTzXDK<~K<8vC{@=~1%DbTF8Y~pjE19I! zI-BJytAyw@Bo74jzUU#Oh+1z3Sq|C7J;pO*Y4nMMoFc=QWXSL@iObm(=I#ZC(yz({iI`J~{&*X@jOBvWGVT+w!;QHP#8;r;5D z__J-YB)Xg~^o#LyOQD(}#>l}?v}q`)b8x059&&Ly$+l!foNglRsJAe1?ZnGl-#N@KIurq{{w%pCFTdOA!6OKEki*=B}j z2?Cv%fvBf9-6Y75)sVbi`w_P*cB9saYHr^ITkgN}kUMJ=2YUX5{mQ>!+0Q^de0Eb# zTH22L?SkDZ?B*B3#^V7Zofl7@=>E`ty zRJ4l$v*(doN?E1H%#U$M>4&QgRO)|{DDB=+wae3ubDCCAvXG3%r;N?3@Psss>7^EN zu&_RhbANiQx|Vp5vG{G?$v_03N)`h5%H}Edyl*}TEza(hdHrxRmm`lny}8Wz1R;Z@ zS3C3bZA2U!*X?ZIJEAxWfhK-hZ!p%zqr#U zi_sc%8ykzw6GRYX67I9{ryYd?bIz*f0T{8}s<`449@~DtS2b9t52IBClM*>^{Y*>6 zOZGwz-6w?0tPFMUX9 ziCI}*TnP^?mPnsX);5${&ziV#w_GYPJH7acZ@+>jA^~?JIc3?GFMK((O(EfR_8yru zU0Gd?CO1Cr{m{4s&(6ET(Zr9F;)^NHs2KX&8O;{YH?%TlV;#Eyr(BkW$`v9Ys7?FZ z5SQ!*a-j5M5jYgAr^MZ8u*iIVWA8f5!zNS-$WJhd_ijO#aza`4gF4*DCi zZlZl8lPs7-ZnZ>Q@`?R3li|-T1-QkIbiN4$U9)@0rz@W99o(fqgU0_;3;-E7U3(XR zJswM|L!0cOE@LTYWq2kVK-HMhx%sj;vs;8h0^aM>Vf*>1yD^Rf`L=1=imb>sD8&l? zis4_PqAbj1Y`;VV(dft0tC~8d)mUfWJYm8!j#^AY3>D)U2yM@{Y&Tw&zeDf8XzAoI z4Rp1{T)w5aPassZFYCk}82OZ|#CI7z8)7g??`=!qvU!BZhHo9pd4@r=+XR76z}vB- zV}QUdHozX_ov8FkY`vmt=Q@0R_2rr!KYBx1{l{5h5v8Jxk4xt?mgQM%pN zD+i*XY1T~2kaM08Q^LQy_RtgDh?n*mH080+W7iPdEfr7W8G)+aIFU`BiBr`=BIX=lp*?CEecgNC4JE*N6xnf~UXMF8c z6SI)(7d$fA6dp`U6ugg^Dl5YZV|?HY`xau~5Ow0wzDp%NKPz`V?5R=N6)vIK`QI>U}y-zsNisjyQVaXyxw)^>?=prL;;0$~%S z#CC{MbE{O5cSyEDD<9<|ZGmCGe-=LNzCP=*Rt>y+c3_gJe!}07>L9pt^qhFH61yCA3Tho3d>07fLAyJpG8M1e)h)oseFh7q0~{* zfEk)zPPL;LD`1Z1cpOrJhAOP@aEWar#fSx?bPHfTZnq#Ka9D86^8vXAvffVFqK=XV zU%3R=h=Q~TNQK+O@|Y__SMd{S5IM*)NyqS0Z;wx97j$5KIcLS%z@`;K2TO+8F?a(I zQp9K8fFQLgm!i)-%$#<=o;V15@kEF@!z)_pL`_X-Eo;{(4j{zn`WEt(XD(4Jd=M)< z-@k#`=qpO^C0|%VBHs3PL)b=d=UkPb33W(={r;2jw zkhZ&eJ8ctDViHx27ajle<6`~ab90`;i-KAm@>nt9ZDSL&xXtBsR8m5FTC#qsbiBYhHz!#hF=>fTm${PM|$+|6GtKmZgyin&Txw=!A zO-<4O-06iwR7J0Kj*%?gNH>ZWo)I~{TD0KPBV8P<3Jju)rL=@xRD3R=--8c}A$b6c zu;5lot%sZK^~~D3$6OQ8lYXMUPCX;U^CXgtj5I-Ahu3FD&ZzW4M$aZ++^tBV3_jyc z{cnym6C>mQp+*14k!EJ+{LjVozb^kT zjx;L+8|VKoN4ncoAxC?SEvCDBd2n|(m>cAuL9FmG+@Nj{^3LuqsRTsbf5)U3;&VLA z=QAr`|ElZKtS?*F^}Jto-PM)-6x2+=Eufgdq=Ig8Ff}tZ-T|JVs0g%)xz*Uj6cb`8 z17agXLxBU6lx7euPHZ5XklCHUo&Y;?X#u3j=I-O-0NBv}3+JQ)P!`}W-s}D+Ng5?! zPRt7LO+XDGc2Qpd^WHGXqe-41l!&O+Z&wNEDTzw5X)6 zMr!&6Ph)j-5AWpNUuQ&Rb)^&o;0Pael>|Tl7Azp8t7owZ<>+us>1plGZ6S3b{u znEwt6$qIk9C6!bo^REVQ0LlTBi%auY@{50`VlNZ$x9siC>I|akhZ-cFFtc~ zb~bi$XLK?X^2B1!#QqtbpA}k~JOF!Q5$oai4x$C{R}}-Dd-cM;(zzMBv zfScR^zg$v*Kc(%y^vB%`o%?@I=zl^Sf035Y^v(gu2=DrqG%-29PpRr^Y2X`Jnp>Q} z{{uomy}#{@>`dMbosFDboB)P@$=;tpQ|f;a1Omid93B7UQ24jX@jq$)O=+e5@71wma&~wA%lc2ekK*!t03H@D04pyCfaSeOCG1SaK(@B;tep`4luz99 zy-v;`M-S%z>1t~`kei*?|4Pj)?M%)7RNd6Yo>|?_(!m8NBmRH3?;?c1W#&L<01E)< z006q1STO%d^;f6-5wrdgzn8$r%N}G8Ff+1o0{U2*0pH&cyqt_&fdFSm7od;VKNbIt z5LmeZrj{no?@j)GP!Rr#E@Ni~0`UGz{LbXxs{h-5n!gSy?fVHe1=-nn08D{q2+WEg z=l5oy`TysP{ZB4Q7aJQzBU>QNe;4$Bt&D6fZ9M+l?te@)fq$sd{6Cy6og^*Yfu_oq z&L$TBs_MVwGR{WtP5;r(+y?l*BY%n1|C}S6_lA8RI+lNC7J!kJoAZBk@4aYZZ3lF6 z0&sEtr2@X!;D7kO7yb`h0JEl&n3{$v{r|MfUq%vkCLmKwJ97XVCl|oT(b32Qf#tmi z*f==>Uaaq}YzlP$t4RRNOm-mWcNc)Yi?a{F4CILLXCt{d0n8u&i2jAR0L-F)BW?h* z*nbc=4}e+mZ^R2=R{9U(VFNI${EgTF%&LE*_YCU)LA>uVjs8aOQB3}WINvjvfNb9D z^=}C)3k!hR^dFEFzzqBcd{1Ql56A&vw)hX^c+YF$VQ&HaXSjEndT@Egw_wiwDV*@mE{)dL=Z_U3B!rw982|NA+zO!@s z2Yla*e;U7MaQO#(59|6jWP2y;{tx(0*5e=Wovi0y@V`4u`Oj$mYjUvs-E#l;o%0t~ zb#??<12rv8-@kAFVWME<>}ct(%kqBgS>NUFkAHvC|E~m;e^0D`c#DdH+`Sk%xVZp~ zY&^UGR!(m2_w}%I`}~)$$-l<`UnA%JZ2TMlnY{oY&>d)ku=opP!XIRn(iBqaE0I50 z22IJ!bW(vMq$L{%wU{>9L5L@wvqc6JF7R*hOQ$LT$xHL;`zHC@6=_ihVcWc}wI)_h z+?ak6**5Z3@Fhf%_&AcS!K5CLuCU-&))Or}@Xwvr3&RyfPrjOA#9;l6-I&`G#3*$M9a_8TiU%IN#YMYLK;~Pn8*^3Ju zS}tGsAD0_Lbh72G6}hVrxWDeF%L??nVIk|j0qM1Oz{mswXE!1!U;>oF_*4d$T+bIZ z=ay=Y^=i=x1B*^I7B*-k>eR@z(vNeDBq7p`YJ_W~n`7;#+bpfxK{(~fSN1ckw!z=O8Wn?kj(E@c%t&@x z7(vl0nc_q0O`>Z%mF931B_J3YpY722FCiE4smT;?n%e=Q*;rrsd3@7a-MyCzj`?A( zrT9$hiEwQoAAYhHHaGTruW!NlIi9N^bE>l#BGW)=E5ziy*z zWyk;Mne@eglDl3!{&?sEqwvFjV0Rw$S*@$8mV7|)qZvOEWg@2%@625OChwQ&9b^L| zRc*Tbo70kS9VH-I&ZnN^=m2x9Wr_R*hB%(DF`}t3`E<_xrZ~xzmry+Sa9&p-J>f?J z7G{+P8%h5X@}9X z5-H)RsA`eIrqEhRdwkUPBt^98G+_jb-tW?Mh+}VyMtnW}AVKIajW@oG ztG^pX!)LeC7Zq)ytJ^aMoYfUCaj^qy8ljsEBnvW1Ib8x#J$ z7;U#>!7wx-QI^4+$c87Rwc=SxE;L4|@~^GT1!Q80LiPK;Hhs~j?Tjs5Zte-z_K%xP zwOeugt~PmXeUIvTZA(`pcg=Y?&k0h{Y79Zxq8+5mRLk4p<5LM+5%}X7ZKlu1QvHiR z!f`)tI}B z>K~H1UlTHE3NM%?FNT4M3$smF>JA#t+e?~S>I7Oqd>G{Q<$9fSr=A?|*Bg)H*aST2 zp?UA_*@MfC3BrH$!ZY(q(S2~b=_-YIztA!m5lfrEbky-U|5~Y311o$fI#Dh9^{VT6 zU1L0LraZ*5DDFe84f|MP=HSjCR^3$2DaM!66|vCOx=obpN8aGmA!`bDi-+L^E9-y#JUCt*;e%>*gofrl&@A}Ut?Xcm1#?Qq5`g zQ$AT`GjnHXV*M=vNseoA_-(qNw*HJ1Y*wzitR&oPqDa~QqZzVsj=Ob!XcEG@_Z&{E zYAwOkA+K}ZAdzE1epcxP7Q_MO*y%7$jp}@fO$p)3d{ZMlGsmtE|#Qhisr^eEe? zZfO&iq-S0?jBYSQB^s5>0j4Q+4w0`R8=zkuykPd4wL%@c3kSz&Q6KK}nWK?6{fjWD zV(#@W%dEt#1+Nr+1}j^6Te4^=CQF&ezE>T%b^aVhi@-C^MmBA8^wj-MDB5^K5*s2r z@I0B~!+Jg@IWc{Q{4`tJya7dW7tvUqo(THQW$4R^adMB)&tE(1LaO`6D#s} zX5p~!at)&4mQ5utoQ?E&{+p$~K|t1?lrR$Ti>rE(a|5f&+ECpg*Us018S`HOSoP9v zpVmVO0>{T!bFY_7t$CJEBV9J_n>{XCW$bfs1UNN9_6q zp4b-bBOXqw-hGoP*)ctpiim;yQp@H}qQGB-m}ID5sAoQKcka0bm6bnzw)chkS>~ms zp_@QgJv_~!`{}1GU;;pS`9W%6Xm0x(cMZG7tm*=OIm*ez{*vu6c|_<-KY^)_NB*vj zoK91n#6GMFy9}9ubxgC?$>{+ue`rG)mcziK@;Tt7ej5&TaG^d)jnP0-u@Yak+pvTL z>x_-X?`HYl6W>F8bLGM*1g!gUd&j)1w0(~<0PcfIszu)xV1R``Ix@n%t+ZYdZrw^TXSf@^z>jX&hR2hHA5 z|0-#z%Gr+VNC;tXQ5>$o;IlPGJfWYZjUNB35R;GR&jAJGm8t3-22lxp+!5EOw`Nl- z&o1k#K^e~NK7@TU?!NGmJPa;ac>iCr{dfb!ueoWW)q8oYQ7d5sf^^Q!zxor}RbOs-xX^`a;Pwe?=>Hde_z5ZnQXGk=_|T23)v<#Tj-rXSSs?9tPYFVxl`%PUFn3;e)Vks?k-)+tKl`NLfZLb8k;I`_SJi^LXO=T z&1@loFt=90exh_TDCtGzfjRy4=NpG{LC?LHv`@v+1P*2AZ;0i782W?cNd!(Eto=@< z?S(qg(>p0jT>n#>Et$2If{JV_)g=IGqiug+d;VD=Cs;aP#1EaJ2Qm8*8jOSW47-a9 z>>)hyglC)eCc>OSm&aTKvsZ3^@$8+D(3!{82eup5AsH#ZdFbyzhzTAplQSSG->%cL*{V3gv9A~bkuyXcM2 zEr&$V$AW`9wvI5kzCKI17MlcpwYvMXcz^2hH4cX9lrAI&O!1?Szsb(Fmq(gr3s_=8 zeCpTZ`iou*sdSUm4A$o!(y=+hU(Tp>$O!lgyrj%*sM8a;rv2y~OT=ZVYbbPFywmH41zj8!L-M`Jm;} zavy1vr3SAtU;ZxJVnzy|(jck};hCD2Expz%drN55kZmgDK&_YmB16ILRc~EcrZ;p& zMaE!O+zD#379!Q5AdO}-Dq|Lvp{z3|CZ)k|CFH;+UF{}sj-|s7{y-~P1W*% zD=E$|vF%spe@n=TcaX36*=l*s0Is8cab~Q9_QKWUQ$>Hc80k&zIy}pb=tAX*#GUAB zM3}8qGQRl6Lj5t{z8ujOsrSd9#gCt>=seWEgK((}DKG6sv&M2mV6jZkV9$~UFo9v~ zpQ%&_;m-ONBx;#?T?USxl7V0LVJm8UNvRJzkZO=fSb-fo=Ch&WY0uiiV5SP@>Djn% zr{Xss<5l$WO9*|;y4_Ub*P(45D@uAo8+tmvmu+702VUfei{RwOVG84`(I%N#lYV1? z=>OfWIcTg)P%mm77E5lcxy-DiT>oWZ`#N_BVJ~Rcoa&ZYRwjqcFD#*LJ>)dZ3u|C? z@QV$Adq=$+EQv})Y;SG|<3WHZfa%e!*Z{mJRO=;WAU?9LcHI<;6eZT}9uZ55XpBTe z)km2_JVcdOi^fL`c75Md4reMTT3W+nH@>$VsS*ADe4C_1@!)20(V?2U8qahR@4V^5 zqjK&UWZtmRn&^DQIk>q@C?~DnTd|)i&nTPq9e*x6VFq!=YkH_bY$Br>nvb<|bm*_j z2P7BWmZWp4iv;B6pX9j6--I1K$3`J9z*UB(kf2ns`#8$Vn(J{`#FaTaT=50?gdB>( zi+3#~$Mug_L~AzLDkKZb6lPfI$76h&D!-l>db?|Hx8Ep-Ik8EJbtRi$oth2^!4sya zh0hsl&1y?|6I095o-y|jQO4CMb57>lqB>2%ofIs`TF%?>w|_iiZXR4XG0bn2M*1r8 z`&&fL8?l|APr5{PS?IKtW&(fH33N#6D;q7*9eS~B%t^g<=8&@@CBirJ{bt0?G#wY6 zn1@*dQE$eF?B`f^@2JU7s4zvGW;q4s}~o)LlopIC}V}r$XLPR!6w%j5QRT< zyF|`P9FgIswnJOVc;U_vB#7+pzc_~6KIzt+lz8+?TGNp#9zIWmS;EQfv{oAL z#6v;1Bl^`C7TIvRaUAz`T^M%j7gbh7kyu$$B>HJ+oVyVT3Uk@ZMXnI<4;K^Eo_J1F zyyfaxIwGzZX&WR(V*k@OWLIqSvzb*FVE@N-ALx!YWQG;5`E!w|bM{e$yOLgbJ#Z1> zrOcFy3jyIBo81M*8{6EUPM8w)K=CT*(sIr~9p%~a9f{)~50oThUQNyxlg$?r8141d z6Ti&exs>kd(QT1bb`OIx&1DG!{AuXsrokYZlw>R}T~0sfJyW=#HpdH{yD+A9t#wj! zANm|%w{a9th-%Qn?}|u-fYp|GZj%6;&-ZIbAT%&{Ih>tqj5nqhUNv~B(O9`5KjTjs zS{DbW{&2daQ+dM6eVV0L6G3Maf=(YpQgn2ojkK`Sh{O&qsc2zfBBmQE6NPJo-BLea z>Lz0BldeG6bNgbR#cZ6De-=lzLXc_z5;$nG)D-}b%6b3thD%9K&cF3}4>PbZU#rfET{D0gKu*Fmoy9xY}_f z@d8u9nnB9?)zFI0VuU<$I^l|)-NQ<(ve%%f4hfq*L*}oon(RSlNJy}Adx&y<3|OQY z4~x(l+3ANJ)~6i%_A{UF^zsX)ghhC1JKzz=<t@A@(Yh+tGwz6G@w)7-MINt=L3kpl=o0%dq%Aog6sg@h$TlqwU#&JN{4jEumiR zcUWGja`^ll4OJiXwO13s=RN9}S9isN4+C+ABF}zrXL21#ZUy&a`kLe*2&bTloloMw z(C^RSHym%x3l~*9G`wcb)yEZ1ufux2@JoARB1dUI>~$arT74)pR1@Z~Z=oe^n)i4E z%a_~ZF)&5k9=oql<_TF(CWoFH!@Ga|T!b1cy=AE6=F-tyH#Qut0>zTBJPP@Y0KurN zc#~ArIG01%bhxxWwXdzS;AWu^KT;ep31=(XK0ff+nfi6!E7;C2~<k6lkX8Q?5;C=5kQ0elZyWKWz2_Zv+1ZL9cE>6-vncLrfTaqy&Sv3Ub=jRm-40dJ9 zck)&ROgaJZC28iKj%;piQB@_IntGRr7nXJDIorH=EwIN&bNwZMn!`^#7$2z9iW+z{ zk}2>v&*1patE!3m;&87m;6+!3ZQke;k z8y@%jc{^QL@#0#+Oe#7V5X^$plg+*Vj|#{3|9qg15kO ziDxcBqz)LF6ywj9176HXot!^)h_IZwd#lKgGrGQUvkdAskM~^ohkCKPnXTo)L*m3) zYY6m$}6C_d@Lym7H|rg zGn9^UCs)4&yv1LYp^(xS?b~_DTfi37zBfMseVj#EKOQ!6T`Y)$@N)>6??NE>n)_$_dilgHNRPh@Y{}}QsvXrMiZdCqQ zOr=?ZO7uh%3rnRRakHcM6-tq-O#51*UG>9pTcQ}QV9b3iHLD~=OHmCzAI9uURgH7Z z)=gT^hnwrjv*@dbN*;mLtHCt;^~ zG@5j)t|s>AsC$+$OsXxfJ`S#Y?W9H|>G)FiqCEZb~F14-5@um2Z%4zJy2J-1iM)7G`)z~GfLVWumY4J_H zRcwspwSHB)f=WmDf2(`7rA#2@RNz5fQKF@O;k)Cra@Nz_;xGo5E;g+8fhSl2(NG(` zNT$Jqm6X?Z@9a8t?;H1hlw(&W+v*v?@ip`cv9g+}#2Ts+*AXepZL+_MQ>M+te79nx znuL6dsWI#GIm~-(YScP&k8Eu-p|@qV4Pw2b-`wly(9KV4NkzVC6lilk_>cOW#|$q%(4ZR1b94{4?4 zq)`x)E1b@DDc|eTJ*70Y#@l{zI2_+PC5N{HV_h1YL9k)XHKi&KDH)!6g>%fg4N+e& zMOrzIwX``E2RPJQ{m#lW${(SQQ|Ko+?bZWK1swz&4&jKcz#DzwCv*R-&VS zXWG@-uDEDQ-Vs;WXy=G~4Ju;Q%~RB6gSl62LRp9xba{?dmLQIcJM0-rc*?T8LJ^sVimM!GUGaoC;&O^}klB5ChLfkXIM*a#vi) zIpCY8gv*F=Zwk_ABDU<5P_elDnqhx<7S;9Kue7oATuQqUD6UcFC+tBRzKveP%2H7p z#-bIFAF;8Ck6m&p^zDX?CQKK3Pd|S`+qfWj6(0ot_zVr0!TZDViPXm=pD?n&43tr4(V{ zo)9;+Li(|d)HB4ckSW@+C!K=bMTI1{HakD{I)Vhd7`7e*nhPbi2&L9d^nB1n^Z zBb1o>M2u_#;WB5@{q%gul*y$~%#P(24!>iTei}!&hhF>4`m<$K#lD!Da#`}o+MGcE zs>QoLkaz7sZx0_WbD(CVXUFFwju86$w?t3^jmk^{n%I?Lb60s&(?xjUFM988cbFfGs8JMY zLoQdQ?^l?8TguV?dPEDm({N?Tm;!NMDP2|JU@W#^N=^ES8n3wk6ZJ(baeBR?%2xRKrEQL zq#vL`oNn)A_oUP_C&UggARL*xP~k#6=A3fB{!*tpcM5;$4Hd&TA(1y@$gl_r``W-5 z|41r51ObZf*_AZ%S`tZlZh3Cv>_5Q2Lh|1d89Rf_Z)_%9a!Ao>B_*4A7y>!)= z7mwa^^KmhqMJ~5y=nj7@B_@er>6;SwH?RT}a%-<)zCe7&lvGBY)YssmLChKkZQ>sa3ZGRN`pn(%kGGr zrYRL4>A%FEt}%n?qq~B4-a0YhcCWH_Ishs5I#g<-jL~jqiHG)THgy*3V6$jPcJBk zHZY!I@gwEfwCNdgt(kN@YHL17Z0=rL$->VV!9{Fd(cu@^%XYR#k%T;f>^Dn9nc}+8 zo8Hbj_D9O-Gm<)oYs9J-;0^jhGt3U-!-?ig*@xq{5!ThE7#dbwMm>|-mDN|dOQHXZ z+Bo*=_pSONy_@mx9fQ^6QAfHB<|NY%5Y$Z+Yns!8`~g|J(cmCWr)kt!fkG2%$F+X^ z{9;NF!}YN3;|}+vGYs_TPdo<~H*$5i~u zZX$;r@+9M#1}eNI9^~;A7Yu&Eyi=oixnH#fFkm@!Q&_Bdk+{G0SmDu z$W?ecHo5jf-EUu|g4-UR&52AbrOrezb6NzNB)a?OUO9s6_lF<)S2?XXfz zB<`I)d3m5dsiF3kTei6dqTO;&S)3Wl4}ejN6!ys#VlHQ+eTk%y%{n2L~)&`K5h?WnRGDhh_8|P!IqgqnL+By!m>V zV$UT>Xm&Kvo?>H>rl$qsbL=2-Ec_j`Pp$!b!n>0-dd_Pe_d7Kd>oTcx?6};)m=OW?AmD1=(KE~m+3Mg|G_kHKQkb+wb4aND7BUcGw%g-->3elmq)Zo4~NkM)=mo%h-PVp>_o0iVMi0!JgyT$`{a zS;&UUMi7bQeemJI6Mntm(%w~suu6>ZxaRR5(u2JDq))(7vS{h~H8<}|pQ>bZ=qTq( zFsC%0>)24O7WO7;rp?Sedij8xze4FrR~1wb%J+(%rdT&ENu4u~Ya?`Zzs~z~1?|8C zQ8LzFOcs;34D?V?Um@D}yh5@zv~l6h=m=E^Ra4#l16+3s3jv#=nCCSeTSKWbGB2Fv7!n4VdbQ}ha13#W*cD{nE*CA6FELj`QQ5zH1+)2zX< zFGzF?%UdJ##kBW?ytD)IKm;AA?qYT*r%V&y8ABSFf0K^;!ieeCB|@v))Xh;S8W{pDqB5 zGG4KLzyHzv^8dK6F@zz6lvBnaKDkM4z|ti=Y3xUMdQ4aa5o`6aS#~&tpsaHf&ttoC zLVgeH%Pm4ap1VK^y4#k`1I_E?lr>JOk8#uCSukl(4c!TB)cCQX-0gL$2$hx4@J&1M z9cz<`dOtijD}VQT6*DEG*jkkX2LybAL{MC=H1sPQ&VDQ*SO(@ndHMYXF_=}isV!xH zX)E{R+$f6<9M`~Lc1Jm4O=$^w(e$s7d^B3Ntl=HUb^cdF>PrDlC`pHz9o2RJAqg~VTbU(Ro@y@=H{&^+>I^&OUt z*x_iC%TkGO4T5_~Rwre9zi?@xPGHrUnH}8gS`YGMc?Q26_e|wG+x#w?45e8TOsc#b z7{aI#qO{z2qWE2T#H~|^tJAjUq8K-#hAgGRy5i>Evv|ToZFd3Qrr8E4hM}rKR$PUU ztYJMFAXkeaJf-oCmD+!$uIOc46iUg8u3E&>S-marM(oV^$n=qxH6MN}W#KfUXBLd{F9J|GEaHu6ZO@jghoFyjG#SUO6hgaMF>F?lt0_~}p z9P}fooDo0yKrl|k&L zVx2ox>e~J=MI)LW>3gPALPHxR-ch(=ci6f zP+H;2J+IEcr@ALBS6JOX(av>#!mP!#o2F(@a}ObwvB#6?71`!;)?wG##&09?CEHeHcj<^X~`xQ#Cng^@Dd+%=&d?DtcojAsTn`;#I59 zGCq>iA>-%m4!d9M4^L-3);`SGT3PQbYm@pjP%{AbTzw_bzo>wbW^evP@X)cTiMWV~ z36aI&lf$yXg^IUlOfQ*{VixOM4XC2(EwsLgBJWy+GbJ~JjWD$k;2v8oP{mffAw6=~ zIY3XPNgZ8?fwcG{SaCF$*6IH8ajP01Hscqv;WqkbqJ2MBl)zF{+Pyj3Y-9{@SLBNm zaNBkC`*@BLztXhkO?R;%dV2S$st2|f4!?yCM{PD52yv-zzUMkpapTSO)UBl4 zqCm=#da49<<3A@cKg{|#3J&jYuy)v#nHOd0d;vPjcv@(;M@lv}SI&IlY-HhxqroGq?%Ja4(yxW&Nj z!-)(xR@n6UB-ADzIh~&10OuApTG%(egyJPR{l!x|jsP0I5oIez3yfKF5KnN}5^}W( z6)c5B%dxdMvzKfQU(&crWVNPOwPgadg?TV^`9Y0E8Nqc9*-Wg$qID_MIop?(rdkN+ zuii0fG>LjH#SI>Hfc}`w*(5#+MV)>A)CT}R-4?0H_TkMAWFAw#UC0+zr?fn(T|C)A zk7<{MS#T4YXv?=Y)M$5Cq~JefqkFX(n&KX;x~pybb3?U)g^AT_E;0{e-q zYF+vOi2Ikkpp0tCA(TY|;_UIK7h_!~X%YG}ZLR>oG1I^o)t&i@2=ZOQEK|gy>F;wt zEC{*=Wy&wYRXw~^_Y_3b4Sw3WA_Yj~R%R%LIcA(sN=`sC9~d%^3>v z=QR&6gG}{1`TRI;u<;VE%gR(aZweXEcvfsP;M3a&HQXoqA@VASy#nX^e&_7GX=pf1 zR8{oj3W1yCG|eYX=`Sa7wj6ac+VgJ*beBDRa@^CvPkp^~Ct89e{58mvLVE8?7Gh|e z9LM@vt1si@Lvv+?>>4*CT*He|o5Of8tqr~>US2Fm7OOS|p`8-Q?OwP>H$WLF9+%uE z@$Dhba_dee_Np4qNV88`=If7*1Ilk@U|}&*K?F zPAi-K+gHu;JsmXZv3Uw)B@ADSIL)M(4at1`#s!sCa5kvca_D`qW(uR|Fov>xGJV#K5`Y~)ju z+A6C+EB820dC}WWWNk@J^gN5c^o1;WT`?pWE%sFO&dK^0drEe%TQ_IUj}gxz&ZuCB zj?POIXX;aLOxe23@$o_-)z)tnMCy&OMYSwvD)s8Q>ZlQ;a(jmHX6bz^4CN^|uA|;7 z+spdv2}GTi_?xkT-qtbJbSFaUB9!dYE&JiVt0jr}&pj~s!%1VYDM!J$K>Ln{kB7}N zQcJ$IOO$5rXRt!s@XV)VN0^O-x>^M#}wV5MQ+@cQPV!^>UH0tg^Bi1mj)bmEn}>o^l^Mz^@V1* zKwUuyR)n5kzet9?a?|{(yMp=5eQS2psjJ))`x~VRkqA3AOsb=?h7jtuFNctSS}f^D zYtUvebU0dwk)L?O8LGS>U-M|}PeBW<;EP7#8qdZAz1o&ftv2)BVZU=kUNbV71B2V* zy2_@CLYi#*P}?;NBE$DHLbF_(#S?zao%3uE^{|e}ZwinngeVBkQD9QaUxCjO{7j&$ zT&jk1OQ4X{fzwww&D~Rsg%`tN@7vAoZJT%=o-B;~luSwNe1;2Y#E2qfsX`nRQ3y6b zgby(h^+S{jbl#I4UVpFt)!=7usokJ{fd`RZrM`^=O){N-9FpBo5~3S16pl|QxIO($ zkNU$H$=x36JyK^B;i^7}d%E**GfGzG%&Q|dVBxsPIzM0J@Bta!ul6~&So!Np+re^j zuP&ZkZO(weXw4aNmVjn}aI`fvjKk)s`m5pMc;Mlj$iR;av)6~rY_w!8P)Ved7!pNS zDx=lzesblwJ1J?1O`k@0{gW&OnQ7Kulim~dRL~mRJPNw?z_IC@PCqvN{nrpr2-!>X zH_D~4O5EM1Q&XMSmOeGt?6p=CL3Y2wa|?&jFE=F%pDVrbw<@eoYk@j~m;mm<@3d3L37D1)ZYB;ZbbpAl*)D@QBy z1ycwn)9c5C5xq-7q`om3-)Q_B}q#v8k%jbF>ms zYE`_XLfH6vZPrzS?5pBNpwn+Hg>kj;Wnk}{8;%y=Bd_3j+%Pm>EJfsO zc;P+@dHkLrG6PyVVF{&SbRYb5H`eT-`J-L32Z0=LqdK5yJb;6Bhd)&mx$qYa{M;-` zfywx7npd!j+{XvU0YyXY41)Fk9CetV(?}K&cB(8w2m2D3`ft`ZgN_llzJu z7&Y<@^09pJB-uOCs$vq)lej46v#6b5wv$<>J@v=Zoav;v2)TOtN# z+(hST{2c7!^+`|(+Sl#PSa`MKN`COj$H>Cnih|`49-YM$>t_SAm2>j2MzF`A9PbLNXm|$XC{sqX#TseB%lT zP_uhG`eCqfaXwGr#5vq_otT5~DH|=BnVu&N&ctf#Yt2C&hZ)l>oW4{I`_u9}_S3ZS z7EMJndbx#JK(<4~M|3Cz&Cs7lujFcZTTi6=W*GMv5KNR0`r?45sbKG?7Pu*5ODthu zHLL5>9ez%a%1=2{o6e%6k9m14#&Gt2YsQ7}tfh12kZGi9k(-8>%<Mu55Qpqd7{Foc(86Mn-yF!CKTdvhJKD4W}E=B2()E@Lz_;IjZT!a^WN-Cek zk)j6b{@v8N6hej@e(|xPfrRmE5jH-=H=|u7`KUoBY`F|oG{|dlc_Q{m_(2#`A(UA? zdil{z1|4#RPd*!XRENkUoy7PHTAVc1n46Fn%s#a>LB1bwgfN)><_1kB3|VGGceUm0 z`zy4`v+~UoP)~Puk~0^1$gSDPH!A0g2^T2w`rv*!h*WyN%)@tW*U+ddSo5krrsn9o z7|U8Yj!E2!hm&ZW&3uWxsilY>j7G%p)(kyHge}<4vDFYpqwOyw1ohmzBPUig$9*EN z6O0$xPry&!TYFrb<7|w;N-N0T z1DxC0oK1@wS%vQ8-P)aJw=~*)p;?*V-#i-V>M{Mhh zIYW0}6LiaIM(c=tIMkLW|D;c~vXdW_CqwM%qbIzgmnPHi%CKZ?IORJ~!vq+9!);RE zTxIGP0|AD7e;}K;Yo94sriittZSp5%H>SF-d8)ZfdhyU|QTD56Oa{cWQ2tsH0iI|c z5j-ZOc0ZiYa&B^KO^29up%_`zg@dNV{`JMq5?z zMF=$OFS5D;JjD{oh8}fyZ~m|qON|>?J*OHgdf`P zZJug196M@vWVK3k5bH|b;}v}U*eo>IDhpQb{7;xAzdLMFJFiM8ZgVJ!Y!a!E5vuH3 zt>o}VB+63jDTGRjS~G0ML-Vt<7YG-Lp2*n1UI7yUUaz-jrMX@Khos~u@H(=MLuwdu z&cknkPBs1v#c}3B&avKcCAmnJ-!%im2<3XdJ1@U&WSgSq6{T3=>PkSvApsSVBNMv` zHP0opd)eTQ$|g%hY>=#SkDBEKOp$zcd}AD?BI_-fKgh|?q)?C3G(DTds7uK!|CnYw z9Ij3pj%}I$^{`iaDTPem$D zC_~nnNs`NxV=UblmDb{|iSlCZp@+@O>r9h{*9uWF z{{4Gbl}ziLC(iTJrr;)fy$WgT;Lhu(a}gmPFUXia5yn=8HGIXIf}n zKFo_@#E+U7IN8fdOXA0eic^Z5g%MZmgUAKV4%w&N$Z05q2pU=3HJQpga67~9wHnvS zdIxfaO&xUKZpX%>!01))@Aaha#R_Q2()S5^FiDcT2sl1>>L>QKJS{1j*v`RZGe|98 z(MeZ@cAL+%d#;pO-Lz;~OdE>b(FPd_ zOQeb~Fpwk|I6^6w8|Q++<|9Eh7eWjkbdzM;y4ZD2*$pA;o^W?)A1lS3lujR44l7NG)f2%3pS2t7DCFM6yPZLQ_D$KrP)`Ra zpbpYCT|-BBEMdn!{p8uB{@D@*GcO~yTs}7=mQL@vDB<`6x^J!-g=k0aUWsU*`LJv( z*>rL^-!$bUq~xqAGjy-Refi;Vqv}@8inFv-92viJa_i$PeF(K6%eY@Z#RyGQyMy|H zavvFM$>p(hgqfjCCqrQpg~L-oF-%gc8@HLtTz(mo67Y;)3g^doq3~zY*p~9CWFx zomU>ozF3Y{b86jD1RFU?Zwqinl3VfKp}HZ@l=$?V=I8y{70&74;&GvSN|%a?@97iZ zzR-M@$}$ZX;c$4_yVD_TB?!xtbNwZaKgBNU8Ly1$V+74P+j3lWLFe2AZMExzY*PFl z>K6&ZTES3_&{_flTQQ*y0|V-yVpr9E@0MP^DRvK*j|W0m%8P`A98YWbwvMIVFdI*5 zerPGZYw9fq5+>=v8f0d-qF0{OkC*a5dyx&`WY{5|DiEKWdPxw~P7g!l@xQ zsU#s;M1&{5j}$o>-lcV*=tALbv>dP1_TQF9*0M|o=HHsLr@CFFnk(PS#5r7t%CD_o zr~RywnSvxGN7j@+m$<#Ix_YR;C=3eipru-bGvMNf%|{{i-+kUZpzza za`&@zgVjWQ?74Cm7cJGr)u?RSyfKj;YCKM8#cbU!RN5p$5ierVH=}kcE~1yS5{O=k zTb@*8Tu!@z0^#_zc#9q6HS*Kme0YPcTDow~h&Q=kGSI@)2|>azRm|l4G>YwpO^gwk zQRNAQ{zpKtzF(M&YLsi4dExDbn0oeb-NS6|kgVStPQ+$TSha9hfeEsPnzfvSw@#VO z@yUyK6bUsWUyh#BtG_lw=reJid}l0x{jgHogrKahCfxXKqIU%G!~(rpWY$SzYGHiCDAd=sQNN~8;_X%z#9BYU+2sI*MC-=Ks znoncsCr8VZYlMy_EssjVtnA^wxru+;{hVmVlflZ^VL=7?YBAfP>v01MJ z9VsU544RTLP=nu2VH6SF_24N!aJm5QI{WeL5?{#VR26z!>6)1fa(rxXAn`$UB3}NZ zk^;7HJFbKo_?LpFv{Z>Y!F~+H<(Lf(jJ;EgE=;s`+vaN9-K%ZeUTxd9ZQHhO+qP}n zcEA6Zlk9_JU!2QIDwVpZq@J2JGe)ZwB2EJ$I&`J$X0IV_KX{e@0UQyP+*oVQ0WQhK zK!DFO4&0NF$`S2BGm=L{_8{9l;>*nkf0Lp^+07d~(gX03H!;Q1U^-HK@Ar!!9M-t7 zf;oMlT+&eU`0$pFL4{H#V^_$Q^YYg!-q}w3@Ebhv)e3kp?IGCf_mAc?sEkzwA3cZz zWigL#*#K0_qjIyiIMYvAIt;E*SOP!Z4>U;Qb zt1RgBAv;m;7|h5*^8=I9WLPePfA^yV_xUB(zI@mIzq;hh=*IZjS}16y%*UfI*_wg^ zfAOr|GPEX;P2->5O~V@@TS?Q2@aVBkyUK}+v^hCQfgjm@hgmd$8md1zbh4|}R1J*2 zX9v+dzvFfd1G&Xo9(gSQtwsSIzELr?j2t{7VyP&zdU_1OK zwf1l8!&?z}OFN1x(=JJ8b*|!`JloubA!Sy1C#SQ0ACo@>*%o$J9Nn-86zbgGhoZ~H zVhzk3PA8yweOWx}*OhkMp)F{i_xg%%@&hW_6B)O3c$n#IyS3Dgme&73+~rSsnL^wc zOU-l?yMsPt7(i*#hCDq0SK~(t%W)p-lkq3I=MEwELxl_J7qnTgqX=7c85(FLC+Frk$o*vZj*2md?7d3oy zQ19!l;c88d*s>{kL#@CgHFh9xkj(MyBCcea6se)u( z@wC$_MyI9x_;jCXq5{(YI&~JqFjr5#)8#Q|6tKtc+92x^Y-nLTPJqx+Gl`B}siQnq zCXnC()y$eHlHEse%O_>ZC{8q%y;)`>_s_nrf&Z$v_u-wc?&$8rrJzv%N z$0;!3BJv)7d0ua115%d}Q8!8#ArPdda5H~HlZZ*&rkk)7^m{mUp{hS8yCea+4K3&T z0TXXHmQtsjVIcDI5Q=)Zpg2YfQ+K-Puy~s?F7o z05r{lAe`{eZDccw(Q>w5X_UAK=I$LOJ?_|d{8MC144OhZI8# z6^du4sp>aDGV^vJYmp+dDi4S!DKI~zFj4Ze;w1KNaAzchSmh8;f9HWjd6pNVj22$f zGq4Ul8ng@R3*~MK$@r)m(55l54%Y^-ty|tdO{_EOTu7(f`S-oCVVOu{?;KN73$}}% z-#T$feT@e*`T0YfIF8Z&<{2TSD%oD~@ue9LNB7j18MI;V2xg?|*Rdc06PJ+A=>Riz zPDx=So}Zs_E@dCoi5M>ZE0$eM{PXG*LY1h+U6}hCfKwvVjTVa82R7t#$?SdLHr_TU z1=35y0SvBR^eNE9wky;n>$E#EU&yRp^3d6w@D*Slm7k}8F%$XBYIn?9*bwYOTYXUU zAL5yCNR;dtGGyJzBic?D7*Wq`9iHZj%i)eDQ7`>;RLPN}G3t^^hDfJVO^j ziL0f>+Hj!%`@o?&SMp>YM7PRgfp{KL<;tTzsd&m#?-HqTe;-JQ!%BVcW6l!YWQp%KOxZ>7h0}H1X5bDo zi=#0NL?f$h|LlrsnBs-;|A7m7-e6vSiUg{Q!E-8S4Y=b(^t5oLDV9!d@;!qaKtFQe zlMDsi?LtNkI>L(QFL61Tet%n1J9%iu=MXv0(=<65=HQtxuDgTtNnHfJ3&FCGi;gyTHS{u1@bC{9Sw4a$gp&}2wNh)uTcuD|So3qbH?1jeojz42wOP`{-ik4*mn z`7{i)Y9H^s$Q3Gj=Yg+Fut-EVOj62}Uak9vJ-5CNrWYJ9670npHk%D+_zo~$XEMlf zm{@G=p1a6)s^A!@CSA+jR?JjPQ>Y2Wz~__zD@rd$12J%!d%U5^6uR+kfVBa-B9$^V z-R&n7u`SLLp?PuP&_yTh@VKNl?%Vboopq!VbueTp)i5TS3$iI4N9h^1%JfJ}9xQZW z&`t0xU5KKwNR)S9JN#cBKX)N$yCo-eLJM^@@WC02g20+Hew~7@NK2M_KQ>Wm{ATX9vN1ml;wuZ* z(=yxK)de-tYao)vQkdr{n}Zjc2ahqSvsfwF*uRX82e$Qjyb$W;X5x+P!{P%N%esAJuaFOx`Kvr`3`sl(+5Y9>^2%}QWA5CP; z8ab4yg#BXN!N8l+Dk$k^E7EJCm(Wz|L9S8#zmm>r5Y?l^EQ8Vv<~~LLJ4OP`K&h7g zLgnRZt7=~=_Ra7Wu}TvYZm`9@JMVvY*=(B)`y}@L%vq!FqWyQsb4JnZ+EVZzL=V%U z26Ubni+${`^%YWPG)DYGZy-(PRIbIGxOzgsZg&!`$B_fDK0W#Jf|LCsK?V=IuCGL( z4wgc;Zp+cgkksJ};KhHL%h?Dkaa{owRFv%M%t9ES>B2dvn1-B=Zyw{`*`azEFus>f ztB5kw7^?vn9CmlQlnEh%&rQ$jE&&+*{_%7G^jW%W$rh z)Xe^~EwDz^6LMAMjN!T~{p^+WZM^_PA0kL)C2$u&$*i3u&>4~2t5wvWTtyfsDhnMm zd>gcJtNAylq{Ht%6jO|t(}YR1Qhyn&3DXDR%4AMeGl9xPa@o6Su)eWeo%jqr=N+E7 z^zBY^QxA>pTPeXzsx(%O4h1C4V^FzI!Bp@rvHL-10NsLou9LXhHH$|IM@HvN@+xMTSIwo1MFKqU<%AxG95YRF8Du1k{}yNn*nYt{wlL-TO11RU3Z78n8~p!Tf|Trgw1(LPa-0 zFQi4{@&#|Oe6Ok$#e~|tgTR@t$CUknhvgMvdtv%!(%pX?^xsWkB?hjRyE^LefiE*| z!LBZ7>d)t@`5`=}q=7Gq6!{P|v&Y5-2P?Zi<7azSpiLwbEMvMP4e5F=fdIelBRFbr zx^rj5QM(q~yiSy1xh;GM!hH*CNOv*R2Ph$5vWpOH*s2-FB0AFa5YrsW_)NcK)|=q9%<^j|44_Jn2@ z^F16!^qL^N@G>n4;4FVERq>umvb{hg#prDa*!SYVnan%ZgdJlG?J^iSb5U&!ER40YG8NtsL9U ze3puA_X*_9g-UykvZMHcZBnVYUg%tv zozgR}qckj0yHw3uTK(srNq+^JF_;TSu*$@-GJl%BZwRtfYu8I^<~B;a5*%0TAD6BK zcF%JMW>ABPo&ZOd>_Jo7MuIcA*2YLxjhn%1e-^uz|Vtdq8%jjak1$KLYOJgKi-2OZe-TSwRt%&WWr_>yB zQ7l%zAE!U_^)H#VU;%x0)FkZRth@)Ef%97Sf)cyxplzk($s5@tM(#v<1at_37V+jR zc1r`7%*|9Js)H)=nyB@IvQ~t&<$RM`WONWvKw3jn}8#j4Bz9`zQ*05Q3s`<6b|K03qR1@?7L0#{qPHhoi=U8mVff==UagvZ+hCP|_)^Y9LlyP=p_X z%iixXZ=0V)cCp9CQs%}*_+xTaGskB=^{t9K$HL<;F|^$2x?aL3&B%4fzXM+LxM*>5 zfEBqfy3BXPtLxd3MyaF{u|XaLZ8efwo?Pna;-QGK*~nV5?|A6;dSW&@_fYlzY4Db) z8lQuZ6qdcS$pFb1!}sH?70?|k*CI2*|Ij|$k2Kx27fg$AnOfo=d!K7fC(Gg>yv8gr zEFUhJ(>V)LrnnP}(5;M5pA-$F4P3&UksvuXP_q^3oCa$KRW{OU8n@RL0DNQ3kUy-A zX76rfXqvL}L-j+~1m1HIa~X3A(Q9s*&2~RJ@x-Wi18#pqm7?9-%ld!{Tk8U zF5R#ETnwF<`GTJWjXZI-DCuhERVMtfs`^|L-CmZ%g2-B{8;Hd{AR4;F^dA~hz+(~W zuXqd+f1Dx`#wt!{YER-IInpr0e9AT)HGB_5w$GK( zSOKufZE}8SbVgT<9iDoC;L>Bd^r61LF;zj5vehg!kHDM!U613ssL z>?$u=9Y^_!ui5S}%KK=#_T4XI>X4Wmb@Iak=)Z>Nhaw9Qf<~{gd(6|0ow$~8NJ@h4 z`FBgw;kFo@Zr%-7vvm*uQrTi%D2$(mXEm<(E{E2Qtno2vk+hm>`cWy~+Bhk_hg*xl zSY9^1jiZ;e@c(0&Aa!~!xE*sZm;)0cwJrkHGTCc?y%o?6qdooL*vy?4bxawZDOw6S zXVle6&RAQDD>Fk$blpf+~mn4ZmT#Z7RVBWGDmfxi7?f!rowv4v(?gcABoMkXt% zP1a2D2OgNxQkq-TlL}rQeC3yGr63JBWBtMBU%O$ar9pdzo>c6_I_T=tf7}9*mP>H3 z<%a`JcT18Hm3s#?sU-9LmM2ePRx-26C4v5B+7Q?!>?mn(+aiDN24(1~_#mE?u;w!D zpnnYS{F(k4#J~}Y40mJ(k9F5+K3|!-)7OW>rxHm(h){o!r=72&6~hnA528bpOF-wQ zsBq(iw}*%sm*kG1m7H}+60Nf(MbQ$SR!rmHI>y{d4xlTE2?>^YJ~lm%E|!8lwbHQW z3R0h0p#y9xORzd+ObAYe z0tH{&9r3325HC7Gv~30k*qwgW8y$Q)I%<}0vAm!MFMM=sh;)JJZ;2EpP$?nsBWI-& z_2ohkyh)AGX5}M0G@aSoFI)_X9|2lwIHM*RevRoBHiZ?-?`JAUD$CHU+M$~uz&L$G zU^@NTDHF9)vTv*p4y^;~}P`=oOuGo)S^ZNiTJ7H@x>VCIosNhsj~U+-}CTkzn{CQ z6(>hE^zI!6^mlGOnxvy$8KSFJ%6qNh06T<#8VqL*gF0UU!WKs%1VjvVc3_g)RF&2f zpz{q0dup{@B8g^H+zeel-!YFiVCK+FpgDFemG*e+OqfwkV7yaRkANid zN)N_yRmveKw`NdxG&z1mvETne3p4uA({3r|{1)0_+7Tbwnslstrc;NjFG|%9kfSBT zd7aBaMC25Yd<>Dt*Wj7*Ci4B$ zXuj&*H-C>gt1Yu1-dWJy=U~TGOSGzSce_UFqu>+r&W>xFlBx&)=P;C&G9%Veuia`+ z0p05=QS!{Iq}-vw19riKiL1D=y@ApKM~4eu1Oh3+oEp%p);uF0&-I@m**V-fodnHo zrZ2Lr%L|LNnv!vFNh9QAT9q4U5LWV6(6%7XgiaA`KytR*j65p@TA))NIL&vf|*}t`+7^Uu4i4z5H+hshe}ef)D@d zBbS8QKV8Ma0QvYyZe*eYdgoki!IZ9h(=g*O4|)`HZ4G>QO6E5n;bj148sVW=cqjxY z!43oJ3Lle0Qb3Y*y+g|dk-#?4kDeU{X)XG)RNT2VA2RK~xb@{h(K*+sD$s3v6kzov z{%|!smDphGUd_lAWLfE5Os)Yt)m>TWMJHY{ zJK`-hBdV()VhtaVv!UcgSmg`hZ(Pj#UIL_buG?7G=14Kfr2`tXQWGqUn>S_@gBVB= zvaIom#4o(sFw0m#D&Q0hpkGXCK{Exn@G$!V4Vv<(s&u-oWd6$eNsg^E#k6AvhZe}u z>@PKmz?O)zI`lx#KJhNoCy?8PrDh3Il>AlL;_La0(*R8B`i;|(qdB44!t2!OG4?mNvB!5en7u2CA zR~?~!-)8oD5$sjQBV+Xbs-V=EZOwa_J5eW(%68fR@dOU#mFZq0->LIk^KM}|lhc41 zh*P}{TNo0(7QIFyYRovuS)&B()^DbVOk%JVo#-LnJ z<$M9j`(@0IOunbRo^*R%!%^lzLwD^a(%MBV!%fOL(yOw&W*@&y0gY(N6GQSj6hj^_ zTOptSX$TC=RGe_YhhQu`Y%Qa_7yG_IHFN0T-rpG% z=5u$FQ(@XHH_A-2x>nkohOIr*ocogz^L$No z%y#kb_wTTl`T!PCZ6;E~TO(qD9i~Tm*v{a=nwCK>rI#(_z|5a7T=ro`mbVJh0i*fk zL8<Vr{d}?O);f`S!I!%GYOMbWna3#74D;tNZ5p<_)u%oF=)fFUkU{ zE*T|8s_BG5jAu|ttMLfmSF>aZo|8>PmQ`@U4+$F}p)d4fe|@tE_%d>xt}yWq@ztPXweINE;&$!EF_(QL zarx;|JJBXHdWd7#)o;Nepz=89kTC z*9`Xmnx-Uu7a4YIRHSGzR}e%4`1aLvJZXkOGZ-Og%+CY^(~gDkCb5jrlPlIh)`rrV zeItPh12}+k_Ty|42fMi_V?vO}-I${@c+ug;l=Cn5a_%^dhYyZ6aJCj4-2E!R+#grLSpKx3 zpsi=tYmuEFO-K73iNGa@Ca^Q+Fa)*|4@A>!l`vd*X!j0^fbsL3eYH^_t=I%G4s3`I zi(?I=v$NV^d=eM5+`{&FP!GoIGX*X!EU!(%;`=VzShbX{G)_Jz>Y^Si8U!C$4{Gg zc^h8lOT+KD%YqKwYcEHiQs)svgRVZ^*q$-MQ|grd@7k^Gt1QAZDAV#;h_WhPZ) zb3x^J!J^7;^oD8LKyo_*tT5K!>>6uYCzzAtqB#8nC$po)h%J=n4BJYxkds<#8w`o|7@Y4 z$skmu;*u*3p31Gr>s~!ZSS)wAdj_17c7bXw*hnAdsZ4aeWWB-V_1~7GGKMgb59mJ} zGAg^FFTr5x4vS+*ND-;v<_*B*cNfeitYy}CP!nMyn$K^bfV^Yvz7;!7D+(uQ!xqq) zvsyP}X-P_z@E4%lLTXarPz&0yO%=r-uibWPR9O>3=wCaTZ|#S1a&CW%6F=VmV9IUY zQnxTo+ps5E+hpr~6nl{|Dp+rb8rdV9fx}VlOk+i3Ufe&L?}*DXDP~9%{sfEHa)`jm z=d>2*1FDZ?&;Np>`!P$#P1gS6s?)FIBy+SjRUUGJlCR^Bs_gyXGzLA5G@{>6z0UX1 zM@)MNZLX6|4-bdw&@a1|nUT2a8uYKyu7OX$tHpPP7(FJ~J%)>q8OlejJNBOj~=>9K~3D*Bb zGQr5k!Sp{?2_`~DW)}AU8U2486U^+)|CeHd5QbjN!rIxyk&s@@+Q8XF#Kh>o{&5&S zJ{TuwM-u}Z828Nqmj-D9X#4s8n>cRFi`JC(wLq$IAB#7~9lST`0{|WxpS?v&N^tt9QO{ zUpiM6NXW5m+0wzkB!W(0+P#oaaIiz>CWtW=_8I{ZSvM1PkGV1+kMZm*s)h(|wTyEzM>BmI~Vb zBRK$T7N+bi))YUGkj$(&7(ZZ0Nw+4TgBmrSFwP{yg2W_(9N1={Lrk(54+mwF@E0i` zR)!Iv@<$yFq==wU1OfRwRP>j`Ra;rTP^C)Zk{4Ysf`v+->;4H{N)$*;4iZ=anevG~ z1Z*&m1uwj0eC}B=K-x3?-z9BK6e6_yG`>Gjv5;gCvrU+Oli+y2xH*N2U;^bQgnv!3 zMGItHT59U3%%Y_^odl`F3KcJ31@f9dOQIyixr`v%GD;EYak#7|G$brT8K{s0770td z#3MOm4rRDxa`5%DUOyrdi;@y6COiwG@2XgY#dAji5>f#n_z*NUGA5C50E?IvtA4<4 zC_Y6S9vD}drPQ$uqBc_@39o~(X?)u?0D*QrFg5AZVC{+;t)aKJE!=FmxWX? zi+)IZB>xL0(m0D6HFzU*22&9U)6h(1Q1H^leravsg{YK*0Y##GO4{k^_vMF=NK$)n zM{P0Ug`xq2iz-MY802<@Or%7F+BZji*bejD>-qUf384~D$8!CLVUn2PfnUJ%cD(Md zW{PBViNNa~3O5V3)PPE7UjzX%1}N6HHKRm{ekBo~_EB7!um_mzRB{jr!Y1T@#6}4! zC=L*Ey1fNaj`Ik^Qkf9rm|C*zj&01oDMSLrYMKmxY&iSFD;-2eW-wkZcnI$O5b?;O zLJ-_uLNcHF@c0DB?0+CaW*GhosHH{^2|~fMvxLI_!SrDlp3uPfX@nNZAj_IuC=~x9 zM{t|u&mr8INN3eG@`l3oOO@%le#N!#>gazog_Mk$q6bdSAAd=}3`*{wJG_fAfg6kV z!sfvQ9AC6qg``1_KBH>PQrC5~SgC7nunssVNxtZKUbUfElDD$0VK0K( z$*f`M_j$H$=;_pZowSGU*R7bz#oaK_P<-{W=j|vA-my^0&ua1CSDl%MO^RmQ5akd< zw?)yvf$f~KKzKBcVq@~Pa7^3qqbb~fUmqc9p&m6EJ^DoLLOajFDUD&OYpkbhC-S21 zb!zh$7Nx3?=KKsf>FyTS{McH!nQ1@Ab!_OuDq^_pOh>o$S;;_%=W2Zxne?s|JGHUB z?P!!WaXfzIoxWAZeYR7&yuxFMm$vn!AY{1`wRQk@{>Iq=Ix73zNr&F`iI4_-i@ z(BABtCt}6vTUa?0JWcc+FPnCuYs7x>jOsdXnaA9CuJH|U*S9d4%2Y_SS2Dc^UazZr z#OHc5PyeGJ$9*1N{hsYf;}~Tylh*YarzHhB*sE2Ti7Sk};M66N=X6u8Ys5Rwxa@H; zCyWIVqrrG6-IQL^YNt$O2eDm@@;P-9^G<`ddD=%G`XcOMo?fpE`l1T+|5e+$u;SF< z8r!P67VDl1%0o*;-fVu%!k3E@_QAokslvuLBOw`OQ#or|TMv3r`{~3*nbdqtZR}QF zdq24s)QI5zSn00bR)Kap)B$bzfrb}ch>8eALosbz4W{?bRb#$4tyw@Cc`7No)$4*F zYepHp+F5ZLcvr*o+_^R5)aSH8S8M+mPdpEC0lD?`QBrmV2z zp73(DyWt{68QT;mmRuXQGw(iZRV?B7F+TiLP0rQa;9~5gUmaNcy75jRxq(_nvUzB| z*JFVy8|5Hxigj5!-N`p#c^rF}+f1k3`CnI+Ai}#6qPMjCwj2#~TQxTx_UmVV2sy$W zLBuw)quu^1zYKLHJv~AMH`1GFQonhl(pQBV#;O=b+uYXouSCdAr#~Vmh23|<& zm8R*c8ii^yr=~|9OC`{wW02d;gikA5?&zyt-)5EeM?nN_v8C*h*=#(x`-Thl!v(?N2ed_Haj~s+8MvD z+iAUp*UI+0wFOwVWrifT?HaF|vJThkeM*37okr2!a7K*A?gM1MDClbAN+K!3W{ z%3(8A?tY2mjHOWhc?5$>&19{Qb=35UICw2^h`t_9`!z zU^3~=5Dl&`LwLhZW82X$*06OW{|c`z_wh2H@liND(RnWKou-{xd1<3jrRC=L?K3gl z3xw4{fke+~!{XM|*<-QtI_RY(#Szo^Oj+S!rU^%JxuN70NCJTFgAWt^FWHU#f5~o4 zZ0sEWlh&9BnHkyH{_o-cJG?P7Gcz#!{}qIJx+o{_?6MI_x}DAM=q&E+%m4{b&y()# z5bcDsP%Oq2CohuHlHx7yz4P30@||_x_3arVt1oGb#I-h#k0c37`{{mGa_NvbT|F4I~On4H=%61q!7P+zLU_SWyqGOr$GM4Al<=3`!H= ziXO`b6b>hVQ;smdkhri6GHy|MQEtW9CuY17Z(%%Bygx}crY#hn*4xU^jmyepr=0TdAoeppFOYuAW8(awLM)9d%qy}_$Kw`q{;gVQern_s65Rm6In@tK{QyGQ|lxN)TSte&kqUeEwQ zW5vI6s_{v4izX^zNQCK1kbp{1Q>Qmq0QILEFSLaTkv|Y5fxl8bWIr&TB36A1>hJ{g z)(o3y}J7 z&4Q+CA??EAfd_y&iLW}{M*P^=N_1I)P(aCrAEYKn0Itt3^K+ljvwzSsm2n4#b4aDPD23#QYjbT*&+4ljmj`nN}mtv9*cw7HQBL{5_R1fNJ1)|gC^AD-2 z3PE6kbVhG?_W-uzC73rFau z`+$4^T`&111P1DQCLn>*Py1j$JOitr`VrIya{b^B$LS2_htd>%K?qb;_+m%)KehN3 zoad?G4~Oip{ecwJxcU~9&-L^}Y%U$>OCVe}dw>+|()w@8zbZds-SRn@L#Um4KMi+x zU#;xy-(x37d!Y3i0C2$-#4nDcFqUwMYqR~QIGD2xn;#)Ya|ztJoZRi+{F5BCtgSgH z&KN&%-x3jiEr0?_rr`%N*s=L*2(vOOC$0$(@jDKx9PSwbYRB-)PoSI+9^fyJg!|vh z&VU9?S&9z+6pw-m{c{Gu5JC#*W7r=StVaUdbHi*vd%;LHZT?#i@%-pdT*{MQwcj2d zAQgRc3hVrH4dr~Vz}s&_Al}a!id-2ljbHc^#{+2Rmn#$ydvE}-2@$ySk^0XAH8Vkc zKlYUO<417NrvorK3%&%z?HmM0-n@zvl`p{^|Aa9%H)R!tWbN=H=eq!sk3wp9cV^Mv z#aVRAC2w`({YV6?+`mDED7}6`g;?J{$Jzy(dj8w|iO(-^!Pmi87;bYS{rtDy z0^o}ec*#F_cjEk+gG}kGPwk=}1}=we4E6o@B_19b(nUSK6%P8DydS(-pa=MzI_Afo z|MlqqA=XkvKKuzyf%N>V-T_wNhK=BSF^_fl28h$H>f>krehCiTEd&hq5v>0wKEp8& z>E?iJK!$Ic`$m1>6DT#VL=I4mzbVHCwDcN2MvYuoz#|J?49ZaDt6SJh$;0bVifHJL zlMwP^YS+r)gr2=OuV$(;dsx&E^_n2%BMSu55i_dhPnDu17BwO&Z-|bx*Qv`}-^ZR= z$!a;`a0-}R3rN-%eX-?Q9b#Sa1WmR#+LS;IR9S~MVXdtoZFvDFwI>-HRHCf@e4w*4 z&KAlUCR}8BIDq}8A4;9=+Gg9Vw=*?_sfXU+7U*#>{b{4KlH&TRi!R*ZQut>u4r1|- z+U5l4xngwuU3cS$3hd%uA?dt+qQq7Qe_#YgN%8rk08&IoeF@era2vxH;&5|{D1%9S z%Ev1Mo<&GQ*0DE}6RGvaaAvoVLU-^&n#sR7yHI#!lp8JJ0XLO8PVXu%#w%17!f0GB zE4(igeFn*BaeRO?Y{%N(h!iPq>^gFsNeIhakhngPV0l%-cHM?i=Ac0tYT_C^X&>%m zwo=acZIB9z6DV;(GLintv?!YC>ZG^ArE9W!hmh^2|2w~&u8#@v98cpch$*jOPr7ux3IyJiKNZ7+sHzqyZmdWxw+D8HJsq~mJ8)krs1 zpr4tQXm|rv)jO*>zhjzZ_>$;_IoHgcnR}A>pgBSgYfd*=(Vb_4Co@FP%jdsFVuNjY zf*$!a_hg=m&EcqruddJudg2G#=$;tJX3R>N$vqrH7Yrl(-V$v(N*=UhzT$)Thux4a z2{KaY%6>#(JYG?)BT>-LkImARG{f!Y(!Nv^EKUmCxYFOHg z4%Z+_rUykV+N$p3)^k4GF#G4L-!4gb7qNahOSYPylCUyA`;6PJCV?T3ME3=(Zw_ zm+f^%7uc%_IoU4gFXM~LFRJz`^A_TUT$rmCx5{eablL8%R(K|}AlmAYdBar8&dC>k zc{ZOX=2PDXTEYd`zEHuzbl)V#yrioAjbZHVZ1eB<)AfwJ*U45-D6OJlVHRFk#Npsk z3-~2Xd0})YYl%_=eRpE#JNl zQ<$vyPK@H3>CGt$Hi8+m6e>sxb*9|VYjgD{_QN(jy7I1&GE#P9b_b#Co_MduO`=mf zb>2+SF;St6^B^t9g$qo6OD|qr;t~HvBr>(WxoXu;Owus(iS{Wkm$NrHMb;v{n-slH zLJ#6F(Tcq@sG$&q9a`q(_W0gnjMokT8CsT9yH`iqf*U;j^RaTI5-R0mBCPEVvJG#s z#HUdX>`y?F!l6n**60=x?Pies=zS+UcsnGGFZMsSff+g)%4gJphBySqzIaD&)v-Kw zX?B=R9|og80t=m+yyK?0CVl#8CQKb4&>tbeD9nZqBalfrx7s{w0-}vrG;Z zr!$TM@^Q8t_o7-|QkNGyK=eesTSL6WK~<2&r)!_kZV>ydd*{=3DYE$re?A$(&O`mh zQ2YGZZ<~6TwPRtkwv}@+lnatD>GF+cWaaE~O&9Kykt)$4OvJ5Zh))p0SoxAhnTJ0z zwG5XUXM^I^%pKVIr}xHnmpYy?-@5kE@-v&hYeQlqv*$d7bg~-Uj;{u4;e5VUo$q|Qk4@R!eD5E}^mn6p0-}E!dk%3_ zGg@Zow3LoEUJo2k{hf_QDDIb?OW_QMuZ{G_5qu)=_l|Ut{fk%Y zm*O$iSg>-&+zz`6S^m@;o4HXxx2Xvv1ZWw;xq(}&?Mn8R_R+2+V6*oEuMjvI=t z5}$L})TcA>?MOH;N%)FC)xGadnnTJH7!?suS+`uejZ?{2sG+$vm zP%kEzH=fQoubQZk;-JIi;OC5`DG{N~n2EW%I?IeviJhu5%Po*hJn-@e)rUhp3MzNL zt$VgFVA_g>G%jpry6~ThGQ;@8hE3kQtz{mqqnUD`AKyg@p2HC= z(kC^=ww;rzR_4f{He(&(=-DJAJ*+pNavbCdY^2C09Se8UuIjtlj3gWO<&N<_Kvr@9 zl}0=G5QZgcwu#C}jn+~AlKc%8M<>aC?LT5_(G{=PiYyZ+Bp3t^FA$*adqdqll>@4c zCH<(Blj?{yG%GY{)GpJUZcALTF$McPvvltnMqc2uDP-f?EWn+(#66I9rrvHBX?%7> z!{6YRf;t(j`)7QZRt5qcj&DrcslcP(fz4u+Q32$Tc8VPF8x@ zRoJ1IZ^Zs4l>T0u`IOT(Z#_imfq;yLZ!Y}Ka&qB@GE zNqDky#(p$WzdqOToNSPSG5(k}ZzypyRZyRJk`7tRs&}P|n}I0m8SvyNhDKX`WVj1= zLhpom>CvR$KwGfj5w@4=>10phdJu;SOBb^XgI9fDnpsSCXx>@yTGc|9F?1YQXpj{2y&=XPqbkUXwTxdOI~(Xx9kKYX2MvCB`zF;T zGBjVKoEGo96A|YoWclAjjq1-x+Q%ppNGe+1X>1o)JcX-nmMX7Jr)ZxswkCc9;-Iq4 zFW?a7%~jaPmOqrv);staj+{{~%;}gCr!okc1UWmQaO}I{u@?p`ssnn^3CvuF7bDG# zdci#NOJrtWy{a=0bl`kQ&YhN_vi*_A#3wK_nQd|1Mv4IW{xg~tbhBh62rVZ3>hO7LAk32e_(XS4-YVC9@932uU0S|S- zhsUPfc~FIyJScB@!P_3L!WbS9;DYhd5!KHyVCuhN)x{*WItNkywdk<($)tj}cr)cY5Y_Hb5Aw5r9 z-eGvSI~#3Kj!b!fLiNA?iT~3&q4#ul-@b<{Xro{57L&CP#YW{_E?xb(-Yp6zy5l~F za~(WUl)l3a5URTQsGrp3zz)N3FP^`Fnxeak)d#6%7D!^|(xci#^UOO-9dMT3MmYQI z;l$Cpm(4|jD)TQET_#!#iyKWBS`O!xW}Z{ex!~|@NVB%Cj#BHDy1;jFT0*C$P|)lA z&}V<&G}hgsfcL%Z0U5k-*B}yM4$zy@C47_e>$#9!%Ku?deBkAmEm7(aJ#*L-mq}MI zQq}djcVJ7Uh}Q>pbBVGxqZ=`^0us#iY3^xgTlAwI#2U$!zd#>(=Kgh$@=b=Gl@V#^-avRdXjH)kR$6QOW z&DC0y^eukJ!OwAJu_zedG;{PeeC8%z7b67k26X^?lYPhvrd5=()$!a!s5K)CsfyxF zui&Ea7HH#soMl!?-^e=0=s`L|WQ}vy;a(%>tM3gI+Xx*!eD&H{Kcq=ijXmrV(IDg# z(^0ZKLL^36&mZ`d38Fb4a_Ra^l)PwJW0v0O${x~qg@S>i$^{8qVos8{Xn!T6?{>8-+-2Gwa6 zy%p>MwZlW0HP%>b0@8Q2d4g;A>)9^XEL!f?rfgA6wX}w$Y8m z*x~*au!E*C>nLQ|j&YNBLSyJsR&&pQK-Zr1a&EEA0>B3boFVPhk-rnVP0n}r#>V_^ zogQhUy7(DFtV@EcIOxWg<$5kA7=aCDjEu;xYN~J=SGsi~&2+(}>cZh*`SoVOuQEvj4P!V_z zo8bWDDxD0vCF{=Jb^meeF~R7*o3hl*WyAqD3|e0LKc^lm&#wykT2%&w1=FXj;G}-n z>mj!7PqY}QfCpr=n)zNX9#gsk9 zMreXrgdQ*t6&S#9eL=_dPg+S0bzAFYvsd(P9)r)E~v9?tzA06jp$zjEZ2z>JKNE@2M`BmqAW=lkg%KY36Pr6!ys zz;)TuP1Qx`u)=f|aSM(|zD19S=P|3o3vylBA9{Fyb707CAC&Mxq;(fTREX^{D3h5> zAOAM1iZ3kV7_r^~J8-?cd8SQk;%by9_t|}5(+P|z+P++Wd3#4LO{&}CEp!Sp4e zKsYfRzNOGP6uq#&!!-W(BK`)z zT+YV#H$`j%co&Z(PLsQ3*sZ3v-BHMmWA{Ettzup7UAyTfuN8@8k26l?{XRv8BDr>R zP%hQtna9GgWnSQW3BB}V5JK%?{^Bew3MPocPrK~L#HBNROnJGwI%D$f%8t4_gzu7S z9=!f_keab5*E#GuOu@=L;cKT+$kustN>2wvPGYeNRRv2@Ij@ZzJCJbzq#I$-=l%YP zkhaeCwnJiB55|oYS-aV|%sK{ZfEx(};XXYHTNg3!a>fapw;7GAUVEs^a=)T9@|KHv zw(Q^HVehlA)Sq!UdR8AgfKKxldAUgdQ3K_hu~q*IxRzD8J+YUAM&De{x8vkp*ybiZ zkbIb@L(g9)AxZQOf1d36!8@O^1uUw+M)^3&fN~h3p1HLyZ~IoEg7K1o7!R-f^Kyz8 zA7A>NT~aw5EquUW%p(EFE*f#NuYEWoQ(As=jVGdOKRz zoF)8DdH=_CGI#FaRfb-9#NH6W%KiaA%g1`Nsz@|`zZp`~U` zqm>sdM&K+3j53Hxd1yG_#S@0Fk^}xmVpar)yus}G13Fr8RyfkZoCYpZz^cI~^Zu4B z3U9raO8wK(MJ;Kub9^O)q=87%d}#b#$18m^-zP7%+CFRcOE1rK>W}-gZ!0JGSNO5p z`Z|2JVGFb1rC9$&wj&j$LDvT+h4&=Os~w$Yg*b-0e9~ZwKPO06H9asmK=r^v;tR|~ zL6_Bx67zN?y!M>KnAJrI2&ocgZXQS*Jj3g zdRG^_q2amhv2SwAUy{ge?99GpQanC=AN1C@$kJ%v3Z?@+8_#`3{-?bi0?U^$jkw+C z=MhggCx4ENn7sl@MA^gx1|%KVoPk_}%`B0O+_$gGsFG{cxu>w~YtoQmX{_?5${fqf zR`roT%>cdkwgsLCNOqf$TBMs_>)Y<}9+R=s(Z}OLG9qyG{o5b3!O^)CGx50{bh}Ga$w!{{v@6xKICD`)dK_UgMdnu&8(5e$&Y9*?fL&aX*=B8c{Tb zXELJpZQlZvVth!9w0HIwV`o29*sV8w(Bg4KPJM7>mb#}xMg+-!`7sQh6dL!fh?5q# zB*Sw9=iN!+&7#Go;*vau=as8gy-SP>m&sSiGVL)XH3@&>GP*Jkx5N{jE6kT=THf{A zy;8pwq;>6KM{9e(!fwNGjhk6>Nl@8U!~YUMr{3~Y?+DR-)1mbBu?!g~_PgkYe*EOt zm@%LqlwMUT08d8~O~w;6y*lkd#^ukL!KP&_fq9b;KODYx&0G75WFbmy{w>etoLA>g z02ik?10s$~l&9$^i)Kh`d? zYgQ>~k(mK(jsh&dv?*>jr78pQuXx51RFBlrTV#da_+0W`6U9B5m>DCvq^wwc^vHfn zYklk%2R=BKJHP>$A1i}QtphmEqI|{3UAD9F(Vbtd;grRhVV}LZzAhDWFbSeS3uKTMN~hBYB{>=#gI%qil9mPF_$~nWZ^;kAV-?%G}mR_?60W&oMj0GZ+GaYlGt`3Z|mDtKo3O!RdIYkR`jE7UfL1UK{%}&>thitvD zh-%u$#=J}Ybms=tx8A`yK{p*XZ_;WVUj|=Q)xxy0HaVL!aJ@C3G@-?L$>ePDO$0=x zN&QA04S(@<{^wmGHlEl!=Qg6K;GV@$xCZ=Et5iiphP~rByyp~49I;%vofvd=-2Fb= zS5bEu$i^Ss{K}xWZU~&(3*i_FDu?ATS;5b}m7mnv#<0pMjbE*{uW6mJfX0s!m#n!M zNGF17G71`P)xYJok&19;Qi;5MXT}wB`I9W^@s?@iykrrX)Bq=V8lEhL=1eQ*r{C<(U+=0YtICIuGq4xpeP)j=y^(}n=6z5M-y--3U(D$Hp=_^LU z{3~}xPeq%dx!S!<*o;VPHsQ1F%i0HcxQV$Y`zBj{ytq?gklJU&S9wDGGK#t+(m4I` z@5J$u+{EM-vrwpmsp_01M9E*G06N0n!m16{C2M=eR)DMy-0HnJ_}W^zIfVRQSr+Q( zD8`{9gF{V`*CO5_HhAF(y&IPz`Q-nCZ!LOes_T^<9Ib@mr$%Mc#I=>fcN0w&vQ<7S z$}=y^evXloITYOK5-ey?r1j9FlmbH>l>uEFK0=2$0W%-Q2gZ&g33wV;OhZGpeElS0T^G{>GuH z>&u>+-dfXX9^sLzs|sre%z3Dni%UvV@ofYOV3tzJV%&jjBW)RJoi+^_MV07iH=96G zWo6d9AuoFIlK&kx;@Ns>W&ZIDILf6nakQ=K$2<}M1({x4sdER580q{5ql8xSnLKpk z4p1JJIbB=Gg2}z;BL?%i6QNmtTJuG!c|7Ikxe4AMv$E)?M#LI?lPqLwA0h4^uT*i# zC6rs&#`OWVoS%iy{UIS`6GzH_uM^9lG9Ab(^} zzkQ85+_}6u+&0w`C|rid_Z5lGCGndl?Jh6o4k3y`jWG#!(T}8?p!|l_C@-p7g_5;3 zbA$rb<1LSeF@Vh~-ITKUm}2L^M3lu*Lz zo3NGWb*6KY3D!3_w&QjEQ%UJgqYC087Fv-M36HCOFjWx&iQ-7|U+|AQ?IkUk+ zYFT=IinQmx$jwUWz7B&0&RaWvD>n=Iv!KLT2fP%KD&$1b@m}IF;(!r70>!L8&YpJt zu59ey&-1hWc7RvE>^@5`_`pgve+tHZI@D0|+m(5${`q~w)QS^1c02zze!~@0(EvwT zZ`^o_!>Bvb#5uBW^#E?mwdnE9s-7>s&|_~2;f*inw~(zQ5YTssum@Peb~747aTSFz zXpyn$F4^i$fn|i3Y--BH9h4WEw%U=uAi^r^xA7Wle+HQbSKbtVQ5wU`ey|Jid;o3_ zGTzkl!G!kLVhjrF!`|+B#8-be@~w#3%|4svREGJ5B;-0D0g#k+qzE|?%%@^uNp+$~ ze+rf#Oc83Ypb*dNje(b?Yn6WaGYbZ?)N&^!OZG1FOCJv1tBDjm1W~Wyzj%}w(msfY zXwA({`z*A&V0b0M*@8-v&YJf)6G3TSl;ujr-3UN%ysNJ0iTv2p(O0=~27T+JO^O3( z`thC%v)5Z^9LTZ&IKO=^-ZOTPVS2{mN@P%vFcY&0q(Cv)>KW7iaXK>gS!rNg7U;Y6 zeBbzDa5hf_xb>sunjJ4XMbfeW|9W1<8xQ0C>mf;+6f>_XT>NyoT zFB|JC#tT1cis#08nyM-HoNKEDc#@0XyQCp`nyVwhHtJ*F6l=ZGGSlcVvKK~V*+g8R z?wMj82jurqFdV%hZ8ngdGSEj<5N!BHiHFoy$L#ZizfS4n(2@!}_;gAs;5+_dl)`ui z3*ES8$WtAoTG(XDz!!>xoG)hcpVdzVb0=B?-`!Vm(*z_T(N>u{9(y#n6N@fVBP;e9 ze#nW_T(c^4An~NIJLVLv&Jf+n%lM){t0=%o+FWg%j*R3!q&Uc)vQ%M9(a&YYvNELp zWFA~Kb*<8yv%wcM5kb-(+w*O}x)Qe}^*SR_f(!<=R;4$=pN!FR8{Kagjsi_-tnFuj zTzRYnUH*IwP>&F;2!@#y=2z%G!zYYl$&85+Vz=j2xVK|3qA@S#;joS=>}~QrhkSaj zFBi?0QpAGPKB^je|Dqq`)jxN4(2`-hk;&`DwQ*t4=0m30$ZE-vB`b7WA=$1{1!yJE z61v3y@Y0m;8H=!srpdRk_ah=6Kp|b9^h~8zueDJ)I)hJ~o#SKF zb(QarH6anvUT+qVbtm=VWGhMvRX&C##_hw=Ub+S~JUct>eqNg(N?7qVI^3BvmHU{c zYdTD;&E0o4qQh{g%HD%#XJq>pDO8NeEds;M1~5Hjuct&*fS3G@pS%3K-hH3@Q#SS| zTYRICl!IZAsMXLWcembJ*`qJ|k-7U>Gq);rd^SxERrA)oYTK`RS01BHa8$`(cp$gCB}2tUl-*8B^(}h32UJ)Y~WJv4x}-f8ykO zX_y3MMQ1Zi>ETC=D-H?=#z*}baq`J7xz(}K5qNwSB~&3 z;+9L}yV&R`GIB`v(zLJEy;|F$x%WrRRFj*z*_VIi1L2@K`2~#hT{Cu7rvWR@Wppe>da&e zu^j8zZ(6aYUlB#U-Baj4Mc;zO4m3_u*cfr_QgN`?$Q74RKun|L!P(u3zt(gIF&0a| z!=6%32g5E4<>Wc8tEH1RN0h%vm&8?Df(`vkrybhA^zRD8`6~0VQ+kX# zvC?EQwkfmzWUlm7lvrt=f@RIvT728ad}qtyNaBUkpjUek+o|;}s%2&6ela!T0>C?aX`@dzr@|Hm-K|(^7ExF#eF7HtQS3?`j=q z7#1TPk~yZwVpXpbKz6xfc`JJD%(tizW79Wl!j?o8jjNIh_sO5jSjC()uJ@bw6iwQz zQddRJGI7vM2;0r3Qlk1pzbF)Zl>QWqiR@d|oiCLgWSDl67n#$z^W7zC#s9mCUx*tW zydC}YFeSMV2h0!R)8R}j5E*umMgJ@eJu$!X^Ndz-z?}tOp1K`V3Q3`Y&WPpzrE_i90|Vop@renoa#tM}E?d4I zkHU_WbMdh{Gu1bUF=XdEykmbD!o6_&?=lf_z^jW}=+1w_8_9N=fugTUr=25a5oj7v$qJeZ{&*WyN z8OhG2=|qlYB6JlPiJBKnLVEfAW`P1D6&Q@|Y!e3raYp0{r|b=JmHYU_+AFxH+>QhYK^dX0LQRct@t4d{T&eLZo* zco<=YBf5eOKZHul6m@oG>MuO8;bM50+5=j*M`uWJCON(9^1CsU(ogV#(SozjYy8 z937D|kc7b3Qaj|pH6PoQDQgOlnEmwOT^q5kU6!Lf^LmIbtHLRw>tGtxS^3_FWOvnx zUGMEo4tpgyr1Tn#@}SW;^6u${@ zzA7?{_z_{S^VOGt{qD`Tno1bOAh4%%`&>f4X0d~5ZED_HE$dq+iEt|wroXCi%)>4A z;>$Pmy%`h8`J99*h44KYRY3wQEiP;~$h#zb1KTNh#p|AR?ym9F30VNqgR6zn)qM)( zQqX5cwSy{bzll-Z;P9v1nde6U%FPrjeG~AW`SYT~;;La~QyThmBu!bQ11zibmvQll zzIT(}nod}69b8-nRM^;iGz{jvQbB6ARE5PKM=Z;~$v5%MK3jc@BxqP7QTSX+j_DkU zp}q+;i>7beKVNFKqx!XB)J5AWV|==a?S!K0z?->Ot5g3VOZ}?=CV4S2KQ2<;8NRt(eajvOW^} zsynUAS~g4e<+5b{+QCSZ;S_^~`OQRdcU_DIs=>A@uRk*h{Va-ELTYX7t&DJ2;;hw2 z3+_%4tu|v`U?0tkh$WBh5rwrE@uVl6)waf(LIzUHwLel_vwv~0w=}IhFbGd7c}&C( z>boho%H7vh3R#p9T4yi)>NzdmZOpX}>3LZG>6H6RErj zR9k!FhM?c0yRn%^tHjH4rMfLLM|lL#b@yVhi}}Nxu@b(RWnTQ|&)dXeBJc#xn3|w9 znNC{|We|h~LttvdZOK9qu@B*@GY#;3jBoI~j$aP2ow*|haQC6$G-;Bat>3!*zH=h%?0Gm+9|7;M8+DUk>#fT1a-8&CKbbAO$oNEhqIRK6`}um)#C{aN>uGkB>Sf(4-p8V& znWkR-L8mdK&U8tKPy#Ld-OZ!s&fO$Rn@6J@#KwKhIEGH)mFmq|>S~Q;=3Ttw6B$_+ z!w1ol?nC}IiDjK=79V)8a-M6a42_I$UX*!tQ``8*d&KKL3-<(%4TS8&2t)$QlUCAf zs`!*cA4?8?_-1s(2mVrtct!HE8?y1SuX7NqjT09Eqzh6>NF1~LQP{QNFo}c#kJTF>tM|PCi6rVg}*05c-u5 zI53jKA(Au!nIilgbmll6C9czh0)>Z*EzSn~RNaxyA<1G&)!JMBTw6koCOSp!cdsdK zEmiLFB0HseM|)1w`*PHUT|(AM4dYqhvJHL&dfKdbSMgTSvSs%CVi9q0$R zGeTAQJaAyD3NkJ|_4_K%B@wdA$mRoMbO*Q*@w<_AT#rI`;LVQL~g^SmiP&oADS9 zo&%i8`3@oLHXW_ml;t#u_f3Pou2l&Q-eVOXXUFiedNpYehGSR9$2A1ru|vITC{ z*}?QhXYK-Dy#io&lJ44z2g*rB23dT_X|HBjy?EjFSw|B=h}|7CV}e94ohGK}tkT`> zl2K?9oTjH{D-1BoAzRVr*>dk`tcM$@^F=#I(KtOUtm{cREIuogdeo@~YA5p}6L-OHEII{Ilq9@%%PKr}i>b0d1~1IAqork*WrFIHwV0w$c-WG%^LD322|o%tRb~h|Jb{6r4BLj@JP(ot)D(?X zWjiVeogW-&Y0rLcvFoq=!)WSV!o3EQp@Uug5Umk6s^aeo43lk9-{T`MnBFwi+qjO- zuDea659>_bqSsfyi9Xx1pW|5jYT;|!d%au=E6^T%-qK0##KW!GRxXZ6C$$5uEnZsL9D0~V=#aC9GO9@?e1xOP(9 zq({>1+|qspCc4OLcpQkS7s-wi(&pUZqnEY5Omk0Y^z&?$a2_RZD^neH)|A0o89gYH zAaXoe>YObWIHI=xMp#XSL5rmRK&6L>5TpK>N%T??{q2jB;!sp(t5-Z%S%TE6T8Q`X zJ(7Z%mEwDk7w%r!=X~6@4{I?_&P4@wW)w-;hf(7_W`2i)8$&zt;V|sxQVV^tG{WL1 z9n7c)c>0RlN=A6(V+gs~)ps%+^|Vd2Pfe8T4vK6mB~BR_F`5hXM+jn{G-gVlb#iQA z#%dn26MJ?X)~NXKs&ViS*sHS=^AW#iG=1b#E)ky3gq5ttY#v61k95G8u{X^jLmD4H z8(}+Ih$ol(AnY{{iR-9W%0uOjoxxF#12*1wDXU;3eLty{PI9Rxd?hYnq_J1fyB+0E zggx}!NcdF<IG(AfRvMD|yo_jyE*;nFM z7*pFy*=uIeHy`kbN19)b^viggFswme*3VxjG&(=C_h22OQXv@O`2;<^x~F`U(W)tP zH{3g>h#Ak!GOQbPn1nb}KT740l$|#I%)&>1?Rm`D5PpLtjqCC=6F~KGLHYDm?_$Fg z+Gbu%;zO+`fec}PA4K%TbjEIXyx9y^zwW)bM0vhE!qJ2}1SWA?WC}x5HcC8k3m;v` z?sMbWK^Ei(>Gkx!WY@m7!Goxi=2t3|ACn~-2PQ^RkM0+vI6`EVAC4q8hT9Js1P~Kn zb5g^`6K6P+=Zq>NUr|SOrCr>9sNFEHjx5jv&{Y{=D+n;YKQW>eT{`?c0iv#Hrp|y% zsz`c$MUF&rFqZJuW(>#>EnUWoBaiQfm3A}28VWL9Yce>Uskq@A!PA3K&mlh^015=n z^V?9SpQ5Q%+7uT~wwDO$KEC3NZv6D^uqNr{N|gtq6uedm4;RcZX!UdDx};V$4Ti6E zRsjuVCRg`pgXt@Fm7v`_+?39p0@+bGi1gG^^iY~|)?m;ZI|NNdr_wUBB}OaOBm}N} zm6YCm-Y6|1--{Dn0>=2kg*SfP{-2%-5zwM{A$T=&=APk|5uff4VVa+cEgX|&!j``D z`UtgElONdL#1=aZ$61z@cxvo@K2naPFP%21hxZHG!2A_AZjbe#aOT?aGMV2_KYP&| zo^$f4Xmqk)WT$Ne`Bzn)S40DSkWcHUuE6p6iAJdnlS{lj+XO7WE%>?H7$K@hJ^$jp zUtOKUT5(1D}Q>HcNeFs2y`bMFg>WX1Gl8ZF4wz2z)StTmEWgIGc3 zGv{6_w_PB}-#I%O{o`7JQ#kRE=E~SY(ws4M8RN)e-N!x(yiL4syHWZr<;;9Re7`W? zDcZiV%ZLQ;#tdZJ%*NKS@-$oAN(G}|U)S1L>O|J{b&a(a*lQhJhgqvlj6igszrQn& zI``g!>op7`D1dV=rrp;hY*<%N-pl3$#Kih2^*6q5i4IKAKo&5oYbJ`?)6bxkmgCC# zW{&C|j2Jh_+h!kZh`|$;a2G#{L(=Bs`Mpq^I>@^E{8;{dUz!gZ=oCj1C4Iw-$M4<6 zU`dxnd+%)nU55@GEy`t3q6X&Gb#J3Q`jvyvBo|06GFZeWzW$|<<#p%VIiq$fhpyl_ z=Abs_+rwt?8-Xx!)v}(NWZdwj^Q^?}v(_A7e6>+kxo%PI2quSU@+|7;#Zs|4zAQhU zA?>w8O1GzNJaz6MBBj$V^`jW6#i@*z&&a%!tO5Vb+clbBV-n{yaaUSG_*LTJbQu@XRFO`%(h{gG(F z?5Ike25uuH4cyzl5_JgV?XMb{Q2 z1uCWuBXt5_5(ys5usGsRF;5|rjwbngyv`kt6^-p+shZiU83OQ&c*TNguVo&m({M@p z8Yw7o6Zdwqc0`E5denH3ZSAOI;y5esK(k()Aa&P)cdBStVqX&4S@_<)eC7IUM{e@x zQVPod@n0Js)C`4vG-PY1CqJ$d9W&6Y>~^hQ%2HbsPnM4fv1$K^WJiQVJRs5Uzv<1z z{rQbWSw6To=eud_3?lHnBiP&sQ#;^3T7>6=lp zwP+O~XG)$@>_WnDRkh&&=WwdI{{y9G0}U_P05wVBSK-fK#PRR`B9aP`0q=38QnAZn z%Tr=Jn)dPb48B!!8hR*WX0_ zgIo!uUMtZinmoK=XxoA?)>uZM4^rL#YiSjZd%E#!PP zx-0R5--y|kbc{>^==C*dz5%-ewgjXX_JfoDo0{J6b!DpMUPVhOSCw;d=V8mOA|KIp zSEHiaaf%0h-H+IEWepvxs~=mkt+;iPuc)DqQ;$Hkd5pE))WPS*cPwBqa|w7MQjRvm2qn0EC|X2r8Dig=tDKsCiwbdqB`^EZ*bj+y9ky0 z#vo!u@wXGO+&Mg<1ote=Q7A-o4W(@SeTKCu;q&2Qm#7`EQJT`i1``~Jn{^}y2Sv4^ z2?UF)56#gkv%oKBs%#nd>9#z}utfvAx+fWez^~Xr8LAMf_if`Uu&NTxts$0DvVGh| z_U;J1i$Z}@A!HW%c`526>_rK$>yEElj>HnVJ#En`Ssh;)N`n4h05Cw$ztXdwz2H;C z*GM6ffd!O$nnq?0mEVOMdjNM}XFo=U^(vZW)zI^ZGg~FfGXK-YOgQWove8?JwAXq> zB_#a`Th53+*1eNPODh7JnJ47tHr>}rC~JVF!X$LGpT)qFs3no|JV(arZI~lWXPO+L zE@Vn?DZ82D;s!Z}9q}dx*~#4ah7^_|O^cblv(8UK{Sgz+DYZ|hOyt8RBg#-CPh!S); zi3l;2I2Pz*Hzhxym}*Zxo@q^OR!MaVp7oN@=9{Ck?s;H%abbp4qIp2?8oOH|q&*py zm$}}5sXcQHH#w?RVJzfz70;AmWltwi?O-%7aGyII3;YX;#Lw5*6&;7TBxKfb^@Q#1jR%E%%^P56s~hG-Oo6VEhvLRf&p&T7^;JgcVoQr1zSR zU)=UP8P^p1W=AF=5jxkqBn{`^y&o6*gqt?#M&gCU6Mkc$3O|<+L+{WT{Vy-t^-FqX zSG7Zl#pF&XmrM?yDeQ$xd-Phtf5C!JIYeOm70JzW3#1&W#xc^!E>M4~q*T}5(!kEj z-y6wNU4OWQpQ*a_AMngjt2iSY@SnGo6;`QlmRuO|e2OFyrwrkS>k--WM}Ngk{YS;b zAgzk7Ub(yLIc-{vFDMsc_&G1{JXe}BFEn7*E2Ghw(PVo2%EJ`Ksl6J4K)j*xg`hFSiRRPBW zXFF?hI6mCN)YPA+yzDd^UrSTX3WpF1WFMLajRw9;u{L>f+L2a`?Wbo}7E$!W%-N8NFuX=%fx4kGq} z?H2Gi2&9gCn$no?aIxmERs_>4EX?_~@(Uak!3#?=a|Sc~duii(IG*X+wf^VUExw=Y zXFwjtA-*WT*Q^zCNb+e_^?;v_@R|$|He>2xS%dt?g(5|E>FFq-E)%3qzXq4Zivr`v zj0h8_$I_}z>ivh{xD3=^?||gN2!VoXFm4_F1W`dMgSvm|Ab~*IOiO8qW>WTh69@_| z_a}e1#$Ok$ghWX{Z7P^$v|!Obx)@3&RaPr*G3t+cEz_ILJ?GILE~^>zcUR#f%t$0U z{00q!98@=&WxG4Qo$;L zQ?HNBRtr7#?qB_aZbFy)Lq79tO;3w*y?BcU5acOzOGV!Uulz3Z3ZlR>@MzpqJHRCCAG@KCfno>ySeUrK%e;QT@FWeaN3^TlV+#;`%!YvVphgSQxTGvsu4KumL1lx+7Boe>Drn^jd%I}6IQri%Vq1R z{ud~l3l+mWt3>HWAF!L7EUEQYP1fRU!`!^Vyj`K0#T5XU+LHzgNfOasgc>;A0mo`k zd-pmx`~MQLC3&;rG@ReZ=f|D9jm<&N(_IVGS{QI0D@Y)0xiC3{9ObRLnWqMUc=V$H zWMNZwiT-dH4SCEH9L#nQ+`oyKe-WDv74zfUoOqH!)aQ}a-rJNEf$F_{xD5e!T6d47 zdKB6FCB$U7A=f4!1$7*fgSIWqz^xmqbI74NC1acKO9UEMfx_JA9pFc_AUB8x()>pI zO}Gj#4ET_z$x^*JLhg}*?=U&3>_Ej+x=073ysIu9l%GneZkX48Its)^v+yV255J%5 zlEmjT!W@3?f~u(sIUi#cM+}M@=LM!J7^i5{-$40?%i;7cbDLy^x9jk9Y8dG%uZys4 zoJ1mE^vY&z3o#HAixmf^hLVHOl@ZRYMS&Qj(hDc*PpwsCyGp-Y+FsV@z&R*`li@^h zz=9rvsLz%^^1kxk-)kG|xW~2?^|d7=o`FraWRhXesy;JH`#(cm2NM^p*(BAbAHx?E zZGbYXHZ5hxe-uhZZCZD*#5f`FKnR00V>r3nZ-f3-aWug}M5R>5Bw;5QL*0wY7hG#V zJxFH~)|ZSY@_-+>vbK8|0(`tkP1<8|-}+))O;^aNhEJ*W;cMXMB!v@9i)t%faqc^< zMcMG|zvuflI6HazG5qMX@z>)uaMQyypd@kz0sLB+4hwvi>-nu5j7;^`o*j7wR04qh zWN#E1t5rXkDwDVu+Kbf>9P9W{&Viuxq%1C=fk&xOA!c+7zrlL1-~AlHK#r}7mTM%2 z({%#}WPQz;GE=hAOwDg@>}zm8LjcuPa2d^z#tDS;HU{)48PzLC+|20FDWt$xc&55m@k6-~nHj(wQAD zua6%l#Rgt3>bv&)dA?j4Iz*p7?T6+-sKQeXWL&|Z&Q3)IlMTD(n`V=1`Npq~-LPKb z;ArnCi4T>6wVap&Z_f{OLY>xVmZrn{OAHyK9Iw?Hyj$UwRd-Ow7GHBL1RMBcEKu=M zTVasvk_x~!K>?k4ndW7cUoSk!$Y!^HB3>}W+sInU(-6GZEUFz^!NiQ}JYJX6C9F&Z zg;-2Vvgac4yG2N*h!!xV3ezO~WU-pln^XGmG$L{!rvlDl1cuR~LlEH3&Q}W6awFW7 z#By zCFUe2R$Tk9A?M=lPztzne*=QHYR8wvvz6x@U-)i^@{-5-alZ za)KIgYMadnWkq5}001RBN-O60?crNnpgXp~+An#zat?LCv%nHdqMCLrWTx?fnyG9O zbfnG|irt%p&22)`DbDA`)gxla6L|7Wny#6&c-Xkf>Gd*&u2eruEPSm7G--wU%PhwQ zrE~@wXX;X^>&_LyVWAmf?~nM(?$~HNImx)=^H0IY_U$vE$9JgmsGVhXsL+M+P3Aa} z?zoSw^|C)MSxG0!};(5F1iYeCL}mj4Wr%6UroW>N!8ZEtm-+eI$=0I9AalXNdc z>j0gjS4~i+ZAZsi#i_L@am3_WB{sErVvU~$shF~OgC!5tdG6W}-3+4nOW@7;9;snV zuuXH_nlOh#`=xx#rG7q#9q~penZ4*ANmijcn-9$Y6}GLR%>-lUXXTSpUc6OV5;etY z%pqDnI(fXE3zzH%EwW#z@}*fjbmm(lzlSM0JT@zBTmL-KI__yWJntAD?bzI zcA!pLQaTLxS$O-;GaQ!k^_YO~y_y)_n=xhV5J?lO_G2&GGd~ASo{Q&{Fmp+FBW+ku zKd&T-mAnB^CPGM^CJc%DUo^$fZ4NNpI4B6a(hd@liTfH5^jkHN$Mc{vgT6A!_lUi^ zcWKuCME8G?Iz@gjw)0R^)Nk9qY;L6yS66qt?9z9^@wq4gL&(Uk@e~v`#zAKA+haDL zDu#w-EGQ1VDR;JywV}#Ooz&8?@?&U=Liw z6q%XXL43m!=-7W0uRX$mx_cg;niEQ!CoU;8YnNEXqu``CM@gsHjt)T@DDGnLU*Bv~ zjs;s`gBzc~c`LJ7YP~QKFk0;*gnxxuV0xH=&New|BCJZn8YC8Y-7_D%{>)ftpxJeb z2Z7{I2WXCvnYcQXS`IgS;!wTQsZPCom+LcYn=5s1xjhom>f-HER~rn;j}U%Ee;37H zTv#eXJ$R-TKSW_%b0Y1|jzy%v{V3Sxl#V3qoL^Hj7)6dc!l(XDHZ7q8vlhrMYawy-u_ zZyaf=FrqeCHD~XBJJA#Riuj86Z}%!RDaZO4FW}X0%dJVV9gMtg7v)NoqfBlaqYv!% zKd&EvdyVDzV!GMjkPjb=1%GCsc{vcl4&7C=b~2+aDz*)6HEuuv{i_Iz>Pf}WZEb}H z-dw@2l{@r;5rWrKB6NNHb}03Gcu*4JFqn_WXAVR#|B7VPk9bQ`2Ca#us~IgB}FVgAkG>^;ys`mcPT3%7eVC%!$I3$i8#<^)2B!FcXwPYr={ z;Z~>jhAbxV1LlQPNLFOSpI!^TjFcedNWUAPI^b7V)BQFWbEd^m!eg=&X3OV}h|hCl zSW#vmzWiBzLf0oA>SOA>J&SazeZ^fh(t2Q5t3P*haI68&rRLUu+gFyGTa&j`H{A*q zWQ$kIv`;g$Y>R7c6GVTeCNYd@(5H;qjLbK*2>^c>v@CS@ zZW&w-KPwWnGuPT5V@PG%ZeG_`2IDbbCa@RhV6B>`z1gU~Q1$*w!Z-9}J=Jvl?07Gk z(4L7na1Yyy7Znn447JxbWRShaTVv=Y^uGQh1o_dAW%GKqAGZCFFET1QtERsW*RQmp z`%&$*(bAq|bWlY_-7FjYk|xN+XX(C(6Zm&umJirl z5F5yg!gI8>=xd^RtjQU4#8zSs53>Fh#e5zqfSyA#Dq!HTh0A@_`jI&2#^kXW%w}Ip zh8;I%qCr3ac;F4hzvj{0wsJ7<^QU~>W3=vet9I45@HN^w`>tiFM5NFVbVW8Oy`RV~ z+p^rL;Ad}m=xm^ZrXK){LZNbXOz-W9JDP|KmieXD-B+FESr6_RT&}mP-7m3 zH=mW9y`IEm6sRJgB14_mM1!T5B?mZ0Jn~B(HR2#J;c3x5W`SwF)KO$%w8J`~qS&u4 z+oGwW!SRxN_Kq{mJF7#d3o`5yzyHT0jI;^?p38G3V>4Dgnz=_HEF-McjE^XacQ{sG zWw!PZ43dBmLt>svfONh)f>ivAAinl*`;#Q5zA#XL==_=#?M;8u@Qm*{7X3m~8%8SE zK(xzXaG`iYcL#ZX4u}P^c%V|I;%!pC!aO1D3=iHTb#b1SEm4O-Om0CZtD01r942-W zAO1JTOymw^2}9$a*u7Rsl#Rlx72nb25A}zHlpTMMwVXfieea}a+whpdu~BNxyQ7z+r{^NQrs|d5!YtN5>>m(1>bhf zvMwK(Pe`wI=C|4nNT$R=Rt~=?Lg&Wjt5%oN{eZB<#)7YQBLVL_i z3ao9WKz$}Dfo(t;f>WQn*j(oYhxJ?5<5bV{Lypt4X_C?@L^Vp|D3mjKRrWxN4{1^T z?BQ+6ewmm+2@{A4E2G0n3yVc5fL;zTZj}HQWAN@3ZJXQlYxSz-(x;=ld8Ogiu^n~a z1%=U@MJX!OHhtWNaqLLsN61PMz|Auuajtv?eI=cbq?&6LC9U3Lvw{U0LQc_VAs<_C zzX$V`VX2ZkJwtZM)1yM zVeTdukp_gc^H`ccbi1jkVbx}trliK#gkE!DING-WaW*Dq7R7zB(Zis$yNRb=)#E-qHYET`Ws6(@E zfK^t#`If^2EF3xi=^J@NcyA%~8qiiBQ0mvqHdhha$Ob2xAOf(2C zdk(gBjNn5v2j#Yhi3M1AsT&~Fk(ZLdJgGYTm~1q_gIWgz1h&|r0VkKssFs!y!|Y^K zGLfW}xkm&I?oN1i4~iwm3GI>3rlg@`SX!NVy;6QF>^F)sx?~WtVNB|_0$B}QhgQ}Y zEs?wEX(}aGYVf7NT=V0nWr^JMs1?Z)Jmd8t8@?mYp?ld;NQ;^Ho|im3Ae|1t8~y#d z;^XAafh3|!vRcO%r~`jRFgfoqJQMUh!{3&#Wg^4Z#V%s`bc5NNF1kFlFSpj> z5anoRxo6zds3}{Qp<5fj*;t*Qv?pk$3pYC)`0#grY@GIF(-Z#) z4Q-qClYW*qt;V=1>l)cR>(<%Xb96GvD6g(f9Nw}GphfWPLTiOXD&b?2Y|VmppEVgy zlIT0KQd=!sgWZim-oy9P5a_kZ4cQ>=MfTC|;|%746X@hyCAw}dF?Q-UH{DWWDBO9z>+5e%_=dgG5^ zUiH`Wop5oH_9=Wwr|e0xo}%pX{=CzhL22#-hcxL=uLdKo763YpfKs|pA$8VjCgH=vUAkzxt z?!>G);WA}DX-z<433(t`)uvFRYn7Sg2pj21N94d{FUXV$gZ8rCV7q)@BO$s=+xtyMl$hEoO~`=SwNGWt-&7RZ3>27jeAC15>TnEzvt)cMn^l`VlS z$L~~IiYrJahm8{TpU%&3m`Jpd#!KY%(GkVUJ=N#ur+xuywsaxVAd|4&6yK4`_|UnA zZ4*05t`+~9klw`iuA3O8HJ&$iaA#5DtXq|UW#(FmaR@I?d;o(kd%}bl{ujQYtbuYE zlV%MCy-A2Sd_@ql7x`}|>ED$qx3NmL@KjtJEnJJ9|G~hLHVAmuEc9_Fy6kX|M>mz5 zD}|Rg?4ahZ?)gIeGvax|Ugr-bb^baH??~mz0FLTUnFx@|vTIK$=;t+iS#mb118qed z1cAggv5o#%;O+WBJ~#Szq>L_-m}2}oYmV!$r{5d5$?M=;ZgC?+VcI`HRzQp&MukQ>hOB2Nnss%yDh&e z1;B=A{JR4EhOjAuzr5KBTVVULjXgnbUIR`x*~R<|Gq)7Xe)VKQWdtpCFhEl%l zITR!gAWGQ`s`sF|M3q@b5cj)d9h`E?g+Rm%Bx&T+F_z9%r{06Jrrdw8j3yXHNL`5* z)R8b9Z;zgcWYXtE3S`>OnP`JzIaFm)_2$08EQwqngphlcqWGPF`gT}_WjDZYb-J+D z!miP{c-BY%KOYW0&0w^!8AyBDLXd&3@s(+M^ z84iHPS>bW8x~&x}*!fkZfS(@iMn2$)R-!QcGqQ#8+{GEKD;c%$(6tReaFm7Gg)BEntY9^$^YE?l88wsCa0p zN>fOU41F!)AQLLD#D#+-)+O$gD^2jr`@LTREDGpD8_zq!3g$i#g%aEX7Zz10g=}cQ z)Fr=AKU?`;%mKnJ@^8gFwIrcOpyDnz)CuSL2%*bsPDzS+GxM_+39Ksjn!I6evL@KZYHYi_e#J?^y`4dIHx@yZ+S9r`Oed>{^KFIROmPds&hqOvlByrDoXh7W*fF%1N4k3OBQUe-&e>ILQ4L%==r3O$D z+|#Acwe4N6Oy@5^avoF!;b@31dW{)_jXMo4Kj)yMOY6zyO;n8l=e>C+*x(WYTd{@f zeKjB)w#L7MA(qm3Y3>2hY5Z9CL(LY*@S_J)bt5&6@67i%!)4qMbBPYv3Uu0a_a`-E?|7wqT^ zbu}hAIlXoSHmuYeG3V2)-7kf3ZVypI6hCli1fht_DBTfS6Ar?8YABl>iWGa#IlzOX5cnt$f zzzY`F#MgYMLaTI%{3?Vh6PV2a7=trI83O1lW}(R_S)0-@I6}h%#}cpwyJ-25Jih(f zD`)K&I!3c`opOMw4yOZ(FJq9r7&#;(h#&*~#oB0m`a{6cg=|J2m&dNye(z{^op0<) zP8%l`W>1*$;|xTPCN*@9au4q(=z;l^vB1Pbkk_`L7@J-{I3C|I*4TB&JIhbOLn*jO z)4oOXl;ZcGyWF=!NMVlHMLkQ#jmZJEzuuNych0!W`*PtdD$V;VIh1OX#UG||239Q+smd02`KzJ_U}b~fe`>}GgWVi5sb zI)0k?VGVbGtqlUBE~SvKrgI-?zoduP-JA{I@Sd-#jc4*-fU}=USWA{z8zkwiYoB&bCXEjQ$b=*KEpdy< z(f^iMEMWn9asgrLEakj6O#jL4-jA1UaWMf(v8s)j)(>PQukhYmWGIvmv#=a_A+9rn zyNUA=^NxT+`Zhhv2cZM8-716d$C6zTyzqhKY*pFksyrD>I&Yv%4Ea*(4V*!C)vWu( zjfzMILsz$8OPlffzvX4~%yALFb~LiULO+!rVdyS7&+6@IHTZ@lVhrNf#zIi2^3eX! zbVXB?Lwq`WM|#44b%hN6)xEOtcb~0aP4uz(+d(!DyC>SEDdPCK!j1cLyBom+I`6zZ zCOQq~i26#Ppd0*>UVQ8UMQZY%J0ElU3;ZOPPsx@EwX-W z8dZ#jF(6xcW9ICUd&d4J9_GS7zf&sS=UBHmZg0bsY(SaOG!54aT86Lif!$^kdm6MQ z%E=5;nVucA(L&Gxn%CzQ1-CpF>g=cy!)=i>YTz(INHIGn;bazN&BiZGSXDy*>|jLs zQl#&96Ys|XdSi)|RnukW$H>mFCVt>;VjIIA>8ShvBmwQilLyOFdXeoGaqA0)0oZsX z-}r*szw_kZAmHae4WNVsQGvYW9W&oKeD>3cNGQ394@GLGTIcjw`^((-|4Hf4W%Qu zkHn=+f306zdwbsHu0-!b$*h6!-z(od`2N8d@cK^-qtCSAQ0AJiYqyo$$3+?zuDp@06fQ$S7 z>nBykV1_lPXhoKM`@ZLa4#_u!x5g%@;!3_nfL$0h@Lwt&LpOs=KEFKl8mOl2?$i#| zE-7_|+eV7mksv~m=wu>(`k$P6GWoLNPmDi z;o8mtIS&sd?c6|Q?A))IL$8n6=Pnh*TQs;__8s6%o}CeNJyJzUpq}^TJnSpG3k3gq zYikr=51;u?p@OKHi~5s9KoG6)6LH@<8kDrR92b-`1dV7h!&w5 zY6}E>%n)f(e};ffqIKBD_=V~xSRJioVkrWOya@Xz6U6!_sW$glOg-g!e^}%jW&DRf zrm(`<4=y~`>4Sk~kJFw0o_;RRN7P9%>1j~tK0%98o|VUwKZ@}T&gPYF&p|!Is9dXD zyJoaxvzz9-=FtxHlSw#!qL?5J;h2K8qXWHlP&x>y)XA&9`zcDRz?l7<>Q>3pGX z7(dG8xdmX<{8xygb+dK`mO z4T1;g$9XfJCb18%T0U*3!eC?Vv z^yRC`ZrE<3qc@TW-qS@l&Mll|wQ@>_6n0@r5@rrKmpnMcnc=&JYCBdw-lo&rT*Mo5y zAf2*Y4y;L#qkL}XzJ_4ge8qL%L8h%$^ z;_z)@rLy$9eYPtWB2=A{^^NRN)Tf<@cxV(7b5Bk!0-eK#Xel_`OP)nl!JWK-{p80{ z8Um{NX|(MTSs;Ii66sd?fiFEDd#O2?Y&;6s>WeA?&oeai&uv5 z{2C$SnNtKw`msq~1@nO1K1>9-rJ#!5IbateXbO%LWi@=QEO)`r$u5(^i;OBiuzPN* zNinZVQ%*PG9gmr{O4W0~IBxkod2U8T=4NYFXctFWBh)f^cw1;vo@R{vv2hPIm(&v) zF_s|?3=?CpZVXJcnm11>3>G)}ukM}6a^3z{P`Ln~hd~AuWx`Fb*zL+Afjal@V5Q8v zx(?FVm1%TIROiAV!3&L@qOcGegzSiN*otB!E2#VZ@#A}k$YWQI700!nYt`>-Ah8Hy zAR*w--V(23Q7ahyMMpev(uXgovXwJ@Fj))Z&G)QOIEL0@c-SqfYQ>~hsU%Eak*I~e=HSE6kTyna7imdI{=K)>qxjuQ(*|6wE$9DK>h|?gsQ@lq)A*p{2eH#!DK)?xr8yk9a z=bY?O?k+iD6WiRQ$dsW{e1dp1?%<~x^d4A{nD6~{=mC7BtxxGS@V1-{2Vl(W2U~iC zm*^L#ndOU&QNY7Vnd{GYgB=V);I9!$%oWpLdgC%F1Knn~&MD?iiNoak7Kfz>X9P87ta&)d64sD* zT`I2R4mzf3ZZme7uCs7oAkWQada#t~zw&AqW7l=3tukeaf`bq=W)$>Ran-o*Nnv;i z`-?9mGAgqW@oHfBgr$g)7WEa6L3gh3v`t1~1HG;dZS!e=@G>l^zn~>WL8Jah60y*8 zo9bHyhV91e+YyUYJGgMBu%diF;FT(XCu0XtPmUPF!k4FO3xJ*4R5e z&XJ;49Y=%l8($cSoZ>CXA~&?vUrSsH9x2}}X^OL(A&$&@xe|{+-|<*nfC>eC=*)4d zOSny~T!%8GC{PVi?J7{+sU6SnX4EC?D> z2Y&#dC;v(1fpecY6dQ{|#E8uPK3=+#7&nwCs;+<+^YTXJ2|_QQrYMG5snOgR9_JNk z;INPd86g?t7tKfAH6|Mq90lQE*6L~e!UgrC(pblQnC%zXDw#B$bFRU8mp4wy3u!4j z21G&>8chI|^n$%0K0QQ{%UBLbS+Hp7aQZ*c@f_GPRTieYOO3Fi>!{7 zRS6)(n19iwKSZ{WR|L<+^a|O=IJw@y(%XwYs7PxR z(4}gQhXxw%*btqE!?efc;sjuXSjbAtvVaD?eham$=C$5}|0N~>#&ihY7bgT8HMeqY z$Q1C@J;a+pXKiR%L_dVXWcGB`-2Y8ZD(@j=X5WEVy($FiaIlBbZP>aVh*g@ z*CB(JA}l@*;tqtALsK8fY=}mEf=y#!#g$xz1E#$0aokT(wX?LAE(Y0(ub6{cDBwiI zkW4pPtfmBflOy&i6CQDNXDhknJO*G%Fj&J8w})ivy_EAoH&OQcW{XZlS)KTf09_i0 zq{g+&bd^K$egCTJJq>42PI5IOlS7|=TfCc@r@>-cNg>|sZqUR+|6f8}mq~4EEBy8? zlxb(PQ~Ri>+z!F6vRgaU2ysr1d;xBEOOZ<+Fim*#)qgeIAthum;6M59972hSp%cQ-(e(7WYV!H558shg z%=O&s^V)koa`Gly;s+J1dq*v5qS}b-fZpYvUzOHR35Bi1njg4Q)fLvURVk1N_RRC^ z$yM{rWh#QbHl1qFNhLzNKRhlA*KIH7FTD7h$CXMAI6r0_q2I##62JBz`GMhk{=h2q zWg_^8-Gm)laaD!0{{BtP3utx#b=tUQ+OYTr#fFm>k@#Ak4%Kt)VvwNh;C|NtcHmf} ze#-X2UR}uKxd6!53F464-mIA<-tXY_nCVOSCE(sroi$8CBt2azgb1M7e&JcOJF|@F zVV0~;n3@ZSU#0BS-u=Uv{vh!FD$*BajGqE5L?Wc0!1rlTf+ zkt9Tqehyo)0yzuhg7KihLE89*db$?)Eg^aS6a`zG;FcE%#lc^`f+fp+5su6g74NC6 zse(2Ds+KvY98y%YD&~jtV2+lQ<6isq>T9oCE3n26*Ci5%X|X@=a||&;7e}8c~!%SEj1ogC8LYy#rpfIM4#~{kW3s)O#`!9SSG8t+SeF;rXsx$o681^mUSoHy&Ab?WizDS zqMMm137n{WNPIYluTZbqTwaI ztii$_rx%VrlGlgF<9bZj_QI$!q;m+F!~NWcEzVNd);De}mK#3)i!Ah|A5ch2F>1mU zjHi+l0kF;0t}OhoI!F746~|vvZwsrDS?z2(hE)`i%WMpE35Yl`h_NnaauZKl2@xTi z9l2|nQkV=Uc+9ZCW1{!Bf$TQ;^6d?$-R5t$qX2APnhz8 zpd>Qc!y$YIC5jCI#q5dDD>t-@CS7-kJu2LJO=EM0+%%Gv?+7ZKP@q?LZJRY5@^&t6 zH&S{+vuq*n%+$(EuRirdtc+@ttck8{2$NraQTp+FEcW}hDC+I(Tpt6^*1T5d4kNd$ zJdb1Cg=c}xyTVF7itUOdazY;ASKFTi9VM(HlDA`eypN$+5k75^lAbSG@<)Tjlk^m; z%0}$=i5A;ig6nU#Ch=sUKaPH;VYab(MnRpIf5VZXSZhBJaqP5Rz_i*lpXIYfR%Pn&HBDxAhv8&X?t0c2PRobxJIFU>=Q&jH`zT7xL&TgqT)<0Tnso_UM!3LNpk3$uD$culI|b3^>u;A} zIH*D~dK|RFf^U9g=4oLcysVAiw-RjjS!u8URhaA_zlLk|VZcDob<$A_DV&2M)?wO= z(5ZB$3a&@Zg<_K@nCjKOTCH1NYf?sKWwepdK)lLI+I6cH$fFP`_~cz$ATqi<+#mI~ zZ@o}C8Lee~_)WiUcvZNBlregkrf(=-MNv)k)Z!}N^l834ME6;Do)vf`T#h?Jf1TWpJ^%Mnx{KTIU!%De)aar| z=1BZM8F(U*z)p7~|g-CR%;h)qp@%$ICWeSAy}nJk#}Ild#e%C-0W7iLs8 zM=Ke4%~aXC{(PY5Y}irnQ6pGQS~Un>KW;+ZR}si%2$19LFsEQ2Isx{WZ78 z02YR() zEIySv<(T44B0p`Kh4`2%p5A6P-ulnpQ#ujFRoMY+KX9C6wkAR~@ob%K=cZq9g_}>` zEtIh^;azMFEO)j~2W{&8vngEYececlujHkRS|Mj}WTt9K>N^yJ zis&BxaiNf^2e{2fxyJy-56(F(h5FlodaoCMY_pIFaRAi|I3+!cIRn-eElYn17FY4u zO8Y8N;kM;QA2U=LAMJ_Z-|W_^^5I(S*s44g9E8-<*JmTsT5g<$)KQ#vn`77E$;7 zeJINcc?&%9B=n=AY>b_x4pR=~gf+wHP0*#>rFe*V-5WIMSRkzxxqpu-a%&$UpT}iz zLzmtgRvb3PVzY-pw@4+z+@<7Yb`%zoZr8i-8($yjna3v>&YZer{Od==5r5&~mS{mE zDhy-or(Y|_Cx^R{jVy1fj;uPs`;>S^Esqc#*-T#W79A?O+?TUh{2*w16;F~tp+TU-Q+gyM7O0BBHu#Gi?-ySq0b zKKtoR6qZD}9U?H?I0je6+okvIPg4NJd8~%-0iM`EHDm9mLrSUsLsb4`)S#}< z@czB}!bZcQNoDequ)v?HanongBNTLguB>JolZ!W+EQr3Pci5i3wX*6PAq^@OB=T=W zq)JfJmg9DywVVph(XWQPZU^e>lRqpHyuFeRK0?T`zhvzfCOCRXuRH|DK61mjR(th!e=3 zna~@)<*;f7JLgG}Aze?gU6d)Hx(hxZxn^RpvN^%@qQb%1bAHS$nQ)&!N5^X#p0?c@ zQN$;$rtoraWeqexlfMz{IzS04l#?3CZ8u0T%v=~C-MU-nOt6P@c5?uGc^F$9oQNiQ zea0oI^6ck<8+$)U)IKL0YU@K}j%^w9Uimn$kVPq&S;yXnEZ@^j?@*H}+)ecA7o`;w zNhhLC+UNskK6H&!06c(W|374oo3%0I&=jCB zjkJe^Z5Qb&l~-Mj_^LY;1gwDd2#}Tfyn#E_teFI;M!vZQCZ1DpYLm#~zr7MPW_dtr zAWeP@%qG`|^NrocXWOcd!7s=&4*ZQQ);t`#dS z_w3*S7fd~G>R)Q2$rY<}F^{IV;Mhg8D z`Nitmyl7f?3C>w|xYlJD;*i_zN*glE#bwYUr9`*GqA2oiKBrE$b%1 zBE~9QAtr*aZ9gOy>Jd`axQn^cGqvQ}os_If7o2%~pH-NJb@q`%oet+PnM7Px7OhN7 zm$ccyf!N2hytNqRm{GwN^PNix6(N=ogY|^*gjtV~GZJz2$Me_Zy_bi6jgoki_iG@t z|0h%JlA=BVq|vN}mOa|2l3rTv4Y=Mz2lHf8JZMM6WoJJuOQ$ht>FxU!XG5Cz7;S%b z(x=G&Xr>Jw3_%tORHLy)NDPaMMQqj#EgG{3;*&?=5t>3W>ek{emCJR!QKI8ZEVgIl>hDd>qh&(7tNNmnq6dJkP_lq{-H)u%zBsf62I5(-~QL% zhX-ZB9@$o|er7&_bu6+XG9w7uv*^w>X{P?W*gt}^#zY3l!tW|uO>NJicMVkI1ABtQq}}uf*ViBlQ|&jhOZhigbw^ zgq3#NowG?=%kP_{|EzQF{ z9&X0sXX81T^?}mrLt#61C=FQ^mYVW%!o0W~DS=z_2(}&*-9~v`l=qR0TwXhHWU%%Z zUqxqati|i$idm}rvs+r^!+4viFYHU=Z=XZgx&T(XL0tOH)?mHk0cbyIq70=_%_GY5 zr*N|L_IRT{-te5??E`WP^43c$a=*JsL9sKfhjhi1f9_aM*i;<|r)8*< zPlor2=eX^1XN_t{rp?JWpvu?*;gki5)>=U_~-oUxr$k zRXL_mj_I0~~fwLlB7G=^o(B9^R;~BM*MeSj$M2Th08s!-e z8gD7teYtD5?*aaASwbhM$SCK0qb!Z1`=_G#^+Hun<^W@_l?~~21)?^pxWslax?2#pZNX^R4fgZxvebg$XGV*HYN0unYn8!is>=VP(aasEIgsJr1gFWqmS5)rdx zEFYuB#(O229Q{h~*T|jI@UtjMt2~c!3FLV{{qwhrmRsR@UXY*%AMZFi25ET*KAKjn z)+maALJP7FN8Ee;sFRwgP7uC26brkOS7V78k!0!Bn;wlaQmqO1#!>cbdQoh44~}pb zDo|Bpg7`~x#i#D33cwem6I(40k^Y8Lc}ALhicf0sx&o;>r&l+nC~A456-GvKKh7N+ZI|NrSt3F4Z0Y-Tp+NG{qA8{zG)^_fgXc7 z7J`M^QI7$>tkDGhl^G(kfTxY}{bthPylSsCsHv1s?nPsdvv}*VNE6#CgV$jS)XsDj z<)aE^Ze(+Ga%Ev{3T19&Z(?c+I5{9NAa7!73OqatFHB`_XLM*WATc>KFbXeBWo~D5 zXfhx&F*qhp zWkh9TZ)9Z(K0XR_baG{3Z3=kWw7O+b9NMxrj9YMrAcMPGaCdii26qN`m*B3!L$Kf) zoZ#+GaCe6wAKB;Lv-dgo{rlC_$kW|VcdwQ;HC01FqNK_oZ02ABlyI;IF|aT)^8n=J zRUGV$?O9kERDc$)w#Lo?Rz_xKb_5CvQD>kr$jZT9%oqgZ0dRpV0g9%ecka6dGcz{= z1wayL4|IN4ngL8a0rEhQv6`nNkOe?({1;GiZ~-xx7`wcyf%XfSP z05zZ%KwVW_MHL{aqNuKSk=|h(ZTtDauHQkQd)r8fwmBd+?{+t280&oL5yIB1R`(L?HzGnvbJGFPJ zxwC`aUjYErmLQNL4-=ESyE~(Ws|$$H!P$b*(e|(W)GV!B0PYUXHh}kcXP_ND>bJQrf+j@}2Z92mL#-_d1<0CvV!_8_3WvAyZLA;=iy>H;wOOZNT*no<2LK_EcX)!F$^4Eg_6IsZG% z|E4bD@P0BqTW?=u_y6vgvAwH{*FS3WpT{{o{~c@_dk1%W@BfFw+{)g}{7>!8TpgLz?X8?#fzo3CWBx8e z_*-TH1Ob=cZ|7~b&XJza8pQit1q6z$yEwzG! zvz@W+f9b4TB&Em zFtBiO{Fm;1EKP0ffi5lpcJ{wi!1qG@SH|zv|C29(NlZ*kT1|oO|BlUHcH;J?4rW&N z764WbPJpqqv#}=v^ZP8Ya&Q2=S>6ZR4CwLKumG4C?Hxex6o8{E$QNMl;EeF6p`085 zCgDG#zYrHQfJx$S#13GR`5SQqm=ynwKCuCqRR4{*xd2SYf1^(TCX>I>JJ0mr=>5D* zrtb^$w>1m%dr-4~Ko$TK@E`DZum3;TA|ehR-VE%ltN;eqPwyRPd%uua zSXjAy|ATA#uN(ER{e7RY|HXf5gslXP{ zl1YGC%A9Qbh%1)2MG6!w3TzI@qAYTdlj70$O9{09sznikW&66`l3Y1)W2P*$ZR{uS z_YqlKcqCVYQ9US2eleh|gOuh_CbvMlgnc@zHKCOVpgwXW%H7ua7{faLhIW-os#n&! znh)(>Jj>FA>1>Pcv51P_{&8`#9US!bh#41c+#|e1J7ZK3hCLz!I zthEi1+QAfcSa;{Kb3O5buxVGiXQ@<2 zO*8_D$tDuf$Dfq8FD~fqK94{}mvqJcUUr}r-QaRrwmoYZ#?X|eumd1mE#nW25=Ttk zZ~gY-W$~W0sKKzV4CSqFuYc3Fh)h1*0XQ)cU_=~Rbq!RqDmSeq!H<%ogP-Dv42-I` z28qkd1fjj2sog6bXdT5XX*XmxV+pRikbGxGgM1^c>8(0q532>A-xR@x3cL!8ye!r~ zU7?H4pq8C=#&rGIYKohjyVU7l56oN_xyP&%SZ8_Oxxa&yUrW8~&XrU8 zybHlCxb%4J7dC3`l1RF575s-=#o2vkp`cFo<2>vyy?1zUo^y-q&?9=b`#mJ2%AE&6 zGT}T_zK92X5ytp8>Hf};YvV~cP}C!8=gZh_C9E3VZ=v*|8I~?XQl2p7j?1?@Z;U_m zK#?55z%y9Jdc%FzSQmYAThnjEqv5y;cm+Ha9ruwm&eig4)|X)M-fzpA7h zv|g`NLK+&3qby8PJxF_62t^3_qHh=MUx^Dd#hmI1a_Ia z#m*$VBVM?<0$syLqBOKEB{}K94xK;y+}|E(xa>6lydj@Ue&n$<4vI5TzJh7NAV?u3 zn&p(q9#;+ST1(vCrkVJ;RcG(0sKc01GBR$WxRhxvPPgo{mGcGRo(ZAc{MV1@s5R^# zz{M!9%A{00xApiP@bDvdy@A{jbG7FeQmd;YPvt>F=~;xGg{dSpLKv1A$(jUJYoKOB z^C0L+Y#)nTz{cb@E{7M|fUoe`P|H5%BbdDk&!h3-P^3T%aQTv-vPZVZtpn zdI5P@k>d8VEjQPF;Uc!dFFDXiqY1S@;qTB~I*DIt-=Px>Jc$Voq_N~we+r+Ye&81b zaTedCD)9)QMz<}Lqd`q^a6s@IplC2_Kc?hDU}6le8uLTyTb6V>iHnTAd8tsU>VQt2)AUxbrXA>kvkekSphnOS8PlLzb+IR?A63@lI?q&*z4@8XaB_uP;jlt`A|_(IPfI1 zwId<+M)4kVDn4k!CGmWTGOemb4FkRLH=HAK@W#N$2l362QRrmcn4j(+BcaZYM*%-f z*2_>0$E3LQ&b&TFR{=gs^Vc`T?f2DBX9Ql~^ z?fVze=}y-a=*`<}*Dr7^VQ7vYsqV?HJ***HL$Rp!7Ji!xt+!YF)R*)Pk2oL5gh;!a ziaSl!s0LNX(a#X~2a@YK%f!0MF8e&dLzKPH8R4$M1l05B?8ko)h42NVX9+fa12p8h z(aL11S(7j{BB_YH5HL`*71Zx|3M*J;1yY6+LDS_!7!f%2%V zYBq1NE%J$d<|g7MRrj398s$MByfq~@lNv7dva5KmJ0q9mDEL-)6R8+qMOpdZ68K{b zvsjj7G7Q{jtQsj!v36a$xFz^P=NFL}1?~B>P-sNu>v}O+sH8T&A_rHUJVSV%yli8( z#eT4bOTn}iI@xeTa@tZ-bMhIkvLl)OL>g~J*vgM;_#{1-9U=lXDy2(r{6g8q2A(am znqd9)#+5nni7@#nSpzq205Gz8#~&ge*$RbYQ01?IomR9`8lDnIJb z*K<6y7#(cj3K*@^!QCu`a{qey(OTXLO?OL zHSTK!^Im&~=Zo?nhdPSCH4MkiJR-l2zD}{cxOuNN;f|B+PQR!6=`OQaAkk5b>ryEX zmmc&&j+=i?JmLO0eiXQ+6#}W)mX8G(NB#8>ZSVfc-k3;$t&Adc$P5%t91_<&13I+|d z{8U|LjBeT9&InmB_n~3=VWy^mK()zxub?iv2I?!An`tdk5Gw=K67^k0FOkjuT>j$g#|K8$YoOvnY*;Yurt47e>x(Z~Gk?YcR6 ziidsqd=vE(n%=W;-FhS{t13&7q}wV} z(JB4c7i@K^IuK&U#-uR#ihp;anRhB==+5b|T+G zw3^exdhV7i{RSFhZ1Hn+6oi6^4&btGRq*l9ybzf+#Hmpq*|Xp?KPeHDVXj_;dK4m< zCfNUkX5ddP9Ot}f0z0;V+W9=Qk+&tP-63CI?eEB2Y3r?v=i}(v9*YqPfxP9Qj*_>H^fZiUDtBnqC4IY$I_G&r zm@Iqp!Ywc9@#w>(H)0OJ4HD;IQQGGa@V2RnoW+=L26X4+HxmnroZ2K;dtwg^#IWgb zbqA8)Tm}=|_N8E>Vh;3}DAA-u)0^>gndOET$_9SX#;!RA+q)<4l5C3k-nm>;bwLsf zYnU`Y*5TBA%$MBEu~gCWB>xP;a{~i3Ugk*@+wk!tD+pliPB2}NOjzG&n=(fVw$xcp zK@GILD5Nhvb$>FZ#hLvia^J0;`Vi;sjD0|Cvgm1 z<_u}y>laqg(mj+5_3J2?WEc7Mf;&c(@qgCosPG%oSfj0Pl5De$FC5FQXTO;u57`jF z92#3RZ^(ulG-JN=8Y@Ze?M}(fxBoGKtxuw_Kp)Wbr{2{*8|(nC5*Ehp<^BuJij2wmTlFX zP1zwZ&_55&6)GYIX~2_=YzP&6f9{YOWT-lOuy1rORV#HRh-Kqx(S}&R&bH*bA?*9E z;O7ds@jk(CSMzCaZ0q^(qc&2?!Pbu~nmT4W;wy+3D8OdIQe>N@OQKqxHcCKJP@HpN z13?^`j|QHjt{veU38?T%$?|(dR>-CPAb36BLtyJn5Yn(b*y61*Lt!-92NY|QPfM9m1Sv={ z08MPhT<9+rgkSE8U%l3^R;nB}SQat-q&xh~T?Q$WB;{E{U%svZkXY_o!)Zs3OBIQC z6A^HWbUtGo2Gk;MDJZ_9hn)A8}DqdHTZZH1Iyim>zt`iI*A$5rx`Uu6JdcETMbuE~A z10Y_FQgYSv>0(Ztq%aJ5|5(?YR@!k#v*w2SPp>%hCxxXB)^Frv8lp(m9@gV$72?cK> z%6pBudN6!97LGf~$Oe@9L0GcU*L&`LIEJHdu?nGSjEx^QnI2Gv3|QZk#R2OU`mln+ zDo)BW6ZtuGIgE@%=Sx^-7Zqv?D6!9`Es8D`Kjfx8og7RGe8#xRg~F7KbkEjlEgKq9 z^}~%zlqxA)A>qDVut!gw!ctT!Tg5l)>IF0*5gOEVP>PVI;l?y+*oYKWkC@f9ByIq!SI^_Fw2&TEhv!gt)ACY{x7@av@uU?0>B@tK*?ym2D z4AN+{H;Ou~X>?>2JB-hsE&3t>wleUV>FkSpl<{TSlUGSBPXUT5vrLVU(iJ)M37(UD z8xy=MpSv*{j}tltts{7g#Px-3-QPLbpp+7<4F0oNR}ty7;iO+>n!X(7$f{M7f_gW` zF{%B^lTnrEz>EvplaCT z#sN$Ohh|;A_R2TWjLldDhVHTZz7M8mU>b`uc=pw&ze1J`glfK+^fA8~OEbxluhLRt zK70xm;|Ew;Yk#H8^n#~^6jMi~5X(}z3GlZ~qvT8Ok^ur~cmWSWtr1K^S2C3LRA?Z- znLnwW2U|x#_D;~DQQJOl%J;i)W}%CRIxebIW8{$mSmm^(-Eyj$`W^# zahK}tEd*#(y!P6E#A16H1;xnFA{>~YOv~N$NF~Q|`^Wi_XNdE7(40Hz!Zt;J6s*r9%a z!2um+7*h$f?2K2nods#an!Ek(|Mb8tVoE)Mfr~PMMUrb>M&Z8Fu4xv_9j60LrCeM~ z86hNHWl^;%lTKYN3lAguTGlW&8r*S5Vd!ny&hFj1fBG0&h_5|0lZSQ$b$q|VtL!a! zRZAe6e$FRfR>IHt1)^!-zFW=%dLaIDFnhb=fykUUl!)Nu3XkZy#;+o?20u~O>s5UN zNtiy7vMMT@xy_j+g;RF*ArwN+Ekcz7)YoA{B@+h{?eT80wUcw!#ZO3L&?T?%K!aM_U09zbm9?wyb>GiyO78ltGpz>yqB5uf^x+59oXEfqo#ck z0D*L-bO#cQbuIW|2# zmPm^S)YcyMY?4YD{Op!FWh7;1cX@km7v{dV?`&xnAmBCEnIS7I$#PL99Da=0AUV%0 z;N@c!}^Pdje>m&}48CdPpjc6_ zNN&N8Nz{$!q7ws5pCArCK-jMG07Q~*0~%`NrA1R3!v|(MD2g;&^g(3#%m|a59Y2+A z^LWLsl&m=Lr`|`#(Q$Z2rSb-s?oD88`1HgR*PDsIq(;?4+$91Iy%7r{>dV?)4jCqQ zG*sb4Faa5_uIVDRRtl~LHWT0#;cf+!DJNMvxF%Ci1e!Hy^Og5fwfi0kO_RceB&pi^ z!x}g~U`L$^66}JvA&J@BH7iM!3Ev5Z!jkY9Aunym4_(xwHyBSW)th($?(xXuw|bY3 z_NTk=+H;29dRJYIg=wIhJ#KXH*y>tddF9TG)eO#|6 zz}N{?rem=*)n?7icm6~u!h?f~Q*rpNq6`0?a0T};o2)*`MNdrNexTQST`4kX6+;LjD)1zNn2!wzLp%3 zpQ+%J#%2?)_b7I-+|9}r_vyz*VM{sI3qD_E3Qqey%RZ_=r>nceQu;LR#@S_my={)Y zKEp&}o|Ky3jebC-gRdTzjIy?=h@N858f7MQgn8>_&4XX)!@@8|X>=?AtOK2Cd{QI> z>8SN!N`hcS$aNB$Q#Nq9RT)oykvVlGh&d2euN&Hl4}VrfhW%vzU7&274Lq|fg46PP z6fH1cUXRJd_jb$fjwG#FDFT$sS5<^y%}O>qKASSkT|#D#hp_r#RIv`AWmQVLuv78GlY*MyqjSm=!AtF7u_PLy~CS3ipB zeK^W#5%tK#!xDk}LK1tQT$KF_d(|2YM&8l=YW*hVdU>wL2C`L6D)8BdnIK(L0s$G^kVqTYEHm@ z8)V+olUYgM>mpu9P1)j?Nj%zst>7xx{->2Jxua%E=@&#p2fZOmCL(ssEdoGu**wlh zohVpUKyBlu2d|G-OKjy@-{)z5H3N*oQ%>KtssdBuWr$kZt3vD~FD`RNom`VmmV@2^ ztAWOEox`T0Y0i8b z?oLDKi{e$zD(XFEYIi)=69XH!z##L7Sab*nC(M?3`N;j+RBV}SKPR0t22h+=Gl3Ck zEj@=IGzc8e{KVPqyV|d9h!t!KCl3`WeM74($kOrDqp8BtD)jhq>fd*90$FUf>z^QP zm47t}Uc4~$Z7n`dJUle{H|#9EIpxjw^~_58aP0BSx}7pKl2+18Ow{rx5+_kw{Nk`Z zoeGtOQ)1AEi%(uYlh+NJi9Hfq+a-Z6#lkFRVa^dGrF)I8>+uLW)F^)5=9=T%V%5$B zap?(=J9iHSIxGifK!VlR{fd~IrXvkV33hMMV$nGUL74EOH05@8m4FP62Cfp6WbTH8 z*2k@5$P0h?OQ#^mhFy}3=kKo{y@{HM%fWg^)p)uXzu#K1_+OM6Z~huj`8CW?S<-pr z1476%fRq@^)h^rzPd(+@>tHpRs%j1TR9Rk1;U2=*5YWJdA(8a3i3AdM1G3O}86xv8 zMES}mN3uPHl4~Qp5qpi6A7acAYH6EEJFpmLTLCI@tJ4xwQVXh;^O*#AZ>w&8JV!$|lCa~k{i=kM5P=sCZT-sCom}j|ba@JYb=_a+A!RdoCVH?cYs!RW% z)m&Xv{Y9fz{L^@28N=F8#1!4^rW&)^+LecJ70Y3rz}2UmDoa}78f-k`CWH=wE1xz8 z^dnx%fOw3g*YFCmT)o>=)*g#It&}EPY$ogQ?VL_l+oZGRGK85MmZItm7wG!|1iVuIf6pHflZK=#bQgjWc=Nc0cCM;xAoK)4h`f@BUXRqP-r$PF33VZ78j- z{f@o>>ZQeh{H!$K+SZ$vKpA{!rFYyHn;bf67<UvEYQ}?#_ z8u8_Ff)aBjZbF{B9PN~G9a1CMI!Ru*m#kE&pN~*y@aKNGXs=v|6 zR86HSjm{WXJ>{fb!{<~oad#*1QN*t_GYtV@>~xB_6}i6Q`73f`Pn+Yqeid4|5jm}l z!dZWc8ti=rADMeC*PI6$70BBZLu=+49OaPndbDaCksNM2hf6*GXi;%2onQB>Q_e;1 zZ{Of38?S|2eLoC!sG4pkFrVyakS@K%^`@ThymNdbZm;vh19-7iDRYrsC*)|ktBl+8 zHcRq4X!5S0$~{|7pjK+4FgIaJOHIr!-Zwui!@QhqC<#H!IK&ZdB?;C_v3*UHy!gQC z^mW193_?ve*BFsG+&=&Hm0txjYVvZhxh`A~bq%i>V^hDlAH)<2D5v>S?mmj#z`OeR z?B<1ABSTs+s`m?v=2jFZXyx4F?4(B?tK!1@+kU}4ouZ9$ZWC`o32rwRjuiQA?ui(q zcGfiWCy`#r0rYCt?AMv8uceq`fUrDTJ@Fnx8V4=lv0SSY;B)j2+7zrDdh zoF{chfdPCl+FiZp>X2>tjpBqN*-;>)Wh~dvVFFlNsK}2-FN~SK#T)4xVMI;#n?t#G7%1;LwqqFZ&2}@t%^epH zV1g)&k>0-e_k49zLdKKq^;Ayw4UV(AaH5V-l|?q=fi@F}3Wf}zIbglOE_tIVOh4;w zhZA>Nd-TI8A?%OtL$BQ1D@f5+bG5BB~sh8PN$2TRhh#v-=G*! z?ag(hE=!I}hbP+E7v;jA;Cr|oFhE%4usc}&o+<#l8Y{frfsj~7{Y7`PqVeb&ns79H za&pgyzOd0~!20d%YkV(XD8lFd^w|D&)cCc)8)+EkD;7U)nesF>=uBkr3QPUhJwA(9s_jaw!7;?h9{(4Y%J78U@G$>6Gzr>xTe04SNihM%|y@{a3X{E!whrz=HE5~e`Kx6 z=~zX=rwx+x3CY}VqPUH|l`sDG_<`wcAce7^8Dz;9?&J4!(9U7T>vu%vg_p<)&G-h6 z-9{+?4Dhg7^^pC1e>Z}`9kPB03mg^y{#<@fVH!+YHtKMM*TEw)IcSP2 zpfbN`gi6bvL+O%DLPaPjehb#&NxOZA2aqX4TUV> z$;R|*FAq1KGxpoXETdOa&4!s4moPdF>_y=!ST_o6fXg9DTWUZ6%6xLrt?NYO_ZuqA zLHk$aoUcy`%FXTggm5wK_}d!bH#kY*GbdQJl{t*EK93G%NinHkpQ%E>4WhVM&tRi- zl61)PY=?C4XSSL-iPSa+8nR--o-%Snn|9Vp|B|VG1Re6#UDMdQ2o6YPBEgyHmf6#K zw(H8Xov8rV6Q*P=$HSYZM~=8fbPhr?q0zP%|X$l9sQ>cJ{Pq_K)-+$sEoTL)^Yqi9E+=Xzj-u`f^cb>4$ zLVSkjptgN94SMP%LW)LUEG1*OvI&mA#9^m>s$)bEPD^|yIpMWFbGY0uATVP{#>wOf z>~6z=idVlmMVdu_N*`qW5&TpRjV_iDK4FqZwk4m}FEfIbyvC2G&*Ez`%@IG5|LDaw z8@R+RRag4Ac_m#uY+@yNMg9+m&4oN7#AS=2bdrtf)66lon5EXgOZWx)&!f$fR+up} zz;g!%xl3gLrZ%ndg}=+4sztydmr!PD`oBMS9*TKkOrjLd@0r(sho6d8jsEqUKXgxh zO`v1%9BO@)NI;8|JyYjX;{8CEAfch8RKu_ChlTLRBv7oq?-t=DI`(k;3$_awX$x1iM_BG&b`2QRHLT7)t~m)VM5-mW zl<%As%`Ctl@`s$GJhGQx^84ny7IJgyYGco1q)|2v+N`ZKB>g`7mR)|Q{#vUsrjR%V zIeHqQC491PvfbMOInc}3AZ6Jex75bt*8c3yho`y4Ci!z929XOqc=(>1eoT(PO#p3c zK0ox#I#8t$Ei;9}vV|IR-hm}`xzT7=r`^OrU zg<7&we*mu$FAI&Qku>n?VF-bf^SY^m>jvwEDYk_DDLFB6xi651g>V;NNGoHgWETqI zcZL4uc}Fmg=>W5N$9a_WZkI@DUuIt08?7yZ)iU%_L@4sjS`ud0Yk@hzgJm?o>J@E% zqBKAH0;h}9(PdjlC*)-`eQ;*I+*(6`CV${V3X~&@Ql$l7^bIc|*(ueanqCCa=R~TV zC!FaBT!dOg6xybBR!q6^wi_>@CR#`_6W@HYjJ&W$08O|utjY1WR~+iB4_Li?F<)A86_=Nx-?emu&rVDrJljDiPBP zJ=l?GhuEi#mBO+Hv$L>4W%5;{??8%4ycR7Fg5oP|@qxuWct2Rg(`BiBRUxgMzQBV} zi)nVv2M?u4vfna~BVCDQCoO#bZ%o)Y5-{adjx?DH-$7{pF>B~X{k~pV&!KEDJ)SDn zQ98>|6Bqk}_1m-G$k{VB_)R|dH4|FWHY%dD+0k6Xoui)q#Ovfyqp;@`B!X^;ZC!1Y z=@qVn9YVFo1w3Ac$WNch>1mZ4w(`JS6}b~UsJ)UyQvWQt_Re;Ba@hTX6_l`W%e7~H zYfD-j6T`8~ciuf=Mx02a+w^+6UGy0I#7@2#ADGm6KPD6g*^H4DdqUc=b&y{|vRe={ zqR-bmrvX<^OJF-NeN+gJ>xRWIc4{fC3qM9$q?8zOvZBFe2fnx>Sg|7|BOUQdtI}3% zv>~OkzcP@`FsMNspof&dNlGbn52i&N=(*E`OFwM+P-m!F=04;;gX;xF<@4E*Ft8I| z`K7<-6VS`|BcpEizLBMX?Pz4hRXzYqgR69`V3aD1uOK)$VWAbFa7LFZh~w?C1omJ# zSgaefXDOO*(kGb`700_^n??6e zZCnZ%#*j|ED$)Pf#@asjBjF@Tu9RMxdDBV#6o0)EWkMyLy|+R3TB`oQHnjjtyw`|| zfYnNF0nE8L*0bsc(+!iEYtrmDzEo$~=eRZ;N-79XGDP;eo6T-n)fVw8(E>j=&6pU> zte?#KXfkM00fC6l(z{F03LrwKiU`Vb^%Xb$_3fqStG)P;ZQNTMOGIBzCs3gJ4Y$zk z&0$^5OKxUhD!7eQGv$%m5B1e(fv^mD63dit(O#?!Z(p4&X9w+BR``i4!On_84U&wR zKn%m^InLKO=E;pVe&yjC{vrcH%6I@mPblKpzPM%^)jeORS6&;xI@Ms|>%r8@Nu(a$ zZ`Y3!18H>2+g)NCKPKNMs`s((rJeE57W6~SSDYhh>?J~!aVFkKpT1781|nLPFYj_L z=}WXNSC#1mgWu7o%&u<+4VV8u8ESzWO%^e=V>R%+SiQu1L04QDOA;&YFG}nH@TX13>O@s1datl^JtvUxn-d zV&$yNB&>ogqXn&WdK)@bCyy@5;(N7xf_`Opvy8@f6?%YKQArpKK1uky zQ+_ReP_z zchs{qznu{tt#RWQy3+ir|7oRrQ!YJRS?p%eJUU~)8#%<&eHa`dgY5owXKPTIiIrym@WOQLa>smhGQRZ_;uzOL8woDXv>cy5zS{ht zz{~EdMf~UaP|P-&k7f)qi35EPn>@SG*c}M`#Ayc?&`hWf_WeRn#njZM+R~UDEr?TRnt8 zYQ(T>)^j84%?rttzvY%uE|IZ%4{QIv_w}TBw;~`_E%|n$L?D-1X^dgB?eGpXbK3FT zr29M5rO&Yf0vR>YA)7rwiQK*Fc`KKj!LsWng9HZKSK{|5Iqk61)?`kzCS{)9lo}Lv zd8Rn5jPwE$m*U>C116+vks(1mv4HQJs8<2GCij8bOr%#WfwS`mstn2+t)>vt#t)B4 z$D}|q>BpT4C?0&Q@0Oh-<#s4t!mxcxc|iP$UpH@Bt5MRD`ha8ho%`s{Lw2az)Go3} zbW%G_f(xseAYdnl32Cp|?`zi+t)9atWheq(Y!xqm!C#25GboLsc3Q52)`wR2RKuST zzwQ}f@#g*p54Un@^`{W{K*qJ#)J~yOLI-d*Z#UeepzwkGbomT`Fe64%j1!bP{5`k^2Sb!)@7LL8ub6Pj6^HC*o^){cQNUB6SF{UR;~jDUAGb~i@DPm|!l z6BP$RJ#rk2D|W%>K$bQ(kDuhR)r&jIs99&?A3Z~n;C{cPd_FC~SL?BsDz`)WENnMp z=Wc`9_B9T3uZ4C|Q}1JB#-S2;xdyZp`3ipSfV$r12&o)v^lbU>IF-*WN(jxos04A;F(hc}?EVX99`nLI({nsI`?OIP(|y5RWPv)s z$v#n^2nGMgFGq7`ZmooGBTgs$7><*d!oZhvf;}RuSnCv>j|jKnupR-!w7?{`;pE;t z9$++q3yvLTQixb2dh--DOq((8r3l4L{sKCJJV_mjAD;V~qq9}=>+-onCTjD^Vgmvy zQouoES`=<3v22asXA&8+3?<>Ja>`Mo3^&0=z}F8%nBAyFA2idc?UAc1de3o%zg1Zq z)TBFKL_Ua4PfHSHZKFbLRWA$+|KVEh z_u|-uRNJdglM{?wy`G^ymPPvZ$G$qnNhYY28;KZ+BOp^@_hb+gZr<{bU9_qDdqVeh zZ~4eu^2c6m#{cBsmSo+b5+V&sA(G~o4!?3%5Skpn$mCF_hSkO<6*uoGUQXw*W~UjBkBhe{AdZj8{$)71`P z30a7X?n@aHfU2>sR^6IpAlen>YBLV`(~~FBy{7AzX!; zTtkDp0zT=%+)tf>S*jorLPu&56a+Mr)aKA8Iewf#y{fNqYowJT)H_b{9p1KP%UHh* z&`tDYtu-L;H3aRSP`&(~#v@0c`t5>N6zj*GNl(xLi>;U9O_2>jnQjl1#&27iM74!e zl#>z3#?lp~Tp#l^6}AIiE9PBu?6f4YDGzZJEDqX;*2${bj&1qZCKnS0I5nE_Z z=fuwlphoB$(2(jVzr}dDYOsi8Zb_v(m1FrsSycAR8zIb&m@oFrbioGUB1L_Bh>6A- zPxpdQ!H7ez?nsLD150tMkRKWT8Wq&sT$SOO4iX!Dpu1)^hy~p{iR@p7koc0 z=~OJnTO}>oA1mpz0hH1X;v<}6J{cJgsyf~)Het36V$xPZC0K2Y=|S=`;M&k4qR#tJ z`(fMcG2JRozfZSmyz3$`+?&PSe7yi35<9_0gBQa!oQ=9+l!J&A(cW3}$M?H(Z(+gw z+|^EQJMZ1q%x^Y-01v`SkV1EBN6rvnf+NL7h)8PP*fvYT$5?O|M+^PY*ph!HEQ}A;l!ja zYw{MchfQWjEL6MtdiCZBKWPVTSY$RcShJ@U*=}Y`wKdv9MhQw(FsGv`Yn=0ezp}wr z2BJc0c_bfmfP(H~4@CVXr}MAjh11%Af_PbruIFihDfeF%?NrXr8f@t`-3jdFd{*@4 zl23KUwAbH%EyKcyFN-X{#oAhX&K`7LO7O2GlXy(4SX$B0lh<->^OT#`1-(pbfX#|u zD_yB*Jv#9D5j>}4v0k=4Xjgc{&h3#v{45VtYoSeL5OcYbQ<#}5cQD0vc5KG6>AFaq_3DtG?3kKq7H-Yg+KPx$)MgXku#}nN^a+ zCvN6Bp9LrO)2l3RR69%f-RETqr$YKBBH2gTdgLR{JJU(7A>5tgm1NaxT=uX2s9K8t zJ6VtqY4ONu9;OX#hO}$y1M*G7eqWwm#ajKO(a%ReZ)Mc+wkPsDGa>BCL1}|CQ3cv4 zu&;)inWmP7{Gy)tlp*|S<7T7q>eJaKb!fzx$_Ti}wll`DEY?+^zt0B&(vc^2INx^t zL96B1EBX82fUgGoaOs8c`!pYnmW1G0eFvL#sf362oJP^l;X!nsb`J>BsyzENO~UT- zg%s^vIOk_@!tK&$qe9%M1hmkjp~ z0%Q6x{z6b`niX9WiZs0}Y{QC7q_%^T1DMAPwCC|Gj}3&ZuKx23ZFkbo9gcHY`Viw4 z@t3rr3QS5?QjNP279%i0!g^<9E>iq35`1~05~xphDYCLt_CS}+l{r&y`0Y5tP;Pr4 zecm10*`293E-#%KAFQu9d?Ph_FHG}B_VZ*8V`-ZRjJT&1+`loS`q*icSn=;mG*c4? zQn*P_I-s$x5x%Vqnrd9AH4GvII<(+FV=pqCEC`?Nt>4dfD#*w&6aEBni49s4?;5IF z)zCtRVz8v*d*4P2NTgm3I;q~3`^)OpWCahx@SN9f=FlSW>Ku>ew_MHLmS#zQ6T-1Sz(+YvOCnTcCzicwv@E?l{FR0S0B=&SdY&$8vl-m;v0TnG4QzX<@&iodj4%3)>b#4;U?lyg!+)9-xd<+4J^z-(VRNMh0@ibn@ zwEA*6*0d^$64ZrhrE|*qN(i*D7zEv(2{P|?+Z${b%`7mVl38~BBNuysqO%cnI1Q_0 z-wM59{$u{+B@f%H^y{oqK140Wn{w~)9M!j$%_0inU<`Us+nzgBxGL(%Fe_c5iHEnv z#);Q7jh~j3dHfctC$s9u!{n8TtD^`zg_&AGY&7%UQo9M?zR{tx4dqY=#p|tM&V3se z)TLcEWmQU}f#h)-Y6->Fh`_;9*Gz;F*P*~UB6wl^ zZ|2Br;V}tLMkl^x)Xw%xeXzw5#IJ0IaI?&y2u8#%egK~sA}up@;6Vro`wV+4N|44EIVAnGXt_MgCR_Oh zf{w;229_I)k1z^~u;#R8Lh>U-<6ym|qILlD)jm40wK0+z%D^sq!RNbysSA0Qy{){! zJ=}{n8jT+Wf;h$^!#5Iz(PNtNE2H!NG!!ANw32u*vpy6~Bs$S~@q36uIjPT25?p6s z>c>G5qwSeh@!@Qi3!%SjU1O%IJ5R5A=U49ts`(GV9qSey&l2Rczzk#B zd0F$CE3+HsgzNv4p4;wt9Niz2jQiGnlL59SS-!1oXL;XYAlzxX>#j-`E}VV;xGQTK zC-YM?tuQs=g#ISTAmGDr?wUXL4j*<9^Vt-Uj1S(!@};l%Qjp?#IUh5G@fydAN;VYl zslC}`J5jdsD}-78XY7NFqo%i3TafqfwZwc$0FX@1^se43qU@aYVn6M$V+H-!NiVzd znjbX%1 zD!UB(_1j5h6c4GW9))slVg?j} zZSZz68bkPKICeJm0n;?Rl#i;37&!R4w`1wnpfPO1(orp)iXT}mkwcPZ}f?(XjH?hXyEd-q{yU+$my z=FB(e2<;55dxQMmSGJ})V$oHtx!sODH{z_UK6DbN#52sJAIR-qSTq5j>RUlh=K+L# z-L3W5L)o%bLzu2e4A-M`X_=;W?p3)k#60eVn0Seqez2Xx7bYI#lz;pzt!kEcv@Gn_ z!Am{6v-C!wPDlH{aei@Gc#H##@SejwP36jr=stxZ)u-rgKo%RgV?|nt&PI`rtXS*P zungPvZ0WErE=9nQF^dpUK@y3ND)}wHwofT(tR_LgTqX2G7^!uGC#DOn>fnsKh@K!$w6iCBjlKhbgJ!k6mph()@jrjp=RH+ z#GAcJhu-}fCxFacIsU2-BG3hUzE6zT-6IVuzftNmU!J%?W@n+VF10akuK z=#ApID6qn0^3VVqcdu=~o5TS8v?@?@Y$7pn*8j1i1u&ihr<&v zgxsC7P0rAY#P|;rksz#(>W+pW0(DBSNEX=r>4A#b)*`v8s?e5f!A-o2V*~+QJRT%Q z)pl5C=`Z%RJ-X@_z-!E6`M%WiYo7b-(2;ZzB;8LLy`K~G=&T2krq#elr_%j*9tih! zUt@&i-GG-GI0S?5Ni|Wl6-IvMq%eof0cKD}brWDy96}Si4>s;;$AnD&}S(6_PKoWL`q95(G|?H42p* z9yB_0h^oiLAm-Ju?OUVp9AYYr1BE2S;HJ%0 z%BU11$>KQnZUuS)0@SEcySpDZy1j-I%l{e2HH}u_Bc`3MzSjo7F>z7Nx`^}}wewO9 z#+4;_HeIIT{+`h{I`vMsDscma^a%%eFwk8c%KoRcLPafGVyT^tTClxM#5l*sJY#Oj zC!}Wfog}zoz;(d^C+0QuDJi3=1+%uleJV$O4PaJc(Lj+(u*d4|3O?Cg{7azN%^ZY7G7f8V)gdGrZepQ6Uo6H<^B$AtAYIG_B2R}k1M{J<8?t72Y5)(|`A{;w(ek>cn{Tuam4^qCNAGih zM0DLI^MTaG-)jqYa$ZQ^UFEls3@8QJIF?GX{o(+-nw$i?r1#h$oK4Qop z!hMw<@?q%gez*jltAKJV}Em=O_wssObPl zdJf2>&bXBnF(Nmh`qwzpA(=`-HR0Yp$fW}8>^rHg`#q?{x4$Jg_L8mm!P6>HSohfo z8loEo=6VRoNj|yPeA=3Dhlu<>-k$=b^t1mGCpwl%9~Vi>%z&9<+DP8q1Pk?aZk8_& z;7%YV`EpB3C$`L1e*j(7eK!i!i#hwws{VLHo?98kUR}L3GEF~3DPli%Xkxyb{V?D1 z`ueR*+iHBQQX(~#07_1fPX;)QtT_wJ$GQyNnn4oqWOU<1Byx=bCe2C1g(H;DQv;}# zv|XahYX@eG?xyMkIj{pJC2i2EPcH5eGE@F0hg&yc2JglvxE`D3DRb*r>~steue)^8 zd(5dbr6~VqHmNx{_z0$)J7HFk56u351>q3pdjw>{IFdC_k!8I!zGBcmktF_Zu~K_U z%~HnVrekSRk?*6EyHKg5;h9P5tg)+s16dTFF%P)Y zR9<8eP{q&yF5+WtG#22cNSUv@faD}ECSB_^^tC_k)srH$Kt-3G`k-wK|93n)W?*yz zNm)Q+U-p#4Ne_g{lXH9-@G!yhBE9vxdS0YHYKbkM4i!`G{#qof;s9K1_|`;$jPavc zDM<6?l(oxwG5Pj?f0RJOzCXL-#hB)@+ujmUW2Z>wIp=8V$~Tu_^PTkm(38 zwBnGTyah)T>rCON{-o{Hm@vP$2M1=*UrV6+GVg$hK3rdH8DDQ&wDn_`0<9x|^)x2! z9JN-L&WJl@Aie6H;8b~yWCzha!gm@#?ZWL8zEJ0%WDVU2@oX16pLiv(Q$)TfM6U0A z8{XZxlO&cvA;W60Cqsth{;b|bvgjx0#GBICI65kgSnEPjym~_eOF!xctTg6z%H!>& z{gEn(9$ouJ&o+1cRFYZuU0KubC6>Rp*xiT1FMU$Rps<*4xkh2D7#z*_&h1|CsLM7SLtiT1!yQ&fvigIPZ-=elFXPI<8uHZKZrF7Sz8ay1loY$rL+-i?6y6i z4yVAiC|nvQWRh2Lpy*?1CzB{tLxa;0afzZa%=}AX`%TYlViBh0L^@^pA)E5qj{tCcTiDY*kpoC< zfXYU5(+>jQ`aYIyjs@z@;eLAhxN{%vl|Vm2XqftT;9YP1#zY@Ghw~+AXI`v-yZS72 zPUzuPu=1;B`|#XF%e6Wi#xQ}H7o^Spt^qW|qJr;BC@Z{(NGB*i#6vm|TKasG#b zW~KikL+a!$FnN?s2x)|3B=~|)YAp7RPR4n#bzS!sh)9=9_)mOebbR}*=aF?6ZyAh! zFPR^6*Wn0VFcn0#JUnU`zY5)!)H~6ctM(y8zd#oc=E5znMDFv6Qnlfe?fRd2zgrs9 z)BTg?Th=E5zE8CxaPm8WINDIno1a&+R*@{zn1b7{xSV2{sE26Skr0|j-kE!z-7N}V zj1J0ygF88rXW^m~Q7`EvryDC`;6z*$hwN!T$w$-FS#r$Zvw2y{lyb+z80Al1-nq`E0f_)NAk_1_` ziBVG6h~x{xzM#p6(_eNC#hf%Et^g2GGCz~OXwAL4a|z%rvuB{vHEWZao4uQ5qgQ%{ zL(i_HcVNeXH}x{32?U)Y3?ls~S!>)WT$>{5fUgD3JObqQG-8betPsTj%`)-g>Mkg;Sq(YV_jqEb|0U(b{R)i=r0k8+P`*@8l7J z6RRvuIp9EU>O?7`DbC29b66UNOyI%yV@$~pBxg~C+MWB{cM3_OiB{16=^JyUJMUxr zxv9!k1arX;p92rgR}%+$Z$${qYG0fBYMhDfUw@M_CfB8~{322Cd&6pWs`d8LM23%< zYjM%JelN@BCl&(mQnZcyOVNBRONO?uZ?WOm;6q9KE^nAZ<*p4o(#nP&_$JbOf2h_Q zNF!*y?>iN%a<@?aZfFD1Kzj2 z5OyH6c$8+Unen4;wX9Z(kaP^+3vBBxq(^uYdA?8-=4`%{sw&;ws(EYC*o_%-wK|}U z%(?9j8ad}1AI(A7saB~5SBSG)i_Y6P;NNkTdDLf@IB;h_VC`>m zxtPvIf(Xc!G$piAvxJ{v6~rW)kGHEhNbhCFn>jjo5-N=%ZxqsehB_>^UW%o@3Mfuo z4LT&bt5p)#N=}w{h|i|#R%g^dOloF%c5=%GX7MN zxSHQx&023Wn7nV+%(#vuk>S3Q1Y;RGyOBKs({v`F*r~2KWyXGCsVw|u-&0YANZzuWnXf+px76p|WzciU~o`FcALYjz~;!D5Wy}Qo1 zjgN{R4Px2&4kaBd7?cF!2+*jPf9VxEpH879hPNM^#&KTwPjsDAcQo>}pW^Tjxxc9O z3JG;5>!a58D7bWs6x-kNcf-q|kqxVgU_VSb`$DjLaibk8^4&u5^ay1%h-H?Ht$k^# z5RYx#>T6C?QfBgrws>%rW0=R{AK}NnbE8Vcyx0r$YB$7_@pM0a2J9$r=}VhuSbGjp zh#Y@m-LEvp)g=7$u`p0tyJQ$Be6Zlj4-7)B8Bl{nq{U&C-1$fHIc`pUYVj9iYUkHe zfV;jJ<=#%NSygap1uX8v<>b_0%El&%SE=4ybNisSr747nqWXoO3?iaKUWvN-6__gq zCK;K!A4)+aJ{#0O*{laMlkdDtn55nk5XDN$C|n`yT&h^{=fDpV$!S9<+N*3^;5!HX zhMlBfZbBP|ycfx>JLrc`ry(O8IL^bw2>phY#GVaw)JHqEQb(1>ucpr$hO~LngpMZw zwvS>hJ`H$Rn6MRu_TYv!harW~oF4)3=m5ftwkyT3wae{0BLS2#RU%LB5?$ArM&>z@ z_Lb=<@G3F3Je>G;VX`ANXALE?^;20@sH z9!0ZqlP{oK@RSj=loRfd87HNbhE24mKySN{R*y zxBUzkj1t}l*MwwakaUhJWb0gb;Q!;@2uMO?8@gVDh(-_vAFxQL<*J?Ql|4QFDtuq9 zY|r*fs+nD_aN|#H!J1EHI6awgL$XSHzoc}oMlOL#Mbm;3lloZMY@t{viTUn3*;s5- z9z421vjpeRNy4d_`kOyqql{UdqVYFO?y<;E0<%{bv`{uawKwR2R_v0?BnwRey%73O z+J677XZh+-u#}9!&~O2{**4+Rm!LCcZZ7`_m*wmfLHG$iD$&E@JnYI>2T4jLtNGaJ zcc~1qeqM~+6s&zmTm*ZXzw*;>vr2`}+R7eCf)?-1)ENUC3iCxP-WeTq?t__ch=`S> z0HXoDj3c8vGPr0Sjw_JndaTX4(W!U2m946P6Ch+Lrq(Lf10zeHajkx;^_pK*=fRrh zMdl?@FW-KMm&i8Li*L+iwF34!&iA{kG=U(JR-O9^AM?_u8ALs0@308vT5Yx-vde&; zl7=z2B{i-p{?N(;40=;q_1*-^+oid8mPFv@5U?NOHDONi8Qi?4?lR)R@$=l`6($%G z(pv94SIKeBBQ=EH)u72|DGXY+s|uxS-xB)*_sLzm)u{DKcs;%9hL{z}3kEINJ{P5( zO%=MjN+;XYSlMX?=JC)C6kxmMdh6YbCY987EuB!Fe3f^erigXoj`aC`0c7^y{ppc0 zz#p}&c)MK48qse$cSZ~KRNMGGB9V=;$k>_D!}4DC9|F93#gC)wyK{~n6J!N;R8t@Q z?R0)hmMPMXK*(h!$|7%!u?8Td1-%*U+LyRs= z7$xAgb=$UW+qP}nw)?hi+qR9{wr!hV&p(TqWHPguWu=ytN-Fi9Jm-ee!~Cyf8uJZO z3*j=RrHua!une+9mjR}DYEokoCXMwfu^?4x{^|<}MAIgIbIx`8e@%}GQup$SH!2Fj zUEnh6o?w{cd&QgmX;z-9+kXE_>!s`#R!o)-Q;C=n%o~`BDTwL7(^i@8!@u*w-Q`%s zqR~VB$lQM93YHmP*WK(Zr)zc(pF3lvT89t+s|8mPkdf+*W}-*wW!;?sbq3#bDm!nu z7j0IO)L`RJXT43XTA#t!b1}<_0(j54VEjQSfhWBQ(;)!=HabYg(th_YO8&^9urr9c ztMAmOZwvcJ? zmG>^jM60lJx5SS@9+bbKKGy#?b^2(FLtRZ01Mtu8m`#OffeNdETyNdvXRXtXs=Qiv zS^7n%jZF*up`KD5yZv%e5Rk-Hd{1~HLK=-w?=Q-8i%@RD6>>I`~trlcNQ+HvTt_PR|M~x7toLp_>c=w?clTar9GNCMvz*%|gj<~3e+K1r55(|+ORcsKu z#;fp`lIEmh(a}?OB^DmMNz0?EBgcD-y)1D_uSq*hqXXt9dj!=K%o2!l;dOHMFTFon z@jetb{vvoi!2#^JJ=1fF#7KbQJuVNHF7o`%!{*lwiuhMEKRIC)vT(e3pe<|l`8h|B)Vj;g_}_rhX!I@28mPVNvR-xSr_Hu8 zWZy(H7p4Clm5b5F$>s>q99eivhs6%Vm_y}Hrw?jHQlOpbf+Buh+d69Ww$SH?dL zG;f45?ggL9;GJJ_p3=r{2P5kg4N-cxi2@if+cyN=)#)M#Z<>fwpBAL}XIE`;X9E5W zyDEWMP~cLc`S*ZP^^+FfdK`9Kzn=}v&8wQfdv@ZLZ5DCxlNhec$`XKWD zs{8l{1?9L;%;j4JNPaJ!(q)BdKalfFc^jDkfABOQ^bdM@EemNONA}fX4c#Fj6P#^c zQ~cF}tgLLG%vC}zl_G!_ZE%l&O=U)$_w~|z1O7#%0jya5-<&iT)BlT;=HOucUw-ue ztF6CVPtQxtwWprrxN2@o%g5GPGl%Mgumx}vSTi%TL*zh;%Bw(JnLN!; z%r-!Fytw~%fN*JTZFMD=my_%TT^jHvw5D+3P=W4)?SQM&_TM(&;Mm#yNv#8E=dsRT z@moMN-+@ws3|;65FbH`9XTJ$V;Exf^+Za6p5J7Y8u5*H(@5qt}F$g$#aCGkX*f=vY zf9syxwS&P64DBy~9-TcJKr?uBfKfG5w1B7y>pjZx0uhek8h|#lc(g$USApmX@C9{L zv_((|s|%}Y%cSNW`x?q3Vqhn31R5l*t*xe+flEfzT1tQg@yG*f+R9e<6zB?`c>Hn5 z1EW`cz8&)4M*2^t7PSwxKdl$9YXb6t2N3STx;nRh?7keU2|@J7Mn z{0z(Z_W2f-$>Y=U1wgb1yqV|(=j_~U58nQ2|AA^~a{&X0-2ptz-DiK_Q%2Q}%}Ur> zGg5cM&Sq-hye-Bv!LQ)~6km7Vhsx3*K>Y0XKs*8*fj4?3i?ixUgcRi-%eRsN7s*XN zXV7rYPewoeL+*|cUR+;40wAmIv+An>o$Zf~Ml0@ZE-ye+GCvZZ-LMz^e(Zvbf$%`C z;6S}}c3Qve-x3S~JJ$g_ccNhLoq@VQtEX|d13)}B`(GGI`Q4oqNI>pSKoIXga|6F^ z3F}_K>h5X#9+mwL7$pJf=^kwm!0*2&{t}P*g}+F^I|4eX0Ka_gpd36uAay|-Qgt+> zcRgg+-{0$&fGe`2gYydGTdD+A0daPC0iD|0pYluE z-f8depF+Gh_QP($od3jY0@cvY65jhj4~&B10yRK%68+#&G9U)3UDRI@#Ka% zhgeHq{z(b4f~W!!usM}WRV-tr8ep}3uR0hR0%1C zA#MipwQi9FUi8&_*KUq3o0nZFeVYk>1+HW$IURpi^>EP-m5ditMNddDK#{0jus_9y zyqc^q47GsIn{;w-_2WLw-ov z7$&PT@MRX}HpJ5fe8A-Q#WT}(nO<T7B=-0pU^H)S}MCS0HiXSpDjyoMAGqkIt|$$w%(=(CHm&z9!N>KDxV6!6#XgS-qyrUVSTRT@f|H z_@WQQlsZ0=OS&e@O%#KR+zWKSe-E8~9OCi+HvN5Ou5vH8(y7#tBa`yG);#Y4T|jPv z5^#<-BaSS8Vhdyx$bl`fZd|{p6>Dq32bCskS$JM8@986j3~pvK9}{Ww%CU?Wme^!FqwkHdaJZ>j$o`ubqA4uX!hL#csSr=vqQu~Slum%(~d>ph%pA?kM=<(Y&z&yHf>2m5!q@sBoPmH!W6Yyt%3W9O zK}@v9N&pL_0mPItYsxI^q+m}w{fGo0>-xOjLL1_~rXMzNxZDH91xtObF!A{F@qLX0 zfvW@NS}^-nmKqko`F`sJ-=up$6u=>zjYGks^_+RlQ>>YMXV>G%(?ny~Fl(5;27Qb-tn?4t)Hj^F!#Wah(R~|x zf;zihj8?obs}sE@oKofp+}`G?eR^NoQgUj$j_Oe{Q`?nx%XHD4zf3OB6cZjXU{)^_ zfG=w_%rMiM90{e$59hoedu5!Ql@ZUq$KbQHAh_e@hXQQ3UJV^kH`pl961bII5}+Rj z{R2wRLaui<6FACfF^c%OGn>_`aCg=yR=PXpxSXah=m)S$FNbolh6gdPA1ik@%h@l} zDAkoJ2+i#E_l_?0hhJHw6wTVz-P9%n3y;Jo+(uD^go`!qyU%0geojiHJ2pu2c~()E zTENBfGbuj)Re@mB$%&O1_}EzXr!{CpJjWhd?wc#A5{meZrkMpYv#(fcbH}xbu?+Vi z`7ON>mtpx;+89QWSP?DX*cGqFnf#sG&uE&WlPNzMYbn>fs+T67+O@H$AC6{FQo8&19Kvh@rDscSAE&YL>&H7KAssNu3GK8A_v*6Kg`u%opQDW`#$~pw35w&wyf(OoR zi}gY35BP+i40>3Fij>NVVvx!5O(j%j?Hv6&FTH}+Z0D`$y|^N&TYx&#OfNteCf#{x zQ1)k)FQm;phs-xHmXdirofEUK>A&v;QCi;~j71rOk-0sLiDuO7x85x4c}4`ztue}? z8LFv5UZZcPNL-_7MG=&B>JDD_p8Eryh%0Rb-(xfjstp=PH__Khe4Yoj71j;8D9MGA zb-PzIn#)R~3D4o1)3)%GTKfw?GeqL`fU`V>_sCrYQ@rqbrK5&HJQ8Np%A;#l8i6@1% zOCB6@6p9TDCcLc+2TZOAl^6zTnPHLYd>U4!eUb5OSwkb4y*3~2S_bv|G(0!0TmC7l z*j|(AtYQOZ-U&ck2+!fo;b#eYr;ywT-R9;ze@6Uh^o%v=+X{>k+z|YEJ!TEyQnb7` zSs%L~dmDpA*k!7S!HsK5ggCfIC8!Im%TCW)V6+$2C!vpr2R@o(mY9AXWJiaW?#4IB`ja6IM~bs__Q#}$PNHcn{+RM zS<5U4?=~#&~YsqCdtA^^WO3tZiaQ(GBQVA zC{blXllKy**xPyW{vZ!ZLK*5%DcXilncnjYlI}+DN_y;jaY^_K9qX1>bORu5>;Kr4 zhbC{UG?P)v138*vN4UT-nfDLr5O8peIGNn8PqCD$DXwqrUHFhuLa-)}TwSyO*SQXC zySR^#R93^6Sii;9FwEh4jxqM{BCbYDQav=IuQWgVa#SsPuAN-syIKQD`a%SzDnqSO z<@czFR#h3_`}^`O;UiUmj*M3`S~~Eb$2z`00oR5fU}sog$CsycLi~>5;Uf9rF_g>` z3LMo;4(IGXQHGGsjh9GW;u96!=UhURgP$`*P{^mupR(_WN(W6Pd8?*%?o)_gp8TkB zw2T|e;|XcQnA$RT$M}86GY&4k_)wOYX1C`zHAMojR&$6FI*9mL*E-HZx7!SJ)>8;c z79R7P%Hw%pRLk+eLhz+A6=WYz*5P2Drq+>FLlY4Ku;8QxL@x>v_hgmhZn>$6r7CPz5|Ao_&q)f zhL^}EUA?%2E84Z3O!aajml-n}#o}hcT?UlGG}K8xGVf(QUT&5~)6Qa7HYn}t6bz({ zIV1NgR8y84rSCXf!Mrl9aPz9~|8PSaR_QIgWcxRb1tb^I*ow*dlB$6@jG?;N;(5QA zJKYNA8H>2E{+O9;J5wDl_qJ*7{gn;5)Q5GQttBz>mY7KQx8{hU5W=vUkkbMF4Gf#f z$Y~@s0c~~|m5poj)b^E%T!?kzGE0?2<^z$eo^G|l2Eny{}UCZzO|h5xk-}W@r~MV3K#OcrhE=6C!TC0tCadg+G52lpCChy*Z4fqwV4VX+^ltCxpDMUi?|k5^>&3pm1`N6;9ZCQWxb_$ ze%IE&1+&MWTNK-zNEpleIr&vO72`Wu80aqKD8hx)#DV$j{Y+kW!$g+#jAa{;0L=H}< zJ|n$K5~oWhsQ$4C0fi|XnZDg@c=L8EHt9CKf~uBrnF}^XCc9_l_T|y) zQEoB9Mf2}|@d`^{Dh`|{2{MhC3M(tc$iM>c&f7{+U$}4iYXY|DA{{B~fc<#GH}a)7 zJ}H*Qu&!)AJH#>EtJaJB45QfW{$wBBla&ql<1<0|GL2Rjws$%4~If@x%JPi>iXz-=#L3 z$ef)@AV{9Kt{E%6%6wdbzR(Zw){pClS1P+*r@LbECD^Nz$s<>TWwSp__|X2f!bDvR zQ1w|{EN(ziau&t=G3!s?%6wiK2TfYEW<2o_LvS`k=$6G^uW>}M@O}HMj3JL6qDtWN zHX(5pm0gF2BxtRwm&{`5y;5RjwOprs7m;ca$aMpZai8E~U-d#&3xv}8d>L*(I30%C z82aHPa{3(w*`zrky80lVadVn|-ixN@i+&SvZ4jedT$;`eF-6oFFb)?=dd`o;q?ek!epET5N7o!r3 z{##PoPM>lL+C3CDlZW;+h1f6~qx`fSH_h_e+b_JASl-R90^#BT-($sPX%W608F-6* zSKLAlvrjmn{{w=?RfW5cMmFg$y<=70OO~_NpbUHwmZPv$_pY{5l8;<2AO8lP=PT(&|9%ptz&C$)9T$-$9-76M-`me z>8|f<6K!0WCUTTD13d7jtTJCH#iR;xPAhmS&IZ1S<`ua=T;xhRLh&nJWL%9i&EN~s z=(1O_lR4fg7omhSL#|&AVnhY`tbU0!Ad3P5_QFijzHd=w8+RnK*Rh5^H@A@I`_Rpc zn>#E6uY+pPKU~d?M{F{gT<=Lz_>~!PkmQOs2BSuv!!RZ!zw@Z_?R0!B4A^6Sh)C1m zhu+vjt19qgv@R^V;4jZDqjx_>*^Hng^q}jD9^9Iz(nmx?R8K=aTF8l?!h7@1XqS-U zo}x<$MiaO>*v!klHelmr%zr+p^;6kBEoKYgdK=4xeZ&s8A$jJsRctvQp4v}hr5`!Q zIoc=Dw5q}PS49j$^5Gias-hv1VJ2*k*7d6%@5$DAx&q2tpiDPm%`~>A?^wY(1rMV3 zbD{$VmG!fZ#6I>#5^k=Su>seR14id5gdYXp9_ThvnXhY0Hmpb4S2ChsgOymnG$n14 zDz1Iia=*%~L*ydX6e6Z>**DPj>MN5GYi)wzD{Pa)-QrK?rjr`6alU70*V0( zSSFiD#XybTO%Z?J;n^AM;I6qiMWr|Wh&hZ2iiUh5w^esrmo4^lR6z(8e3)Q&xZ=jV z^6}~@n&E{Y3eEaCX{Sjfsyc|A@@fg>I|z%#M-0)g@%HY(M{RiH@3-}lIy`S;CS2SG zIdt?g$3on9k4IRHCpnJv{(?9xccO36j%Uwp!9oLuIWpU{iJ15)BOUfWNkxXzvAiOi z{&D-VGYfapeq_Ca30MoFyCP6diR=i?Ya-I}aMcn^pzW5u-B>$l`v`+sTy+EO57c|Sc*wu2OK-GV<;$_BUf3RL+p~Q?MTB76s;POjRs#-ogT1&57>4?? z69d6%l~B7!Sn+X`6Jp(Bqu;gX=@c=ADk8gN+8T@^r9D67c2jQo=3+$H^A4KEZ%Y!Y z_N3uLf>mrau|M==nH%di=&*L>)h&-$hBqWNZ` z7R7h##QXNH_bkL(ioE<)#UDZ5qG-Z^I+|xN&Tnj`m6MN#xEr%&DQ?Zl9{(u z#jtLK!47wp{_YadG+Z%D>ZaCa$V;BaiM@{q)HlRk*GShZjS)xCe@oz*y;FGRU$z&|!#2fM)m^9W`^6r}vhJFjL={_k&BTqF z$Xj$qQ7F_HnPjLKSUgm4KN%q~Qe=gr8P5#Ke~~JOzK;#n3yHvif9W;k2RF^pb&T$X z2(gXyyr{xK5`rj1u^vR+SscpP>53x9|ib1Ep<$+Y#nWqG|(S2usm=xTLJqun2)7SGgH$ z?^%l=*_liOZr>A0ErJ?quXQI3)*27#f2+=9-w-ev;b7tsYmG#SCKJs=nciGbMrqwhG9=>BJPBWSw!>CXM*3t9Q z&a75|b0@BIe~}2-xbxKo*MsDxYpxy+CctGEtVWA85dLq@jNYP2(3Sf{R_uOH zXD{D3Yu{IKWCvXNQ*5?S0(6)NKQU4klZ&}e6QX;$%V$M-8rMNZl2`-N2R))x!w^2?k`(xT{g&_^`wwsBhE$?ij>C?!loN^Z) z_CIAEGYr9aDIyc*L_gi{iCHD$G8!w)3F`4- zefk1FxUdL@MQu5hqH|J*-SAfna9>i@HSc(nc$7hhho)+Yfg7CZLa`CatAwkvoA&p0 zCtJvqijg9_UWli2>Q(kL;kATv3tUM81d?BUuxch{&+q8FHieW%Ic4&hxF&@y%U6Ha zDUR_RERA*OedB3#Sz&taDzAciXQFp(z~aBNU?h1@QGmJakg;~NRIRntw#thX`|RQ5 zvFCWEpUk`)@|Cld+^*J&h6jK^I_G^NZaN*A07e0_ZD3A4Ts>UQ{w(H#%6I|4!h*`S zDfX66!Ki55UNM^HAlGz!8{0~qdLS5C?Zk(tYe$mazeepdNn5#=f8hTWFhG?-HMb0b z+FWfaX|Yj_r+EahUdn*BDvEN%i-2(N^}&imxTxL#R5%dWU*fT-Gu&>ahEZ0TMYim5 za7UE)(=Ax%NJ>t!HsQJfVi&ilYQh#N#lU(O=y;FLb+=(;=WHn4LmLue8hk4s4(=xF zw%8E2+b0t+1ZedsxA36N-2nKUxH3O%ocQ$8F1YG8D&MmLwl0NoPT7CHr%po6vXdO# zMI1mO;SxGG)8{^|WS3zc9>sw!T zJcSXc;GtJ2J0qbo)Ai}Y?%goZ)qo;fs+o8$IR_N0ODAgtIqD8AJCJt#dxZVAUJE;I zjxzD~l$@o+XWCyI?@YZzsHJO`BpWBWcsq+fXMDJ3YZ;9mwdLyX{NE-P(ey%-Scl02 z#hDob$~dL4T?UP4Tks>#knP0t#J>oC*+1mE!aGAzDw&y0aNRPqn9bjA`DN}ghT?4W ztXx$|p{Xcc$exwH5*DTpjn+)*+-72plG#SBQNVOA*_@IZcTnzm%Einbbj4*`b(~XD z1^*gZv!*>?=3&O0L&~5|;1!TF#D!sEnpdjV15@2#y=Iz|uwhjN+dUQTNtw79`sWa4 z=_lhCDkvFqJ+y+nfkJa>e~kO_1a`^pF6_Z z8Ix1k`~HJ_7Sqj3jk~F0I`jQuO$R~G_C*lePy8q(z#H^JE`6sUbJknO_fFpQG<{># zVJe0(DqUoLJBD_l5><4z?RDewyA7=Nun90M{!#E`7Lm)n*Zzc?c46b`dpg5NXZ@EmqYRok5GEu?b=^h8D*Z2!t)7tv@db)N{jbk9J1 z>!ufG8_2vpBK$m=dicZ!y}VB(ouwH{a>2SKcKE=smiqG5Kwv;55=(nk=VhKfHd673Suhf&a-@0bseV%a4JIC+vB+`BeRBjU;f})#x$%112OW*u#DNVccsHT#MdgTd)*H$?cawg^=hh9 z2Ec${q45YGage-nOK@VKB@=$Z+?(7OrksC2imU;gPm+Nw)^i!*?mYUr5;9OPS%e}o zWwfSzh-_YEwY*YyK{7Wy8{zT_;i%95SPlc;V~h_+=KHU4m4j_lTw_2Hy?rv zX-)`(=46!rTxt9lQYP1#R%q%_it4$)zLVo+taXn66XaJo=*k)Q0_skNq(jH zas(dTpRIQ0wQIjlWq&5E@sVmKKrD)%U~CSZ_$-c45)d;*bs2D88e31?h%S$sxa7cj z`)od{&0MlcZtv+2loBG@goda!ZewU zR*L@WLe!qE{9tt7IHGacl}#tH5b3NY(LnkSC8+3mhNLVUj$&@qrMW%>CSi)012&^| ztC6R;j`O~VV}XrU+~GH))T_xWk26F%C^18l(=X<@HY1W^5uvPsazN+vRK=J|> zMJ1}=ZsR~K+4?U9=?*=MkxQ?KF1D$)%&vm8^2aM34XdqF(+whsIBJz}!4u7hOg2Jh zl6{IgjHwTq%k_ruiL?R+Y@knqAcuD?nVi`}E%=q|8MQ+=dpN)h5qqgZtox|u-^;W} zL&%PL9S>TI{u|6GzKDDc{LfU03ZgJaQ>IKc+Fr_}8q|MH8Vexd9>%9&)K)4J>9mn`>!HbUVXsgr*{^qJCr9DWvD^`bI zmFiHlZP4VV@zGyu$-8pAMtmOEvk6kDjjot;^Fize;HFGxv>^NiE`PAE*C#@h(1y| z@`VLO@$e~G&OJ8TB-igatGaxH{!qOZo9Dt5*;Tp+)ja5t_-nAp895m?hMt9hoP!i< ziTLoCu;sN}li7>Qil;CFwffI6hizjBX=^jv0Fx0SofLX;2J%S7dH)hJRA+&ih5zko zt*|K0I~IW^UD-%aS~xKUp$NFU#&gYDy7bi1jYZ^4^Sv699{5I@sWixPKi?CG#({~w zORtyh+W3e)+1GS=ooK!`V*pip^5TE`KC4!K$JmvTmRlLgxheA1TCd)S zM@TQVXRJzaY}avkXGxeNLBx;P?rZskp#+^(LrE?9_BMFAJe%~}s0RI1gjXaw#dgR=K~ZpQegz0Q<&cnE&!Y47AVufY#6Ww@@T+gU~6-;-Q>XzO@7izd{N|na*kzU zF-{fmuyJbA49E}|2FGK8i5`K&s2{o*_xh&5SUPQEBAZj1aL^Y~rL~y0+$-X7i^;?P zw3vxG?tWM+j_Rx1hC)G`Q<=@!@#Ep3oc}c5ch-Q33dsg!sYFQkbj|cg3JfkKvs!wk z7MmD`sgWoI!GI`Xpf4g>fwuw{h8{@tt9K$r=G0EvjZSy}JS*v`7O5%DmL>A?54tO$ z3JMtiMW?-q#`byUL>)VnFIJDO)uKJ5qTp;&4>qxQe1URCb%@7Ou`_@ZrlxU5dNI|V zsL`XH&e zZ=uh$anoit;`fnp1EcJBxsAIsJrs~*t$IGpX3PGBitb?&T4)+RJ|~oAL%Ka_5i@W0 zI>>@1GjuFRK2emaNL2jz_o^wfz0bjaA)@jv()2+sMt4h;23uT33Z(ti?P_JcRN_wV zOMfL+-+6KPb}V2R@N&mYbXFD==TE5}xn-hO3eD@J90djXva#hNHHE6HDN}NU2*%y0vC>KPjkGK$!5lLXT@wV&S!)lKbB9pnW#@dN3 z)}Iigk{X=EY}~&}olI_(qZ<^ zmqK&*?E?52{UDIWZWv90R(lC?^z_(m5^-W2bP1v_T4AhNC!iADgfmh$s;26MrpG&$ zpWuJGDMEOlQ;$SMmg6UIY?&RB(3uw|*jJ>5DGfNy>$Q)P0YPKa>zrr&Z=!Plkskrp zzh5~k$o=Yce#{f#f|PvA-mF&xD+tI73t$|AJZU7wwzz2$vzB}SzJrm8dR;uaiq%O# zJydwWu2Y&)$cL^!j1Rze zf=WpP$3v&nxy#4?%)r_ABUhy_aJY*SHQ$3XHfVIqYMYFoDvYiJBH#Uh(M8Le1}mVc zYpj($0FJo6$;aAGf)pdx5hmOz12T3+nC2pYna}m^MyIK}UgyfH@+u1NuJfZ@Ml4M) znm@TU_slkrdYPycEXFG0;nGm>x@L%b*Gt3O1U7SArr@T z%@F(Hd6_7f4&c13rZ*#p%kj0&Ih^m!lymq$W119ATW%5alAnve>%ssQSB$42M)k{r zu5d#9sfBK2zKpy4e1mMJUYeGxk}7gPt9o*dIRnMyUZF0->mo5Y|05`q&+Ouq;mNTz zGspxiI-zF2no-Im(?j!`D)C&@j1fKMCmm#j)6L{I({^U{VL%1jn)iRmw?U`pA0l~P zio0E^u*4M*FYl3dc7j?PWA0jH-jL_t-$76DpM#X#Z^tVlS zqsT>PEQE_usnL4y*bQ!LQD=53@>?{4k;o@a5NOF+=-g$b`?jd#?^7WriJOKN3_Sr2 z?UDf%@IeJ>i=tfr+yi>a6?2pN{{mchnhA;dPj~rzZL&qfNhGNkXJ}n3t%)|s8x4G% zj+ihGVpP(?Dde$Z&Fk@YJ#$T<$f#<2Q>KR!klAw4ZxlnVu_IQPva_meK*i3)Emhjf zm%v{c904f^OofW6{yf}(me6ZXte82Th(nJ>q3i8UXbVw_^E(Pov?$pvX2#!XN;kDh z_o<(`V=o!sWZ~(N*K3>&N}Hu&H@(Du?rLre`_#I#^Ozv)#j@DF#}W$BJKY(wE0Uqi zRd(-XyGHc2qMow50?4m{YmDReQkw?D9U?JBy;Wz|G@?BRF5b)eOctxq{XRf9Nh*gi_jm>!Io6X3I2s->nW#hB*@nhwWv^ezpE5)lM(W z%i0uP0DkUHYeS^ zyM5DmUFx!y(By0IQX1scWYm>!N_i8T;js^jAgMY^2zC>^Wp@0z{4kBcM4(G$N9xY~ z5+R;RDc+)J935M;qQ%eL`5t3a-mtcqmMxCBp6#fn?9sTvr1~urm}4W?8*c=`g^1z6 zpHJ!_(gItrB{XP>p&V@2V56E`W*k1ixTNSsN1sIR$nC?I#G`UOjQ3{{HLB7}@FDD} zM+=o*kpNR<<8{A>H5-@Hu)Rg#v4yFLYW`!oWFJ&Jgc4hQZlkZ2 z0Pn(6E;=yhxLkOHlp4P>Ka4m`xFLO{jAAV2!M>f0K40s#rskcPhRoaC!4+noiQBcj z+@Wtf9sDx}g?**vsF23?*=6_pSuD&IsR8REk8mD0&(*@?TWtVj7^-k|p1bV*BLnPU z{DKR6G1I+_5W}APKx6@PV{lDpn(F8&6 zRi(J0TJwASvKn-TT~0KmXL{w>>+nu1AS|X+R#c$EDRi@FPN1y$%iP5}E5pJ*J zF6RCN4!UnHs7A$640?mMUr|xiqy)$hCYiF|Efk%}S@B~fII6MJ76BqlurAu}Q0O7TN3y#Rr!CH3zp{Fp1&=iYVvz8obh+U5FU)uqQ076KBGPEM6OK1NliQRzJYU1M<_wQ9@rq%8wg~z-|Jl$ZVgT1O>zp10uy$ zK7hz5D#?ETQP@=XHWy=7gKYO*VlRMiViU=v6dhxag?X2)La0noUs$f$h8Ai11bE1prn(nG;@*t!8_+2 zrp9rjLu$Zy?CjgCWaIr^SXW?^zwouBX{yf9myTCw{B6obmqlMEiWphbX;V}BJox&k z)IGX_H5_Ko&z=|u=+s_SIUa6E#^d)t$V zFm^JB%U^v?KC$U|$vRk*@~Gdlj?+#g1k{sg4|P!mixxPlF); ziMg6OPSpJLS)K)1Xb386bcbU89_VrGp4q46sXb_z-L{_-uL8Ptx~$PVlAG02FOlXW zhl4aCKSF9xM!lOZ-4G!BCS^e`FV7e{7rvhiq86WQRbMucwku`n456%2mYnf6MEKZN zw2No1Z=$1x$usFK%gkkpN}Gk!8i2Spd^a?@J*c=N(m>!2ZCUd+4y(m|K~!dAtL|x= z7_bK^L`AZjN6mX%Ik~H`>9J`bjYZ=pu5y|n+hQUZ->Y%DPq_xyvp>&h_OP=^+DK}o zLR+R@zJlPfcxli*=G01ftp|6*bIH+e5WcoY$z?|PDCI}F5zFU0t7rBFz)NPIOB6vL zOwSk#VH!~aQ}0VT7AHQmix@w|Ku~AwyCI;_)`@Ug^0XbbFWmBXQ5Y= zr{~$>dl&<2nxrJJjLE5g$ULS0sGkz&j*$uZi*}*;W{99w zV}rl%sHA{j-II3Mc*9Os0!1Hcgn$9&BqITMupKxDtfQ7|ElQG5vl!b z31wb&Rlz5HS!I{MuBGKU$LTy`i_UjdVIJua6i|>K)I7+##W4A8fB)H$`Mc>~Bd|#O zZ_+J$Bj4m-9=3|Fq}ffk+~Q-_idpA#}GeVE@UVMRHYXXw_> zE8Aa%!}k0i@yLfY=y$+>G8o1(cx9>(CX7e!zg`UZx}a!n2e~`w2Vi8W3{3CSdxDxq z0+Ai_0|6Hg9g#sWyB@DHm`Yg;z8RwH*GqaNjQgVTnDyKCp>9>hX|M#_(z?Gqax@D28j$5V=p<$>B#!4wl1W_KYtbWdZ2a_@|rK30Kak2NQ~%#3NV# z5vq({|7vOJyXM#G;GaMS5iJ*9<&IRl0}-|`=e2a-D>S}T)=8jPStqF%LI!bfz2M%m zs281YSo0C};)kKrV8iHA6s`9%W`&i__J|q*r_j&NL>epvCF@*jbi`i5lOeJ^DtK2j zW2BfsS?={riulFk8@{!Gl)RS~OQZN;tc`Hhdgj;@;2?l{Zh<6-A^3v!EwPLHje~5~ zA22rEOmjB*V_UhHu=V>xO!CKsI6(Wp&;k_rpsUtO(w`=GN0UP=dvq!5$~sMMSPcGc z{7sLQMrk2SgUILkFJ-0!G7_9tIcsOA?B5v;*K}rculDf7r1h(tvt`Y)2(2-uM~*tb2f#68>L)lC3y**`NxLFa-87+H z-KjIf0iU2dVFE?qVzbown^6&JV}jY3l0HN2eL06tpik^W3!#!ZBF%JIcf z#GfVv+G7I)J^rVPq`5w?fsnH*?P5r#oKNq`tN!-|`g)H)zMm%TqAlwYEwZy3IZwO@ z+(=%PMh*&1lZQqyML+MSEB7l>{F!3Ox*wdgNB`CG7)~~F~gAM3($q0 z{+EkNBE;~H9fwKrWyB=s05zI}YhS(%e^X!wBZ8SnubWZE7A(L1`2;ww28iVV^Jn}w zd4lm$V@MMIuRr#p`kir9oZI|3zrF{+w1h@KCP5w3QlpbYUhDdO(c>VtD%}%@SVycz>ZrA08vTY6QjqFu4f!|g zvZuWI(5!OYkFnam+1`Szxr z39FBVd5qb&D1u5`?2KB)8VtUgOIwUlB8fSuT$su2SJn(OQx1g^Kl%G=dsLvU4}F4+ zOsi*(fnUk)%4=?lWUL5lBNnrsVJ(hLI0jug@+$crG#bsDXp~djgcGML&$T_Ta8&W- zRsLgjq9QFb(6QjNLA1P2=d${i z!lhwTLLa(vs{~&^jn=mGNmWDGwAT&VWF?)*@*$YQIeT&<-?Yxmv+{V<+SR9JqeYzm zn>!pNwYo=y#PljnbDbICFg!;sU}5Q!QmnLZgklk+(H57C~R=JIh;?MUPW!V`*cy((!QWUK77gF_cXi=_jl1^sryIfm;TV_BG6NiPU<HS0jI;%caffk9xs?t5Gs1RWN7b^moftMu54|t^u_&`lQPa zK{1Y0@SqHW#_3;io=O^-I&mWSJq8hB;ApZ^S>U(c7AtqvVDP)cY8Xt!qQ&FmBJ`A-!ZUUv=M5QrjY)H5!aKfg;(f-CfaX#K5VCd zob61G3C~nnk5!E%vh=qOm3M*cMO%7pJ;VWvc<~v9@Sd30bU1x6g4!-_J3KQT*L5*2 zVI*WioZUUzzgIvZUEG%8a><(JShLad43D0ruGW48qZYbAm7pC4o#baxnvEw@5%R&q zQ(t&-lV$ne>}Or068{!{R>gv1H>`cq*fx3pER+mZ^pP;pI@+J~6;sol?1K?m8Ly=t z61jAYIPJ1UwWs~37T0y1ERFvIlbH_AYg1-pLL(eLmI15P`gBT?l$mo&<^+X)bQ{@e zbeAZ-;CJf3cq_|`e)pP*)t)JG#~cJc!%t>;MiX*%UbRG)xA!8>{j#`IfzE5j;M4~I zphq2Syk0{XmiG;#~7h(gzWKM&`2A}^4zQ}TI#jr$?;wR;}-!!wDd=z;Ru=-yPgr%Zh z^DNE$@pFkTa~NrzK|Y^a9O-A)S$DoP2eBCo_S?D+m=Q5kfy4*&Z;cAj=P?#kx%n%T zf10@Id-~VVYR33>ycG|s|AW&ft5Z>>isD|p zxOw|W#-U*}nThs(+9O`$1n~BJazZa2a`MyLa{WO79|pYFc5dg4S(b5QRo0nxexA;L zjl1aU^?+IG-O)LkZJwQRQ*&B&+Iri2@7XO7#r9G1w;n^jXv~}F2zS*6fv-Aof&2Sl zDYK9#pzI%+%3DKZ5|Pzxu=La3l(3vqZyYuxvTwB9qe%K8(od2^4ISOpi zdfj^(_~s^t-Eh>>S_f3UGEU-Gi9s-hcv5jy(hE8haRY0% zVTF*SjxfWdQ~Q&m!x_1IB?1}{$VC(HZ{)I$5nm;dP#ZEg7=w5jmMxfG(0gI6?67_% zfIs%wk56K)d;GdjI}W4s6Q3p!#Uw&^EeaG*XN-zGq#vPcT3zae4Fk<{Pb-BVZuR{c~#2BD4mC`{%Zwi=YFsB zKJ8LyAe_=157p~vSK|5?QmlBVYed3@3?{3_OqE6(RYt^p=_F>4^DHXPPYEwFz{#jE zCtHFL`C9aI)-2`n0d{8>DuJEk+jpG^M>W4k(-TEEe4vwc@^;A#V`T2N84(nHuw=Em zBWp1F!Dq2v6l4sW&xJ43$-fuvvIe>CY3r}t(~yK$^WN$Nbfp||*zSyfLesDppE;+| zSgk!kqjoK^J|G4TQL||EwUmsaNVm6WH9TDwMFIy9pF1x`E~x0Yi$S{ezYC~tG6dVF zlL>tuc0(F(ETZcMn_VXze&Ao1liGg$iGS=#nd_Hq;Vlc{S>>dWLW;Cdn7Fgk^st8y z2|Ir7I&?Q)bP?4LJCcU^4VU&qXr(r4PpYK!`k_nyLCiicFg?j;4yvsnSB66LB}^+_c5P1%P)&a~L|?dd+dj|^8C_9~EB-09F%?-dzf&obhECwbcfRt@ z-%2i~|JBEeQGtnjONS*3cSq(dP}=iQikoCnhUa~W43G}BL+)qGA9BYFZ|EWB*rS%s zD3yaXKgO4xE2Do6QrXJ*T*h~85rTEXe&OmBPNlzQiE#K*M>Bs0ir`j!V@A}%5IG?~ zQsbqzFQU_PNTV<=l3G3aS08HM#?Z)gwN{tqa)6wYVZ?LgF*Mqz$$Dt8@@G$H3Kzrm6O^<%FgF@A+z2bEEB$)`pBWsD zD}81|jR$zld5KZcX$FbH*?^58dRA$g?Fm6fK&#DXL_2uq>R8css<3e90c&Tr)Y&G7 zC}zcbrvC|Ie4<=Dp`yG4L@>-Ek`aqMv20~(&}m>xup~G(#0HC9AS|W%F8L+&p)f?a zs->k3rbawoL!%JT?sE<8?0tVYNNfYX9`)rE2%BL!WilzXaP(=CS{NOs>nN4gAk^|- z;|J)q{PAOXcMfG7rCw@WIFe(hSrDn;4VNDsYq@6uh#*(scNg7~h;IXH(E)>8S;+6%COr$)5$FH2SE(%;j? zVq!-UCr+0!$F3oj72u>&%a)8j&d^I}Ep>r^D*=Lb;mn{d&hx_LWlc$YVUhQR5dyJG?YcGb`y~;qklj<&f zn#?dB3kP2}tc6swjK8lkWOdpMax!j+-hyzX2u&VD&10FEQP+UXCXpCT84&emkw5~O zCz4Ds)s}@fzq>5hDQ1U*M$x*LXkM0z3=gVx!ITHYU&LxXC5!^1OyV!089i%sZluuW zq9{kpZ$=HsvEA|WR>g=hRzM7bb^Xb6E0~6zok$ZEBV}9>_NmeD9Xj3H-lNUQ8yo$2 zhLtCR1ykM{sVby2kE(5xyh<%-MgPn&fRcT+R=;%boj+((q}4;2{*^yjW1GI!sfXct z2*T)rxI>hGNkFM0MJnNy62;!vfj)@bNUxK4)H2GzBO(lp+ZTe@i$MP*gHY|zP4h?o z)zebjP2zjjKP~=Nt;bP<{ za&{#lYo=aC^NyBMee3;9uBnF zTgn7#OdrR_cYB)|9MT$mwQS5F+MNW&C}S)@N$8oN zI&{2CqJR^70AeVVD%F7o!@j= zD~=m_CWX&&heR0bCHiyv1Nq$cv2lV#71A*R?2f7L+6|l zeeNIUrG;`4=QCB0^$P4KJFJDH9VCX^!E>!p7V#rUenZ64C@DWmo(~<8wD;>RI_oGy zGAqhDM1o0`N5+Ko;Ba~-v%)G{uS9iEMXQ(x@&zJ?Ww=3Ukctp@81kcgTlo~YC+77k z8$aeSo>@n)viL?Z8#t-Q)?D{OfS{!Hy)W#r>Jb>Mv&tNk>{K)xuf+J>Ch1gzz#la? z-wF{*%Y!WHh;Ei8`(;@;ciNWS7oU|l7RP8UOt^#}C3=1iJ1mHoxLr?EN{>A~dCeay zxA82AF-sLx`!DA8<#BD-oa)48?YPZQuXCy7=QR?iWezW)Dm8BMT0IBHCS2qlQQQke zF14KgH;)~U3;j>C;%VlKwDMoAgHT&D0etI znwL01%1F?ZJLAdTe{qJr8zplXer5e-@p>>-VX|S-yongyyUOLhjjFQXY-f!MNffmV zX>lD5=sF|urAEXNC=;eJt4lPZ&2Ea)+YD@ZJ!%n`q3c-T=quT?cMV@ly8RhCeRs`< zFn|%}&RLM(khpTkrh=Im^bzU5HJ=^FZwbIC{um5E2D56g7b`HkfVr_P&>o zm^lYyQixtgW0X-LFctU8sgy@rzTul*U@Rx7itKQ|6e^c9a8vntbE8<(^qw6t=LXY4 z>!tTWdFX0XRC7P@7sd=`Ntm8MnTzV+@l`&pGnBz)`lS;0(EQQ;u z>ZuPj0Cd(>RJ`va+X6QY*#nYL!@;~)Z^g?(_8{KOdybp85L0k+3)y+3@_B3T#vxKC zjOyPG&YEr&|LoQ^h0iMhKp>Jpq1j@tVAD&TNsPICqs+#_Fnpf|qzXrDv0j#iMIYK& zS3iUn%dR@hK&Zbu7o(dhzDV>~$%$KP`psvZIxN3a3~T0YnV$qH&`Ue9$CB7e zezh2~`HI~*7?{Y*o!TrkDG|c~MY9BozNZJH6zIehTYT@rjjw zuD>xS1}`MsDPNvikyaObk=b-(?S+(*s+fJaAM_znabt#c?4=ZjdL>q)CQ%Ouj|6QB za#){ZwCkDro@-QPGWj4N*I0{(7g)kHLyIsO_t+W$^j-a}4w3v5hr?SfuDZ}kEyWP# z8Qo=wDqPVxN`6g(+;S}HytnF2uy%eraMtt73BB1?0fYU%Pp|4#No-kP5XLBgnTZRmSRNm z7>8F(k3li2hW7K&D!i<~??2ngNPbHFO?$C-;Dhg(G9Ia=f6dX7ybw^|`{(go{RTH4 zD(w(M=wPfkxKow#%x-a9K$AM=M-MYE>r$@k>Kg^OP%%jMTWl{!4JCGqa6AFK(n&X zB|sbBDr8tETO&8b`ZU(kK@oMI9!K>2Kr?44$ShEZpY4yeMVC^i_C~$8q2W?`EeA3c z%L~-pj@0$-|9X8|1O?E4ZjjDJiSOeEI4&YiwT zohPu1#z?*RT37ZAR2_&(_;muzB)I6xXcz2@sk(1FBBdQs=86T^m7(wTRV7`4qJ@;{ zI~CXz^L($*26MZba-T!&gsbg%uvMsvK^PepTGvCp0s61OE2RUDx65@v`!hw#h0XkP z$=l=u8pyS_e_|M8dcCUE9qK=+S4I zKa)&cFPYGY*=znL#y)+1NgdprYkyyZ5~-GgpB{0y3w)fIYmG*ZO>2!DbPORkV4MxT~JJICj-9#SC};bs7rr@K#t-;Q zJal&i|97EEn$5Cp7Lp?Ccl>Bga&tUOY&31qZ=r_!*Yk156XjZi-bsQs43bFjs@(|D z3fJc)$r% zz)U9|s|U<+mttwCP1_hs*CL8>ti?x`Dae2AkG;A)I;x^cN7vBwZ-x0qR5&Oj>d(NC zn%Lz$>o(I)sVr=PT~B^KwdR;h+$t7gem&msESrwB=Grtps-=OW26|Oje@O>YM-~A| zFYY#CuoWD1gqkL{TL_y!rjUylK2}-}Uu%AtGWVLS;^0u>=NJ~WoeymjQn9YBi4>~O zc1R}df9(BZQvR-$#6LX4V_@*Ja!=UVid|fX=Tx{Q7*UxOtWV`%E{uo=nhq0)yP9$@ z-QjQkzsT)MFj+&a_YVJn=~O+Xd1bw{{p0Rn&`}gx5kCGNAg~74lf$ehFhla>Kg1?J zT+mb?#u-i9MTO_Y#;vdtsvNgxE)+lRf)Ye|^+?Ne(&?h_Gav)o?NlZC#g(aSfB@Gd zO+)$qfXZ)^kq$Wy6+ngr`_!k8!9HmLw)_u9CI1`05v0e5BmAa4nidRJS3C!W*F_Kq@2xh`>h|z5dl!^B zp9mgl80Xm_InJb8Q6gsz>twRB5}d@!_C%q;mUpd^$pYd2V~t3Cec72;RUaC~jauvo zYn(m8_jbNvWO$0gR*l2@t+>C=EAo*L9o$N_E6vdNVIEJyDO2)UiChBpJ@09#8q2(i zb6lSX)zqP<1lJ=bJ*SzoW^FGJ&7HcQ-`KG4rle=FlAcGS#x@Wapg?pFOU;YwzO~gov-0Y`=L-j~^~l>_e;{^LIB+}*PRaOAbuby!!olyH zOjRIu>e@^xM8`3GI!op?P`#mnLQ+WMXIG z_}>#VArmtfBg_9D|8J<6iG_=e`Tr9tZtieZ&D!mti|*`1*@_gQLXGU~lnL0AcW@6o zsG+;LQN)e3aiqMd8Rq}kJl%4B>3Q9?Hzl=SDSFJY;kO|rDJW1>HbZA`=75k|+YHQD z*IIi6a(bYCX%XbXH%E(akAj=~)7L*RG7=&xLUI8H5=3`>JrrPwNDu%C09IrL?k($) zsH?vxo(Ccoaw9BrFQe*P10_TN0bS5hHGGHZ31QLDhuD> z@!-_T)Cu@q*)ui&(K)wofr1f)ur#*4JptJM0oup_G1WZM0)l?AyA_uK>h8wt!!!kM zVgg;u0$vi-269z%L{keWjx6j*qz0YzHd4cYg!C@^9U!i(ETw-3CM2)QCx-{J@&S^p z{8#zys|LE>eaP_(q^Riq`g{F||Hh%GuPdi1rz4e}^L)z(v<6&5NMSMAYx@g|bIrrt z_bceFz~g;U7@;55)bOL{nBw3RwrPy%8wnCdiZYD#(a@N#>#Rv)n4|10`iO+iaO3i&qlg48!N32x|(QPGxD zQQlpB&0}p*-1`)v)$<6};{IUx-Roj&YjJCS`s}|sO?xdn{ljd3anM(3XK{W8l8F9U zd*+U^@Vho8WC(-8_&)lU@ zQPG_f1KEda8ah;Ih~S3-_Wdj1{6f%8stW+12DA`<{1!h690s(cvg7}vzY`#3=J#rz zqg~|y0@;7}8(mWCTp%;3wSwemf5fhOuNDmVI$uJrr)&jpvmWB4zHd3#yDne8uj;?N zU4YZq*ExR%U+JY#YA92}fMyViEuY@wZgC8&VZ(RsY@yrKL0!LL9W8{LIJIdykv)X7^ruJ|PO{~5G$-f%myogt(=QWcrH83yk zpPz@lyXU_;pqal3PrnoBNcHZo#wQ;`e+*Cdj)Cf&{eAq6PWK+(TXmECbC7;5A9C7% z^?&gcfuN4yng3ghtmAmc1*B}CYG2U&*jB=(hNc}CgWN`?%JNPrkElX21#0e~lS$rr z3r6{qU%PvAQStlk9szX{(`6d!X01N26i2G%Jirf*kn=9p;WmWWO z=`6;&kKY=r-+{Z9UVi`z?akUEM{T5-ZM-aqIsJAzMaUn<$DOXT8eYM5Id$aU^|h&+ zL!T$eu4(Uu*;$={bjZfd^M`rt0cyXQ*eM>sZD5yDsIy#^w#HsycoG41Iv0#FuC3o* zpA6$+nA9PI=+`tEWr0(DexBN#p?6NhY*m^fJfs&zXa{?-7kLSYMMzmV#rXufQ|)^I z5$0A+xaL^Mk*T~{kSv(UYIgF3mMZu1H{VFVyu3D*A;RCAIng~5J9rMYtdaKTW=vG? z>*6)#k^?ukiw!WP#KnR;q^IDeP!sp}1$91=q#FB?k%7tRAol@xWHh#4x0_|buXH%x z_A9J$9YGAigs2EuIdtGJBS~q4csH}0-Aa;1NhONM{^w1z0UBK0Ds0y%sM^hs8fsc~ z<*p=tly7i-eiZ+9N(?W0!Nbikk8BrMCIJ9ip>VbxQ9B&KUtS>LQ6t$tlOXTB^;{V^ zz0V~YaH?%K{oTpSR7Xa%^^KJ70548AXtk--6T!YPB?(gSIS3@l5;keol14t=Xd?IY zMmh4Z2ew^c4F>SAs;eKv@WA}7E;MU-fQjc{X5k{6m`Vz|D1q&A5xGl!+i*e1qY=(* zo_Vo8zMgOC7wYq0$8INN?l79SGX6oIkb)bnC^nXpn$UtyK|kn--sUE+3VjJr&+en* z0ZXtU5U)>q`g{WQWS<`-ZJ6B#SJ^q8U2&;=Dw2xnnpDA4KxEfaPS za-ItMC0jT*kKMu*;)*XFH6a(~0L68AW&K=6M2y5FEg6U9c^Ltyw+i=#JF1vyM?wR{ z+gR892SRp;6Ns});@O^Vetc)ST;z#=jai8>L%JbOUObKLT(H1TDQ5ac1i*N@_-q%f z^Dd5pRo%&_L6bD^GR|F-7AFwe%T>bDXmC(Ph7E@d=4n~!CNJT3`>Ss%%j!KHA8+HD zBStRkvEg=H#~npq?zC-Jimqlk#7Y(uNd$w@;H8|ypoc5bFa#<1rouWB7V1-JQ(g5n z8JRWf|B*a?p&0r=VG-MsUODniEr@gaY-8Q}1db_(mhAP=?WZ|x7yB+zrahp|Ls3=` z2+}^{N^zDmHjIMPIk8d9+T6;^bxrcmLXdAkpQ|Vh#diF!sVcTcm@?rK$T(wLOb`be z4aE>haUUr%vN7AnN2mgZ(BgA@NP88Dp91MSJXXm$ih|-VYrQP^GD4I|bdjri%8Gjb ztJBP3hL!Gm&vGx^|9oKLKojN|)+vkgg;2`}hA|tUEIK=Ug|4Kg$9nT1GBU(3RW1Qs zMz8_WzJf%hBuXS5!a#2dU+F8}_lYyL~bcFi$K|vacv@{#bzIqHF0msVR=1*w>eP?#X(0=HFIoc5O zdWKN&^V4c~9V!(z3%y4KH6Lk~F7TJ^O=xAQxjOqF|2Y?-cQZ1Ku&r&HH#sR=bZ{j{ z!420d`M&L$B-IWg4j}6w{pYuZsB?rUeJ%Xnt3>~!FV<2ZBhWnJPpLNX`;Q@h9dv4W z%LmtvYCaUHZm>(R)gkFK7b9-8so2dTX^S6gitLwv7T&d;s&uuXtI5y_=|#-43L`;; z&K1hM;!~qWU?gQUPy+UPpv=f3?+D22r<({~MC00sEF#k$+5=v%in`N$l|a}zx%Hs^SpcSW5o2bo@m3^)hP{X+TKEcB1Tos7sU&t23z|) z;`LNZY6rdW^O{E~l}DiT+|a!etF6sXDDaUqBu$e$msRUB zy&SXQh4u4$0tvk7@8HbZhEuv?Fgju#vJV_19^|VrwqohAD&(w2A-u-;qspSsmTDrJ zBfajezR<}b4Y0Toe2~NWfUa+L;?jkppb>4T8TRhd;CcfVx^lvi^x9?lgoQ0hy+2WE z;zwFq46Z#zd@tq^8>LULs=6NFVg~WJR9acdf~fhkgzVkc`*ZbXik)ym2}yEbKJ_UD z(YhOlRD2zo2n_=C)N3i z*vZAVR3Bs;j4K~+!YE;R5xZI~e({9pCk)ch|gG96Q!nHoMBE+H#-6 zD(O0Oz^pb0?Yf(3*@Qb1IGB`6c8SgZ22B!vAnfEd00d?sus@N$TTVPpcJJT-UoJ|g z!-^KBE76xf#{3htx4h9RU(8jw{1&%pFoVmpT@>jdg1|&5jWT&tm4nttK!;`X?nD(N zZr$@wo3Y#@bici8B*QU$m{=n2ak8_rxpqcy3ZD*^T}C{i*P$*ZG*${;Wu%iE(dQd_ zSo{mLl5BUUOzXt1{+;It;#wKy>bqg=RsUqeRv`w&l=DGzu zl5pRb7t803S8jEgrwYyTg z=E&j8KkN|kP%A~N(Z6EVn)EJq>oY?UKF1Q>MyVV-i`l)<*R z4~Im{9hT+^q1I<e=E0RK3Dqo(l%1^FO#%N34aE}5mDBFhm%5Jg z$&Rsgc^qKCX3&}vXab$l1>1v)nL)I@S=GYLv9>CDa)PV%<@^}8ao|vmoj$EQ1wJ+H ze6%6In&j^c`d6NG8eM4S$rpPT*DQ4MI`WqCv6JXDR=CP!;j^4d1g3#!x%0{~MQ{%< zG%RjofH=okE-tS(T(|SbEDTeUz?y#sUENCzb(G#aXd`#GXOrt*I!6D@|$|r2Nm^5JK45~38l9f z>+Ew?eO5QXGC#{6`#-tqTYBu?K!(mDYiFM#s1#8T_t3wrG=Pb5mMyI13qzt%RSh&U z-HAdj)+s4eHc&2yn^AUKww}!>>cdgfm!C_?nC-ho;0bh?m6bbIzp5q`U7BE^76v*0 z;2TYGa%rRtE@%6&eOrq*<(nK9K1i(|&$tA6^nOpIfie0wnQh5*c@0VnycP5>M&kJ{ z+Zxv=^i~~dEP&o@zdJ3_aP(J=E35PV^u}ltXS!Y%SWFA-e{(}n`1N)rm3)6h&z>?U zC2q~$#`=y9k;z>4K9i3u+~v3-aCgrp^PQg%!hZn`X^CP(L8y{Adu10zLuBZc6aRFa z-HC>#F;6f`JEPt+{PP*!UN=QLswFZ1_9;n+iP)?#>vpsR256y?^F{-3#aE@Qxp*87 z^i3nc%k_OG90^h+v2DXw&xn-eB3z!y;Jc5yQ1B|zr zMCW&23|PQf->+YsCjWP1APe!^XRdbON*?5d4wx#)L9vzvBE01KnZ4E_^QeM%=vL{D z2987Kcx)o3t3(5K?~?kMlW-QWDp_hfc20dG3BOENJ%u^0GvON477Q?42`ZFv9R8Gb zQ1>sPM-cq>({WaEE1}vO>^T)Ci!V_mlV1@&!nKhxgYp(h0t|M$LYs@Wi|sST59&E3 z4%=1FtYGI$q)Rao%7_*lqb8~gWDM!EH>BQU4FK>#W;aKss44%qq1-cCBiJ$lgaZju zg;@GF33v-;kI%WYtp%LW@u{|C@1=>*|3ZQAm5{AC!=-P3xJ_ASRXiZIIQtfaj7zZB z5wE|2dLScloa?GI!+f0+QnHwAY}bSq@fv4;RIxn|B#ZgqOzH)g9(BQHL(_A}uo@(ukqy zm2=7L!Di3!fjf6HsxnB`p^hKVfD7PXdQrExswCVG5r-lD=4`%8?!-2j;aVDnzk~8? zL-7BFLwRx8g4n{#^E)aTh0TtmgMl{bww1KwaLR+XycniV=Ko;GJunmUjVF#&K-flK z6^JOlR>7N;#m9Q(6|7z(Mk@BIU!a7GbWK^pm*)0Ceq*v$6rjvPCBD)ynFjvbn^4$C zNKS{F7+ zzZ9qUxA&ONfUj}4L8yT}=R1IdoZ<5p&2H~qUiO}j@xyxZCVXV$*xTyMT&EO<#l;g+ z2!a^zL1jxeXx6gy4amRcww*IHHV*?@j^c!|eqwo*)xYA4N0nA6ipdpxgxU{^CaIi@V1Xpd z%;^C>==t8$Bo5{gca_iZ#Z>*EqnvHJ@u}fyO1zm1jdRhXa)KE!KR44AdU#sXI$9|M{UCDdOu1!J^5qI z!xWLqux_aBn)N_19^VFqiTfy)ewIWGPPiI8A6ZIuVg4Kx2)7u|07UzMX< zhVKONG}}hmghIg4!usekChh`foI6-mVKueUl+UC?p86JZS`n!6%sR%aQX%~N;C*zuFzUB)SA%@?$iLyfS^6eESK5BurAn! zM;I!Oj>=2)3HkgpT7S%-KL3lirjEW5a~VHLV=#Ur&TwOLvXEWs(#?uS)X8nrx3qy- ztC8MkC_%|kBPsSw6l#@R5tShpD1%#V#uGYS>TnchCWpWhet!tiPT}96;1i_L(7Pg= zS(vqF%=)==8AD3kp&6meR}pQdY>+MfJ^lXAhJG^h!|g&; zWu`+I+QfWNM#f$g>&7u8S#4h9@DO+-N5c68!^%_gO+^2zj74DSP}O=3iv4L95x*G9 z+1*1c#N3716@2IjTh_PrAgJBe+)%Feq6v|b2wWo=*Z0njla`}c^qpg*htO(IsJ`QN z@__ibKX1)4HmReepQnO`_UN8Qtu0ufxG1^k4mJv1_trIe9H-*R0zVtQ!tb6_H<3V~ z)s+#lHp!1EjuKYnY5+~T;VNqzD*ngw>J-q}!Hy7jzn+;l)ewKm;YsPmme2^{486%l zFmIR~m=}3-pCI3g`U0#%9wV=JS3cljnSnF3Q*SS3hB+0QAvm$Y;;p)%!VmRg^OcxV z^2r#q9z)>LIG;S1B81qS!Iy`NT`(DU_Li#D@*+bH{4YE6IX}QV&n;>e!(J&+Ibb^5 zP+Uqy2;>56{^?cy2s}r5;S>Wbsw{8zA?(inkO1yIt(?`UBd=R;8pgzJqFWfsAK-EB zL+t;yZ4#t-VIz1i35FH$Ple3HNg;DkXzBFgJ>|(JcMi|d)l%bP(CJT@o~_VdX84%Z zLPnQ~rqKr6jJ({{nRnY3FWa9=?zNiZTg04aQP`A zL=YVUkNx2e39p=Fu-ZnMk4&Y<&VL}%i2|$0Ub-?9yf>#IyHQNgE}dlxD11$a^3f?` zG)A~JkrWG0f$*8vK~m$(xT)ui&dKfgJ@{YY+Hw8s9X*SaBK+9`F}TYw+I?$)%^@X; zlM2bMG-;$SWXs7MqfA8s*5KHYMGhjZvW#R+d<}1{C zzlpRFW{JA{jBvx=NKVUU`eF-Gz;{+KIcWve*X4Q0HFgiS?Py_YhA4e!NYXFP^je^^kUm&3^P&PFiNufhx0Ry#hZDJ4k2gOjYmjC%T5^4N&sAh z26>o^I159`5+J}%)7N{g?ZUBxy9>ucwF;w12R3XdBCrY%p=iXFT#34IdGZAsgRr9G zhK{67je}9hrJrIeXtBX9fLdwUA>u8Y-%H2nexR2_*FYukL>Y~GpZ*_OcRT)kb7xso znasu=`49M?zZ7Zv|7{2lK+?^|95;T>Y6x|V&$p@DhBcYOOEOojel13{DqY_nt;VQF zljGw~+*47YKy`bDJ%zh=zO*b#o};gPM)#W>#p!V1Mt z2KW?=VK#EqaU#?^Iy;V?gueQ9H|HEEsap3|Q;z}av3f3cgtzVeLJq$^E^^D?Fo ztGU=J5R>n%`JMmxwKU1(t9R}rjg~llE_=h-6gXP-Dg@rzgVCfL6rinXclhor!F+Z?@{cS z4cG5gnedBoaS2!ctDZt>3CHTWJ71!_ye$uP)YNOFqRKo`e3BRKbNR{DNEsF;Kp-(! z{N$*TqY&NHm%B7Lo;fy{h>5Q0mSN`{z8dHvSDyZ&S>u?mul3siYSZF4G1i03&QZ;K z!fO2!TZ3fepUT!HcL<>)AkI)F_L^^Wk4Ve>y|W*Rv}LMNd8b`Xk}UrneCfVR#c{{P zy)5zKG0?bBdB`cdvs>?Dr>X4GYdnYKnMXVzuyjUKq|#%-V4-|6F!WvKWTa=Q1=kz5 z1Fl1Myg(4wWx2o#h3H0<;yp!-2@|YVU@?I)MqNF`Md#H+QfecmK_$B<^$moRBU{+y zZiy<9cOSv6bHH;9+VO(7ut8EtP1R?hkXpAozvj+SL~PxXJ(d9wwI~gtMj<+G zfuD1o{IF&M1G4u3vPI_`2jDN!$3tw?=J9 ze|;Wu_R971El&xzK+sSrgN~*9#8B#!*1`h~|3*)pBzE9)+!GDRJZ!!vxq2^meqzqq+LN>(L~zxK}`zKU7^6)(ZcTuo`6A zV6?Mw_nfQ!t}t5kyBU4_%As72BEw}M=Xf=&HI4fPqYVAS<@fZ^iS6*=KejKJkTJOb zeT|y~#F+TKU^3`q83kl3SqbO*+~r z_zvRoCh!Ywigx>Ek<_rpa4B$9w(kwl35XCA@_qdocGnc=h1F+>wvl!NV>oXISH% zNMup;YS71Aas@8dgdV~cy7QtLkhUp~H@_`)$n>cJ==vJ|GZ_LuRlsr>M&jbmU{T=6 zDG7unTt5_<;|aX&Xj|GvJ=GoYFXOvT4&MfUmW*KsTt2VIc~chCUU_`tj-Q6cQ!icB z6TI=}&_Q3&r2Q4T13sI}7^cgI8IHZU%byUs#%|S9!HDc@78^i=K>=+b{G@ZHKT?Y@ zxr9;@_u%0A8RqV8)oTOAzD6QEv{&kqcVXvW8=qbCP3Q&6G{s9dK&uXA2|Z0FefI>8`c;E$ zvAo&T9)!?IvoeOvY#%Q*1mHiboY4H*1?Y^INRLkZt;jSwmQ zskx0obXdq5{)e%5=+cB~pe@t3ZB^Pr#t ztnu#jVW#NA8uqK*E=Gq+RZ*KGO3+ojD?3voxA*SAIhS}xbm8_mM9N2rKH zO9kNd%LzzZtU}|oDThj$$&PwRB-7M=Q35L4QY=e{t5%)n=RBOA<3S_;(Rn14D@it! zFz+9DnT+yT)cN>q$|)3DWCvmEb1~OnhpniIsTy-iu1Yup$D9ES?OkYDymShnIYRtS z4*#<~PN-gm^6s!3OUHvO(C=+j@bAOYPm^~h46|${VFPu4YdtL{Tm^Wpkv?XJq3$6n zp80G}p>#%TLcfwXh8uRm>k%_gAYdbNJub>1u3m@0=3PmWWn=*=0lrdX;7h@H%{oB>f}Z)u z%YNU8Y9Jz8*c=rHRTNleMJCj=YT_1;t+tcY_)p+8THf50qZ78>E51;_5bVUSJLN~q z>&7;2;(Sy3_vGntkh;3g2R82>n?xD(80Wtm+=O+m6(TMzCQ`+I1(e%#JHPza0(WIP zCy18VUzn31){|Gz6XxJBUX@&CUH8V1^tm9z#&4?S^y?UNrRW<+5$cBulZd(tg2v}# zd*!tofWWGW`r$oBcz|Mn=#}&z-Xu%E-ml9kCv0Pz7vsGMhfGgt1r~O35;n~b1H9R< z(ID5MJD6oBZPjx;;_Zr36aBa~T7w|P0?zL^jRn?v2O;_53lk^1I@3KjwC0;b3P$}9 zgK{+bz72ssLk4MN<>V4E^S`fX<6bny^?nsdfz+Kny}z*U7NZ5PoAMLOko~jFU<_rY zWHK)P9djQQ^D|Xx+=TO~Tm#Q@cG+4{#kaGia^Ad{wO{Ypp$K&vNxzPYozG($uELDm zl%GQDk=4b&udu)OLle&rrWO*tJt?0ugW`Z+-xSw$MYgk#BH!2eS%x z9mYV*grA?jDxR$PeQc}jUv=4Rn6SE=ZVI~ELtBNWQS1gX_$?6>WRlL=c@@<&c`VL% zln`hu>1lEKqa^B&h?5gnJ+>{7kKsLbbP;Kc5GELQLG7JaiVOVVjLH(+eUk zFP^?eQ%kH+@2&)4y0nWgE|ICVS5bZ#Qf8Zfa0UmhX9@2vD(rTIBt*j}5}jpGJ1Vs5 zC*WuAAT^BS8r-6zFTnS=bd-tU5*p?X*+?LiS7M{kQ|F>+fOH?NtY1*hC9X^)QK6SM zD*V(Qv(MI^j^-7;(@6TC=liLwuV7yv&;puM(mUOT2!LO|ayZ zG7^~Im4H#x8q9pcCnMiuTapeo%ZWWn6#4G75vKeEi>JLqo((&u538M*xr2?m*5xiF z`T(@7H78SqTPm=m)1~dy1YZX1KVhS~A4@P4<6VaRo;gl}H%mA*`}BUqa(*DCw01rr z@koqn$ywr?E+;GzgQ1$B+q|yl1rSNyP1N5_3HdxB7U9ojzFwusClUcq!rMBSLNW(L zgNc$umSrPR*XfjN69VcL8qdTr8$>@}DmUeIDOjG{%C(7MaHn5*+@m1FqNEgVtr%C# zj1-9}z086vsZozS8$G(nKd-krXJ@3aqHEVjfLC72f%kfH<-csC!dhz=aSm*HSC_`7 zmE39YoCVKX&b@6F%JnPl@|9a)?U&V#eK><<5F?+%q56%;UCkY!zHv0jr%C5ojlrwV z(SiKE_geMq%WTZ^)JsLBbPROq1-9p=oQKsGtDY!=D(V^W)oIdjBQvBjVz{-M!yeZ>2hU9bnjsnB{ zXu36-rp@1NA=DgWp0G3o;}VL62BW{k$LM#Tz+2ce1`bub_3+Tt*MIaB)PJfTU>Wnd zzFbZnf-Y&->WR3TTUYQb-2@;uy)xvf*j6D;s9BH?Mj}8Y53rzhNHDQnnR)+YG!Col z<|#bf;w2$GT{-!66I_6{2zQPT)k^>AK_YnLt-N$XJLRIoayo8>W&wCpa@`kh7bO5v zDzB2Lb+`@w6@ZI;Re*i9^-7UGMiH(BlM;|EuAGIOYZ{HI*|!9_BZsdm_9iXSHW#$J z<+VAgm}Q+kX|z=Lr1m6La)g!cZ;VH-tV#siZXJ3F)cUWh9V@D4Lyu=7~wXf6unN99=9iat`P3bZtr0}XEN)TAcTR_(bsB}H*Dd7GaQhWhgTwv zLw}q|`~#RQ%gPmaBHhdrBy`@>e9&)%+UvXys|VIszvbEgCy`mbg{ys1Q0Gbz$J>Y$ zgUG4!)P`s&pWthI&Q33NFko%M_qWWq9yZag)f+D-c-N4dY0fT*RIDM?OMDB93{5(L zX790m=?WvFH;R$JIwAP)0V@@e;c=F#i&H|Fv6$FKi#W@~H+>@*P5M{9P)-ZuZ9AP*1Jst-<&w0&EM!AjzUd{>+|sleiA4h`CHig4GD?MXr7VbrbmO0&a=GcC7Mxc znXX#L?FHNB=3-9hjsIR8;N+rH6XVIBt371lVL>LRk7oPlLP?jt&6wl|puA9l*x3oG z```R0#H`5Wo_}T(P_wpZ_AC;64|65goy`{4Y4VHJQl^z3Qkn9%-YbRc3fvXN-OG1g z{OVY}en~FP!e6coCXzz|<`pcFJFz`tNC0t?%xGl|M;sc3H=lgsq4p*^FI-TO;a!?btFBo;ud^VjwtO=&40 zsyy+vhc>f>bd*NVDCS_`vO=H+O&z}p54`&;j8&X!o)Ct~%2MZHrr6|_2YU(BocuYF zx-m$RxbZd=G}NpA@~4*hNuDu@sVrFZ^tnhIc(Vk8DS8lUY7BFp6C_^6$|`M_w)0_C`P$-lWiTeU{k7k-8QU zlW2t$^vcuc)puci8NZ*k%WsfZOo47!HJ~-+z%D|1sNE9-g1f+O84T`WXUr#PevM0d zU!=LtQ$EUsDsPOxr6z=PN}`czitxy|(>36}4_YzmYG9pLT~QA1EYZ;cth+smFo%v2 zR>4Y8{QR~@D(mFzI?Mf?HaGp$xPs$-o0+t0e^3Mz$F%CG4P@jqVJ}suO)F$JJ}!@Z za7+(#S>Hplw9-c7r2jg<`K{j`GU*lJK9)5=?>q~zt8;#1Zx~Fn1jF%8DDB$#g&tT( zyw=`~#ri#qh$kweH$j?fw1SzC`YOZ77DlIhGE$;NL@@@Cr*_72!rx9x3oZ@HU>&io z-_lPH`W2JziY&zV?d!XXB@cgIPiOF|EiCN9k}~on!u2bWh0*%lh}RZ0Sog>iV^t-G z6RPU4blppX$eLel7AVV9!`P4L`4pYevKj>tohSEvu2%>1xJp2-sIsR>9%b@T>OeQF z%T*C@pt>n%Qr-v8NFc*CPoB&iQob6qLn&$k$|bx>Tpt<1CD^CMr`w@JLmC!!=qU)j zJWo50l(q{dQ6@dSH{ulG^+-jmW^Z)(!XS^lT}4w>(yGgzkTsC8>IEOh**LW4Kbg~5 zH!|mKtbVz+R}s3>H3ETeqGWE_jk!PsSszjgj?v51>n8R6N z7q935WTE6lw+CWU=XR{z=KLhY{ro#kFKPJvpA~=aexepC9haU)l^B13!Rm#j(8pv* z>^rV5sKnVkzXc=pr0`mZHFe4CY@j;~S^3F{WQ~II$6eiNGC_z7bSkp%=k4yqQPeaB zS=PXQD-ur>KWZ&m-HRMe=fr2e&b2|d^{HUUVmna;k8u}e1{l*jMrEGOvmmXtW= zBx{U4gr~ai_mvhtPA&B`Pi7wSG?zZm6x_A3kmd`w89isxr0v-66UkO_3KDEVZ(K$Q zqnA#NRZ~eLTTB$G7g0}5eg`%p-}jP2=oB@>WP*u{ORo)873_sE=4D1(Ymn4Wy2Ms0 zZNHQ6%)$Ay`u_y@#*FR}#cctCp z_><(exe?DMf3tS&LgGwMwl9fz$j9QaaroHa2M!*TZ%-GlCdE6QGOM+ZkE-u`F(NLC z56mDb*h8=z&YjN{X|EgygL~!bd zZkMgoO+v`?Sgc;APrZmtAN~0)_M7@euXRXX{uy4n()S)(&C>A#+g;zN% ze}(Kny8P+qXvHpxG|>iLLs4r=xNw`~YDv|I$RVV^S^mFs7ESObH?iPNd=#oVLFm7x z!U{-;WwI|_iF=d=Kji;YCR_Jv97<`U=`}Lu$Wz|UHwhAvJ7{%$2=w<>39nT`` zFN`l}-8qK&#E-@y{ukr4?%wRp6cb-CL3q&z=T#8*vSi8A8_4eO_luPA*DIa)djTKe zO<4X5QtVqJaXMvzqTH#*S7=vafxnJb$55q0I)}~_fKjHLe_XKhn<<#g=B%F6D_)uv z;n##Ec5<$ThQ{%AU>-b8ICwUPbcdj+xY9j%y|IpuXsYmo zA>^Ald5qg)F3(J%yk7M)B-S0t@rvMkR^9y3>l3n7r4nlRt0V$OtY*{T%L`CrcZhR)lmmSz1B}0IaZeYYL>_S%`j8#9*4yR~EVw=baxC zGXkU0aK@HVVwmrc9QQ}(C8kj08D9DM8!oN8xvh&3Ng?r07tXsKQ4%LUrmMB9M}M)=_rILTyS><`>F-wSG8$VSjpOj*D$U9LSGzLdkzP^!qO%-ttu(#ch`B#f zRp)USvU3qu&yte@L^Z9pTj4zsYPHoOJuib6w%<-NNHj>)snNfUDCDtzdLBf9D<4`m z60%-GwRG5zLFHq1mR64qEd=oT576IeEu1NMaontUoSYGz5kO+nY1NGdMv(I2TuEGb1}V z&y7}hH@!u>Pfn=}smN5`QXA&3?(QV>fk04vW{D7Xnm7`gy3kbD3fG9>)KHS`?fc+Y z^j8<)kH65T?QWi5?x|bus)S-m(KMsgu_b7dODOsJZ(#+}!0769as&iur)D~`K_n$v z5bVU16zGh613XYr@}hW0TxK*fh&{;-uqB2hk0KZlZ0ZUUMgd_Wun3ZySPc+(#XwpJ z*p++N;6M<$J)z+668Pk8uq$Va@BzC*N0>+b$Uh}1)L_;oF+H<`(=+}B=Yl$LFiK#$ z8+={eYoefr9hkzf|9RpndAfFg6@gGmau0wR!@D;^aqED)k}!dGZ8Ym_fOs*u=_A^i z@97)cX;M>54)-@o*<7CeYXLQ-Savl9MG@c(?5S^?`K~~M@Q>IHM=UINvFLAbIrSv2 zr>!BcGASg5Kj^px1p({$jlwwh*$f;Qb&No`16X<#t-wgG6@Y=jsswCkRc1fT0;l)NnU?D z#|PjVd6e|PcP)|w!LRI5#y@kB0y@FGI{+F#g%lAGJlyU-Hf?{} z?PMi8;FloJA2cOk2JkLGIwfzN1I(4)i1VsTvx{?U+mO2d5#H}zRmXcQ27xCLjSKC( zzEhZ~g@SK0&CjC z|JK#sQac=93>U#i@OFz}nEf34Rs8*Kn1Kq0a0DZn0f~$#V;J-66teBq47pkMFu8kn zcXkES3L(@Z=~J>9-v(dZguH?RB|)u60o?ZE|7iY&ya%r>Odsbf-erKlDY%Muw2=(l z16=S?06D*7z~ln3(%$W5>j?Id@Bw*uj{i8x66gotjg@u=fiJvlHO*+NaC@7dA^gReq z=X=Lb^Vv>A+7MyD?G4GSJ8>1V`se0KVa<9`*kFB;~&o3i{KyVljv7IoG zT)m_|5F1H|oA0Uz7jSNjTlE0xU=XBdpcIJ$N+9?H0CgC4FAyl-)57~Be+1^p{Vjh4 z>PYfK^#ID97%&XHRyp+F1fpa;{)c8>;(d|ZkfX-Gp!iiTZz~S$>ZuJ%K`aeLKE{U? z@_lb#?(WL{RDgU3V4pe6tEj{i?OgHqfPuI(4t6^^!QX9j4P1T-HKhk^OZ~TKeB=AW zunT|&ff)ujj^g9EvHV3~o1sRS4V2-uDFq2JKcWS;&-f%sIlCtbDIfYWAm;t60HGrd%2)xUYdJFTo8J13cFPj zo%7+tXg%S&a^es6fv-&G`S4r$Nb8wI5VJu`o&x)u+V-LQFB&03zd+*{eAcUl5jMd1 zDFq;jFs77_LDuIiYkM|L?vF>HB!HHE{)7bX?e{FFw*AAak^qY_e4f7sGkTE=W@%5+ zg^3Q;6BHv8iZCgg-3DceJ-V^$KG4=sX%QHrO-?i;O`W#hdQUYX<2v+kZhpVM>wTv? zAWkVRE&1(?r&gcKsxm7EfEI#|kqP3_15-LyA(sEzV`zye&DlEIu zQJ2_;o_JS8Lz1Wr>1p`|s=0Zh6h0tSbNPbdwH08;@C#z7J??$lavs&}-R(r)Z%|LS9R6gJ6{@Z+ z9Ko}g2eVA_ACq)T;*VqG_^b$Tmt+cHe9IKWul!B|ZJl8Xrw41cI9~m{)(AODuzWA6 zOf|{SI~JQFde(P7K8`iH{dS_MuKDxqm_T&ld+B^m_$Qb2!;>|SG*qdxJ*ljg62pRB zGfk(fi2f?N-+6~1A#$HPZcteX*cetIxt{~44}NovQL}0I*dmFkX<4tcDdlQ%)mWhb z;9E;0VkzXe8MPskLI+0MaBg>u<=V-?lcByX1+N>2vQs_`=_^*yX{%Fg+4Ub4OMNY4 zXQ1(I&P6^`F)}~tsx*9#U}MPWpxSX%(YlJ`wi4$1>hvZXDW0GTgq$e8Clt&Ls*zFl zfBFtx$CmF*uIRvT=LuUFrRwb1JHh+>PNH)`PaP-nUb=LZ zscTuS@p#=LjX8*J-XZLh3O%8Y9B=-DJD#E;ar@?=O>9ZF6JwrNq~s&PJwiQ}5Ra=O zWGtF1Ke-h3-B8AC;*PYXpAx_5UEjZQ-7{D3aV?uDKmnBVcY|uxw04t=5`iv2l$HVW z?Wj@=vfN`KfShDh&((%5iw^IvKf!lbLk}{}JYn=MHW{IoVeVmMG41sLH%YsT#2d?E zWBdTQT36K($K4uRghynQ97{9=qFu>zq=YI_3g!DCzD=uMA$-n*CFk@;Ut&wmK=gAt za)_~1PekC@sxM9(&ij(~QI;fh!|?TEi^V@3!Q$SE@{(sl6Jm-Hj~eIF+9fK6r;Z7-3&kh><=GpY23TIXK;ez3 zI%OH-Msj#MP9Z36uVTkXvLi0>j`A!P!RxTGA4iv-| z?2|MJmqOSQ-_>fbh}XoTNv)zsJeA__hD)7X@;=G@z|YBl*jc6M1trHgQz#v zN|`~YN|WBo%OI}nqj#B7&h?+a=XuGsE-rw%F$JBk>0Xq38~DWdt5N}^SqWryDj$fN z8H?9Y4o+vq+x{t4u=oILtVih^)8>@t?5ojfcv2kEI`3+Y^FgF>6mH|n_Y+Nv6*hw` zl-HVUsOq>s*o|k-7aJ9FqFbT2L+ZP+1DIzK47y>I;g%$hKT|aqB;COcf~#r*!c;9) z7!d|Qn&XriH*5J&C!x5>wC0gex6U8M!s4&0a0!U$%`cPD649a|z!f&-XJdNM={@Ci z79BKRLie%ROxTHuzj&qC6y@N!k3wO$SBOA{AGDq(t+(K+e-IyxR*MO?@?}o#F7H9i zx&y{Jbq6|=BzGKN$#3C#ZenN>ybQ4f#B2$SIw2D|{cEt4IftOBFXDS=+tVe*&3-nH z{CbOF07tt+Mo$?CD5!z;(}`Y@9R{LLt~_z+=ijJ@T>;wZnqRu}=fBWmnvt0LjHHox zqHJ4K%a9M(Fp}FZW~R)708?H&FW6cp^}P&$VqdaUblq+EQcdMUR&RZ=zx6383Hok{ zBczE^H2dfFH%nk@TQjTCcX6;Ko0*K8hJalMo%oY^ZkWHAB8pkJIu2cC@k+uxUK8-2 zgiu;PdM3WnptJ7MdWhms1}@0BIBhA7Ubl4v5A8-&Fe4rqh{Lu@Za#rK&_eDzFPagJ zbsm%mmxvJN(eMH?bp6@@tPP7*7j~Re{{j=Ur+{C$ed2_r)?!o?p<@mK-ql?_*%`*L3B8Y#GsV`A711*~bH#2Z=q)DK-;_!Pc z8Cx+Wk_hg*61wP_7_F7_F*Y>f6a>_#|1}jImRhHR*6wwykLB^Ymd^1MR1HhpiMhSq zp*t8EDt+QX*=`s!Mb&TC(h87TKKBvOI%R~te&O2{H(P5GZ#*P;vP)Iz=*73xH`8GL zSFP!*`5yt{@YY16q57e}u*0ihAWbRA<7-TFsJe`1_H3$Zp&?{Yo~a%V)8)ZMxbg2^ zn|p=RtG}i(b-|G|ac`0-9eI=2n=?;VPYDkh*-SkMvfXPRVtA;qrqVCe1$Ws2Z`>#f zco8!JUwdr5>%65LdqD0b=FL$vPey!1o=Ys)4?B8Gdf$U%FAa_1DN-evtudBt&=L11 zH{!uW)LEN6pI&}@qSvPvws~Z=&UPJYR{EE>dK6(dRWUqbKfVoW-}v1*WH@$8^j#E# zlua>{hyJ6pY9|eA;eDz0e!XNPkm~soa`g?D?NcY2PnMmhjL(Ar$p3ndw@i&Q`*tHzn*{I zYDwnTi8P>-TZX*p1HTZBUY+3pz=st>a}xX>zYAcw!%;&Kzgnd`%Hh-n-2Lqbt8 z0@Lwyagi*$rHyfTlwj>Y<7p%AuXlC(u|MDj`ho8Y?6%rFDrOfCs!sf>tm*+`^VTWW zTB&}0s^&7usOaC?zERa&fDuWnvH<2F*NG>)cCV_qlZWP1Wq<^Szu9b0;<9L2Vra0; zElgq7$KnE%dylCg8=q-5&OwbZh`#WlS*sR{$MZ#O@@)GtUp59PdV68wMkEw4Cm8xKUOPTH{^1 zqJUOX2%)*TaCzK7wvLC$c4RNhRC(>zS(n;_R6`u-H}{^udgYCnC@W3&falO&DQ?5) z>Dl45##`@XV0lu8Px2v1^m~YcgEN9ySF7f`kM9L6DU-d5pvnF9oa(up1*Fau+#P%sJ@+G37p#W#J7RMwQ3f|{K)c@ z4ez!MEPX$Q#9%Rch7{QL41U59WBgP=Td~3XINyUhIgLq5f zMT|4O#X;XP_fG{=$pgWHd`r*>LLJYGWYV8$)in)-PY_tz7wrBV$M>Z)Z-?4GsP#iv zQZsekT_46)R^-Ie2uAkS+n5aQk|z=K_qGg{Ozanqq(_;{%m!YKt#i9IQaE?3p(pQ{ z{j2WfuTmx`NtAS&QK8)A%dnveNnS0S1F_32^}ZZ9F2|$DK4^qyoVXW|>e1bp0~|E& zTrmq5C}jHKGi4-e-a&fWU@XD4t3K zPhmN{)v8_DRM#zhERJo&RYz6V{2NeStG+Xwf*c2TqK#J z4|ItIAqsLQpc%j0EUB}2%TIU(rIW?fqoKh4q6|89(E(om20p4wau!c!fg1+rFqgbV z4nxVYsW z{oIT!coqMCST1do8pY2#uEy=Je`_0d(@mFuDx6TTQPSC9E-X=shgJLvp&ZYk@IU<% z;@}Ok{9s*jc)-y(LoJ8Wl~YAaV5Q`M;EOjw71Nm~^dYwR=WaNk^s5DS+5Dx7@FwBC zpi}o&Yv~=Gj2>4k(*I1aNQRp&mMV&=IS#9ek8@99iASdeZ#l!;VcXu3P ztNp7!(3PAY7o)^NmP2BWPExo!@%L(}EM^Lja&sQ3V z3T@9~bV+^dyMC2T<@rAfG)3#n@FOfvamK?m=P2m!@!6>8<6%|DC#i3Ta%$9<)0l|H zynREjBM0zT*VcUzC0~%UM{MMXl>ZuIYE@5-EsLtqp1g!&9C*CIhP*eK$UFsgFQ>fY z`o(nwcYURbdDbp@7S}O(SGw)%nq2pvZobq>R>H<_PdvY&Ewr6R0XxQmj@Vz-m%vTw z0HlVNEjICq9kTv*qqlcWi9!BXjNXFzzi*ZL=o@nXP=xyC0D=vGzhn$wG)e@#>x*Uu zZAsR8F`S8f_(W)*^VNMi48N^nKT2&?h)1oFVm4S~i9b+VCxscBN$BochaqfEt1U$~ z+I|6w>|R0T88EQrlY5(#eJiy7Y{R7#%CV`$z7o10Nyy^%m(fn*sfl-cxunz3mTP{K z#jb-tlA4~ls^94n6#S^ejh(TPz01p?&-4`9EO{VoN3v-eezgF}n6x#c%l&wm-hTG=I4z~odOf5e9i6SY26K+hACRc$G(U<`DA+5Q}c;qv(O8z`l zkyP&W@m+@`Y_R+MN+|;;GysR2uUCxP(HDcyiqXqz{~+gl5AHS$nQ5<)Ou{VWJ>6Yi z>)1?_NLB4(EkeHE6>M(2Un^h6V_(L^pp>Gug=+3`6}q{Bt+l%6hsrFY7P?F~{a{2# zLARdRxAb7nVGk}AVrqS3G<(Oka#>xgWfLU64NdA#;zbtKTm?hP5p!&m2D7i(aBYNz z{RR;TyixzM`M4Zb^aLD_1Yz!#&)pEK5YS%aqS^!)2r1_@`r8I+BRqc%t;e;{Own3IahqI9g^f`*!bSZlMNdD7(bmbRrr3Ry>D^@Ntu)% zOzQcXP6K_9CpiXbTpy=T(+UCeI9ARrD&l7A+5AkF?o;<37)@iMA~;8qMUHbpE=srp zzU#d4e74DKec%!3RS%%LFkZbLi8f`k)@{|y_VbgLj%@aXHs{zQ$vh5VJ@{=K+06ow z@1Rpbbl6=jI9$Zv=`!?cf33KDJgHSKt0HS&`wG6n%#a*a@eAh&0BE(faff@mC+JGS zfAyPmto*T0My|7kt9N{K%vFs-8xTMZUva1~6#p$B0UIj=M^12d8-G-M#7RP+T}K{7 zaFn&k;0x8PIZ67YwDIUz!GqXWu9nwnr3EqL;pMi`sRex?HoK3@#tyh-iMuobZN!pg zAo^$XbB&YPcof(l*?X=*XGL_*BR2HCH(64RHu zDxE6wVTA>G6P}(VysKCwP$h-*{YRxh%*!^aG3;Gk>a*y{p8hRDHg@lFKzjKf__e3@ zcYN4HU+X)Bp_Xr=1t3p}4GImn0(e&uKyM=)Tj;04|EGedJ%65|sCj_8!>pxct4~cBqt6$5^C6Dcd z7fb!QThu_l9D6)P{$&|6dp=f1G+?z#7PgN|$N1RcE@$>vPRUn@8pw)v5XR zGUZl+uz?iAP9z75?ZU0Y)`Hq1(23L@{8HzxTf@P;&+Ak%Yk>WBh@q7)HdrM|{u&%7 zYwcB1QR#5{J4thpZdu!AvK_Ek+^Mg>vVHf=MfLH%+oUQuC0xz*DXXw`L*xX%SYiDm zCWPKFjW4(}P`^Bf{%7EFkR?n)G^$(N<%v7I)Sen5t|bn7FO`XLF}2v{xY1<=@%&K8 zr&Uuz8ZWdj84(>f?-(xAGxlig^g5HF3R66j#n2a@La3Kzk}_6Vi(S*h!_Jn0n0KjA z>N60|ReE(+j-^8PDS`UR?xpgNnzIdPqTmW_PSCJL;p}2gOx66EGa;*OgjaK}-)pkX zn3TDEmv1b%h{cp-oakTrZDP#^Lflm404`H}ws+i;VB}oi4)n=sgOw53E&M%sj17dUp&?d8+==Mnv!EbIH72dSH(AyZ)$ zjgV>!Tdj%kKdwl*3$OU@8<-#mXGc(+%#DqN+J_MG17>3@)|zX1YK94>R(9IhH7-J( z-i=EGy|g3I}Fw}E#RBCiPO?iIc1@eTE$7fu7gJX$+c2ACwmO4*yq`kqJD(6 zOr+b&c_(ng9G}KK`9;FYCQ_?WI<${xpF;G1>< zR_zZqDpI;j#1sb4l%T~gioHK!(da;%9uMQVhw=CkSG9326KXuai`4{J$R@@0AUPI7 zJY}4OTdt%S#Mn7PSMWKL056`E__Rd>rAc(>3iFrtPZQ?xcCp5&k(112ZJ)`)oJNkc zw2ZgP~h9dr3fH^=m!=t@InF!PEE%LL8&09j_zTWK%5bzHJ$GECR&jZaa2 z-FI?-Ee7p4_b#m3Ea0Wd{$LZ2(E(qWjdQrTrvG7ja%m9P_^x zOdgEFl*stx_j~&J83ecesb{^QH~x*1wSQV5Mh=o!8S{n+{`{Mj*V6(0AoPbItYdqW z(WOoeR`~+vH|AER1+g(fQE9MUFkw__d0)!K;)NI=euD7V9tg`vJHmM>ah0o1OalFXqBE^c0f`_bzM*h^am z$!@b3ar;}J7)Wnou`V-=Ix9|nE`8&YM3;y@Q?Sl89z+)GsOuoKv^OUHoqdDY8Q0^8CM~NTC zn*6-;v&OflQSRrfNjjoGSQ$*xx7!xwlsxp$kmHK{|0oTOI*xG5zG=m( zABWat9won9J>d_)>v{6fL+BslioI)}jeD8MYMfVkW@}^;#u586`)71;FogRzW`(vT zkzvjdT7&mx``CE)7d{7PjWXJ$U`mNH?(yC#S&YR11=*ZKMtkUGu?0I%bT**Kl=`GB zT{&$w_IKXPzt7VyNsRcuv7Vpn=RVk2Nuv#Y_DRtb8K$MOi3Zz4ywtX*Z$_Q!rr*h&aJsvo(WXH{uQITxA#NdkkgxRW?=l;x>kSrR85O2_ zEuhTcXxsl^2bCm%z~ZB9B*$_gWoT$A^SRK*QcjgqE(-(%@vOa-uR~c`q;Mni*Mwr-%R03g6uvR zj#LGmaGjGz`h-bBfVBLaHsv)O0%CSw?ZX~rz>r5yK~EJwnK<4W&5Ll=eXj8_m(nGo6%q3+P)*ae)Abz_OdKy zpY@tqZ&>N*r<`5g?K%@V@i)^ zVtw`%bO^sh!akz73to6@wJtn{X?KG+rl-fV*o!gX0btPwT%cs%%uh#PUbw+6LwHUv}HgkxFR^dbpdwj%AxH2rE zEf8Shij7vz34m|v`MCQ6w5xLR_8t-GyME+P#aUtZo*SyfDw@|!5s@n+{A37(&0~cs z+>{SEUf+O+DQqG8FrDT$uU&aZ9!@RCTkqSDU|Q;3FU@uCd%E0JkY@y1;z?2j8w)6a zhZqbQItkX1I^03cn4HILjnxtH_!WQDW#SLcnN3=Q%sTCuQeM`4i9f?7{(34LyM_zu zT#q7zVW*qqAZvm_uAVXCKl}Uvf{+xyzyFUG8rT15p|LV^aR0x(GgcB-b`Gxpuax$G zudlOmaIzcp43Y=Rah zI2-B}jabxCAMJlPgsn%NmbHMQSI@JtvS9k`=jAa+2N z2#igNO3TYh!Gtk1wSh}CO0|HXS?lbD{|#czryIyJ_jh~)!O#Lq3EUcVNqt0J4K$fH z`a-6fJ@z(8UH|I(ngAFPQ(2weMIj(8r}bN&0A#5U`u_~CzpH_7;DHlPpM%0#-+l;r z?H~d>CSz(~Ypm#^TA07J!2==p;9On7zNJ3_TQs{_pg-j=H`XVX7LS@>Agy67EdosF zDLFa#C7m%TY$bUmJSDx)fUhNbVFeH_K1sTP&t+6g#IG7=#-ITL_~l9DGd4t0D5 z*V4w+=E45V_UP&Wg857KL2hC#r7j6OVpxPS2`IfA>L+YI@Cx6Pk&;A*vUha`_H_SIaUg`s50Cxd zqWZzt&dIGL8pF; z5AZbgTaytM+r|cx9G~PlK>ewII@VTrg$QW&2H?yd5HhU-B0D6eK#UVwRzo;*HoetE z0?}mo@Oxi-8|ykb?)+qb(5iquGAp+NpMoC30Z>?hL18J^{jRsymd0lAuFhbNEk9*o zec1c`Z+)6S$hsgY9g#h;)iB!uTvGszgElJnauQt^5ISh;f${nIaq#^2UU)4YZV&_d z+iP1mS741ih+J%&`^+sPh!3^}abQ*+`By>R(LNY6?lH_;{a{5Up zy9b4Wn|fT@KB0XpOo1LRuaukp*i$#Rw2l1@13MeX#P`}elt}-b{Q@=-qLuOXZxEW%X2*K=^P16 zuW5@xWv}9IAQlkep9mk%F9W~bQjkq!hG;|j%T+)ffUd$`E-kDr&iB|4HgP0u?7s+p z8Jz7X<9o78$_mNM`Jq*I&4O*?U^5c*;!v`fpK!asM4Nhdu0C1%`QL`dW8qfxiFP&% zVcq<8hVnfhY=Po-Hb2f3Jes!byv#Q-s$PA`YhvJauZexann*yA-WC zVxz`XZ~?1znX*f}L<=7JdSgZ*TL+&ZFLBA2D56)BdWT?Y%4pi+>sK()Uy8g1A}e4TB7^#ZqZg zu}E>_@_`~eO<(&f17~AXj5ccBZv%HUnGZMyz6pFN z(G|9TI|(?b@9VKR*pIbhOy~PH`d!(ygeHvT$K_-?tSOA>+iy@7Jgy}_+JV<E)3QSyeMK3*qXCAhgqvDb&w!6){bt0p5r%0vm8#JeJc zrSe*Ic}5A~e8&qjL3Csp{Vke^84Oy*g@y@f>=&+2Z`|9Jw{G5C=D>QhdAtAba{?C4 z!{1zxo8kn6r9290WBr+}R%uH6N{phoVi`Od3(6W)P96E?$?b;Rzxt9h(9ALPoN;6t zd1TPiMk=6Q{1#gQEeeo`CL{Qnr2ZZ1j1!PsqFQkVH9;0Xx(26^w897uu-jlQgkF}#*8}F=1BXyFD=zX^qdu#iBD@n2iUafHr^C2AIHWLmbto~Ux9O1KTU3KVz0DO3WHLTxkc$OA8^s5+oO zsHRWEiX(!4sgfJ{vDa~y;tAk(5S~EC2_3pc4kuy?iLZympn@%y3o1Sy$HHZLZ9@{wf50T0Lz?bj)km;$$X8NtY zBNMh=A`+Aj<%mM#x8?^89eQ$9mUj;aA&emByOAdR`&6iNqPh<;a`$BY5{PT#G)Sd8 zY7!mcBp|2_b!;N*9`Q^?Gt5sjd(Nk&3PdiWh zOW~(JL};Rgxc3$BQY*U2_SSvv`WRA@yGBN~gkm+6Hc?3Bd_*?HI8AMoiP}PC;{m(qjCMg;$kMwm@s!s%%tGehu{4s&QW{v!WVcOAb=tbin{C z&xxBlO~(#0TK=U6&1DON829WKBFr0e@lAwkaC&2t`}N{(K=XKQvbVlhrSnTReyrT@ zH}f;RhUX^yLJ9uQ(7Z7`Cuc&meK;|#6(5Iki5rR-u`=5fE{Bq37H=NUauDof2s2|R zNeaO@`PxP8a2HZU?K@2HH)u}k*MXv|r zR^%Ng7^A{MMf0)8ikjA{MlbKy+Hu^%mN` z;X^+@`W5~c2`grv1g0v z7%}Wu**NC)Vz8*~FOEv%%j;qXLG%5dVDllIZK6jN8RnlL-!MZ}o9~L*Ul4H*cFJUh z)>bsVcI0hr*4Lni2yAgwr=Km9TRuVZ%p`tYj{~xaUV@VX4E|v5>ltZ&BG~Kty5@?yoO*gK9S2iC5hZKbiK{ zBeR5Cd9kH6Ao5Yy+{m{lym$x}_zd|ZPD}av?oYSFWk119y7}wu8B?gi10yo?Qlae$ z;w#ls#Z#{;C0@4|BamQ2%OirpF=5h$9H^ZWrwoI1H5bE|o$uKn>Y)OkCgz`97fhjOL7FFAyNX%L04W(}u@`ENcP+s&3Z>^A0K7iSG6Jzx-lO^eNBif9bB! zj8M^iXf>>3cX?uh zxVUiHg1^z)>A4TGVW~g;TxRDbV*EmuT-bPvE2d*MnVA|yXZ*vm;h2Fyc9=UYWL!~) z6$1bFa3ujx=(+4o-4Br~q)j@PEq^+5)8Nqy-SXt$;Drh|KMctOm>)hWtcDSETaTX$ z2l=cK%8s5FbZsmOA`R?asTL#9;d&O6Fn^-TU=TZ7#Wba@5BxTc(EoH<$XXTc;CQ{` z!1H=JE5MWpe_3y|RWja&sCN5--Rza4OzT_Ib4%+Q8o&0=PN%q!y<1nkC1dKE`$4F% z6ut6~JdbuO**(g8erJp+XNj5VJd))NV4Ker7($VQjRfN9tJ(zVzROp&Y^`kAg>=tX z1j?V&n)G%)is^dOuEv-B%G$=7UBFC!aYCrJ;}I0;#S|I{ULiTI*8$rhXt% zz(KofW#`H5ZRMeAQA8x@B1Z+uPAt!FVB>W+q(@Vk8+C`L(O&;c*c^R(mfX^!FPq-X z{5U0^0Q9};Gv7*1dFW&D`a>O0nBptIv4jN%9CzJK87 zu9JC$=7s)xV5COd*)uXy&G)tn$K6Cn;Q>-Y9cmySi)UCvRv5;*IgK~i0&EM>+T~5} zxGrGT6i#1^Bz0whS?eRA(46L!5J6HbzLm=28`DSXUl==$6y&;rY2hxrA0jT$u)BWpO^VQADvo?MdDkU)1fhIO`?omJ(Kc&L^BXhbj#B7b z#ssxvnITWovE?{3$$VyG2fC<5A#B6wXu-bpo1j8r7+ldkuTgA`Oul?jeoNjPEtt|U z%%jJVx!1p(*vO0e8sMj9lHR_g$kLmf+S4;?!A+INkHaxLB;7M#-13pBe=;G0&(nDG zGla9TCeQO>hUpAyV~r}u?P2F~e_<&-qjnJZt~9sEZG7uXGxLwo(J_ED3DL9p>kX&B zOQzZ=7EeIvff0i6N|1*ygN4yHZkzfToSkz(^qy4Tbzcx+?p zxJolPNjNHD(7qlISsi@7AGZ6qQq|rqZ1#v2FWf?0xnOl`NY{?1l*3~xUU5?O+&%9j zVk&KV`d|I5^vP}B9ZT~qA?1GhGteH-Mfc~wiA<`e8qNB2B!}(y=s~?KZ`}3+bQZOU z^7)?;qfu8f#P|qXX%v^`D0LHMU^OfEnNr!=c{H;4qhPnotYQvHFb(_m*?UraMwf`3 zR@4rm>rB1 zfJPKVQxS>ai?@Q)_W&{4FB0CxNX6Wp9U{j0Hs;y$%YI+#=HBqbtA-rTDxXX|u|Fk! z-O=R*3mRFeZS|lefa1XaN!ucT;kLQZQRMsWo;#j#T@>rsFvjxkSXxqXk2)UiN!CT7 zVVt@PD_l=-Dcc_QlsH0~G-qB>{5y*Eh#m=w+f;3;aR=(_q$tTzBbYh`JH4~;mTA*4 z?)~h+tLy;pbF}9C(j5u}jRXVz@Jyu94#sJLrNe#`8e(;Y5u>2_Z_g}jsVOO=WQkfC zigqE2ty}KCI$1`J!ts^p&C{{Fo0O6^(oR|GCCT2JwNi4L{QHcBdn-6I3Ex_Xc6aoi z-5+3=CAbmbnl-^Z4U=<I&=ljjL4CBj{CJ!s!&=6l(P+bDsH6o zm=4~T?&dtasg#8W+mmOeIgj9`2APG74dtFzBH5bnk&q@uCsh6cO6JM)MU(Lqk%X=Y zUZi}UL4M(`l}SRUKGkd*aGg;UW5k(|IW1IQ7uPnS1XG*rIRx54$#{<@=QgY@r+|bIYvS{%=m|^+|#4J|F?shp|Eyll+G=H{r z^Jc6s4uk!qsTZ0W(BCwXgPbaxV|#f}Kw-;2ok;gNPmMdzLHx>X^jc=ic#!0$b*J`q zjEsDvq45corq}L(9G$zlngXK0oM0EKvy99}w(0w3YOkxrJh-5-l5;{RrH0RP;K|XZ zBcVrnRvY@^hdbsrCF4#pChv*zgA9gHa||S zs>xD^r0Hr!b?_J4n$Co~zVJ3A`dnD+?Lhh!X zqeqaY{NCEk8!p*!BAMuKO8Ai-&MGUdjK(Cp0;Sa+m1046T5UsX?(iePX_14R2`RIO ziQx3J;q7ROPav}WxEXmOez)7E4FN=0F#lXS?Zrukvbp_ZHkDl`IP%U_%ud3OwZoEC zCZV^aE(~Ls`kbg>xCwNY-ZE10>~dUQm1~au04Cfjg(?WWm?Sg8u5K>&@HXV{B<(IL z{v&LD`9bZXLTkqZvl^S9+;3>omIi`kbcj&hR|mutwqaX}%Bs%U-68HZ!(5_?M_x;O zOG;61LQr8})%F5Y^AGpfI`J#^E-ZPx^j)a4=U|4JOKu#1;@$lU%gaP76Tvc&f>8VT zCWV?q?j*eo8%RfkRgfwi+?ouXyyy5feoAc<8|7+0@a@>>+Gnw$S(hpZddP!~+l2v} zP}D|P0P^Hi8n#=Hp8cN(o6!U*JJU{NL5`vkI^F(72rbjfFvPz*qq$0Lv%^rssldqnrX7lBq!fklQ2$-DZUea?ys(kFK*V+<}02m8TAZgS2PD{jqjVD zlpn}+%_#NS*)($trd*S0O{&m2pmKeWea6vyI^GxN;LL8dwN&nnK^R~MksR?xZZ`#j zLS)4_55WnJ1SAAw)q+Nl4=E&N=xhPM{F8Z7)P5_O|E#myqwFe$TP5;|xSz!QG;C2I zFs2=%dl3ER9Ht-EwOiSGNytEupQ~9yv@HyJOQS)?&{Svv z7ol4=a&utWI3IqDpFJ$M5?s*8)YEWk2|mq|+WPMQ4X5Q(k{(zWBtze_FFo>fHJ1Cx zzM}tY+p8YO3NL-Z?V0Am2j(9B{R{HbQnmRl)E_}!Ooj=lhvugExCO5l^TPAP0bUY&(^JTYBZW17 zJqYr#A*@VRF5hEJZ(>lm>T-aMTLVk#12}t5BNJ%pW23YsNyvfIWoiH*T>*2v&_4(b(KrJSFZp zzwE|1f_`o`+DjS4;e5%B*@AZSX(=9q8;Ib0ke%u6&B76%HUCXy}N2k_lYxWSw{HSER5*HogjmTcJ&RH6$ z>tQ0tv`?N%F-7q(>Xr=WM&Ii0vr!NrF&w_DmClsM2P9O9~mMJVgL^5%ZL z;wN-j&wgFV15eaq3qcET$t#ayn5LMsOu%#Z9PrFyl0kijK=O6#XSA_#PZ7*4SEOO@ z9jap=9K~RB{5gj(?&!_Jpux|{%!DS*v^5XnXTV1AyYDbIBXDF)d`3&`YVj!{XmNp! zc#BY|WHk1X3c$PkXq+ndIfMhdS2*i8&1`nHSX?gdixS(4lm>rl1MVTl*N0>~cT~WtZ4PohN9o6AGLb)K)EjlVXFR4nW%Y5Ypo6CVm$@r~Z#~n@S9ek3r=BSrWfPSWqoy^# zmtY{hzoUXw{6f7!{3P45b*U)s{$ZbXaBi%#YI4qr!#~tz@F>zf9gUts>vUA3Bu*XW z8OhNT%NQu4;`N#n7F{;7)jDG`ZL`NFg-RQlW%3dBH_otm{rcpt_2^Vyqf3VDUM@!A z;90+8ybwA|6Jt}i(y^A$eh|c%uCP4MEDD5bh1` zwV2=JES8F@XZyU;~OL0i?su*9jRMXucC-WN(J! z!#uSn6O*>4y0cyo3pCDvtoH@iM*>>%IXUtD){`*iJ!J4P$F1|Fb7s)0w;0xF$G*qs zLOH(I?rGt-C#7-|BS2jR!f*zA!x!dMaEwP}dd`^+AAG+)Gh&B;FP@bU1V&LUU30fx ze$>SopqoSF1Ge=|*)~-be&skR^Chz5>c+I5Jr{J@#_5Ce1_BO8Z)>l)IPB)RM=!mK z(2dkHH95lMWha?|+p7IT!c2T2U+%U`?zpoo25sl+GD$9NF+?uJDw`@}VwL3ZX)N<& z7GA$pJnF0i(V_Y~+VN!as^&m>UHC)lAH+gVS3)S?AiB7UQoTjDWj#sCe6a9!HvE`L zz?mKE9z2i~a}~Et4wr6NxQ!Z?X?wyQ47tSMF|z&+KL+c{4#FqFkKGHPYZE(B7%JijX+W@Tf7$As8?W3%km4#gR>3Hb|9G zeQBlRq!o8?n<4D%rRm0!cW!pQ0yO38SY0m3My@9$BTe{pa4yp%U(vmaz)lTuG+Cv1 zwl6(JxKKVh?9Si}MJ&ZPoNmRJ)NhLtUN<-XNS^kkb(2RVPmruV z3FcMqjm2t>!9LS}vNag1VXnh89W{PZdE9UJ!ar$CU{!(2kEdRjMe=r_$1sPrLk=3v z7wMx&Opeuqkh zLR)vV$$t7XlnbPdnaeK8C~)VJ1DW$27c!4c*rHX+a%qJ++de-;hxpxg1d|$vRTp`~ z0wLP^f0nzR2t;14ra6-~@=tzfwDdw5(Z0)L$9kh!@yc_yqxEt77RSS1G(MJQwY7_D zoKR&VgOD0c5@%%RAyAKjS_U60j)}bUc!sdG1ycj@5N&~xBdY?X9o<>>@}}b!)%1cr zFG)U%?d=zrHQ5iQh1x*3GKLRkQrKe2B;{43z2?a`Gigc~1U8t0c!|F#j%{4C#b})g z59Q|`GSweJ@sp{awA$NnXo5%HTec6<2#GgA4OOidYGk4x_hbP`=a)apQ2J!gqE0zl zgHvYJujlB@jxmp~p<%0xcIq<$mQlaAwp%ajm-7o#g(+En!m?nMaxnxA4V;TS@1kSJ z7a_A-1{!Mq^#_|-h4$BkvPJM_?d(2I5S2@=9rDbhrwm^?LmCHPNmd?Cmxff0pvO3Q zKLSoU1SKs@{tBAEo)4)KnIy8=vF!3;AI3N3NmMC2UaTLGtUG(PpR>K`gWbw?33Vt& zH1+F>2MxR^{kG2nVeR&&V(CF#H5upU$e(idrOP35^5cPx@ITCI;Uk6fgQUt{S0#xT zr6P($rbR@%CBT$_Xis~L3~1IhG*zn3(wqB31PtyI$>mY5ve5OSCh6IoR(Yu{&hW(v zFm26&2NoY+vC5Ph{O%Jv_F$|Z2G){d0Hj!-M{k-NnTMYWNSOL%=Noo4@NeMSkaQGF zjoa2;H(}Hw0n3Gu!Wwv=W0!VhfO&e?1X-we5-cjGM(wIZe z<2l`g&G$CK2gfo-!Mp80mc)t7`c2Z0=r;b?eAZMpje~2kgZ*$&bkUN&uh#7s2yro@ zIaL)A8abpTEQn#>T=5{#@r0fVr_r%heXkcP6R9%*=En(1dYhwU{JAP?IwG>ZKZ}~q z$C_cPEqpyX?3&iAvBu1CY+=J`26++=ZYdlKDe&x@DiW@q&IdY~hessQwXgRcJ+`1{ zkFae=-wja6rh59iRcaWhT5xh%kV4lW=^Ud7%L(E0NNRVHMw@o{iTy#k7dZsATD{`u zHv-ZiVLMShOpHt6}Mq>@d|4U3X^mk)w1` zBm%*IxN;NKzj1UfHxHK(#t$z?-*lHAf4}vxv@AScNv+6 zN@T&VBpjxNpl!>mQgM#pFjucHpR0JRr+nfb88~b4$VEEl2%^WTLq#i_EUQJ|Q%~aU zYnBO1VJX3#2IFLymMz3+rs9dCt-daK{g5Jj>|w-;OiI;{bz=;RH4pz&|Esr*b}9KE zCJyV5#-tCiu%W{b!7+(+{p`F3p3tLcs!6L%2}eo0C%d^Z>$kmv5uZ{~;VmqGIwOit zk7IishihJL0c?9ACH7HZ790lBE?sg<2qJ)l;L3avuvIDcA1*mX-FEK{*gp#xE)Qzr zMhOf1>rXVS8yV0yN&63A)u;7|X4Qg9J804Hun7=Tz;(g3V`!y#X@yxD!f93;^_W#4 zBePj(g6*{@Q7n7iH2db@(?at2u&1@qaoi6%SyqVo7RY(o6eCXy)>YEe4CuGQ7>N?A zuvWK{)9=|#@Nbx^%;2ybh^?u>>SLE~HhAYi-$emw{EaK)tFa-ZU>^hXthZq<-H6-8 zLiZt}_}LRiF+-7|RftCqpe)d08gh=k%xWarlD}t*#?dUt(@sbA>JN~*#~Vd*$L0Lf1O#hKo;@~H0@|N^!W$P=@aiXhT3QS z#5|ko)SnXNUxL-E3T+V-CZe)^?BVQ4ESf^-f=j-H0aOnYVLoW_HlnYf?1$eiQRd3% zL)Y{efbw~6nY0T6+S^cF44pWRm`B;R&wk8XR(hzo#IO%&wVhqzr>dli`b20}rs*l~ zwYU2&$9;TEV9|UMH$aabX;0PrBnaO7lk`|VQu-sKn+{~siJZ4H94(vYt+Ns{w00mL zTbrg46IaG*mHwtvt)2>4M4+iLZf8D|tp`8L@$E<&aGPd9oDt3suC&(M>l7&mK&rofj3fG;#6wPhR`hog&dd3Pn)yKH$ z@LbXE08$$R$0CaLw^V~qt^^FS#7-Mg+kh*dp^AB^*5#Ogd@xs-U$Npz7IT^OEQ5MR zCDq&@kcw^Lfn_ezEx<-?{mT>Ta2Juf$(m%Lzf?KgF9+DPjyiN5AvHXVpnczDfeHyz zN$^;VtV}!I#jP&uwl1=Kz@0I<&pjk{?|r6+CC}p!^cFNVS5|f9(6vm^7NxgoaPh*t z`ThRPD+X)(Tr|NxO79WBc+YpoWh?JgW#a`N)L%r!P=*iEPNkBLd`6n3@$j%x2wxXG zlZttUvmAWzjmI!Ih;HuZYL>#dvOO*>8uhxyqKu_4oYJUj3b9FFpgE7IpVyzk7}@+_ zp~_EGWE;DFU-+ikn%6oZBAL!Rbk538<}z0n6Ny_myv^GFP@{~6Hkj-+Q9Heb`3dK+ zM{^BREK6+-E|kd3;-xskd@HPJlWZB4cmStr8yEJy6b{u=r06RSN759d%tz(klb)>3 zo5&E)R*dq@ZEM}dfFOkn<06AJq_`s15!y+%&YrCz%3bihZN-T?1YR|pPG%digx8c^taBTZAE^D!q%wqlVyJm8% z2cm(8R5QKIaUK+fh<{}m1{@J^?l*V?xLgh-mu}`;*BsPVK0vntwSJSu`%02jVJhc^ zs~xv6h5%yM&Ml{F)Nwn=CEAG8qFFwGYkd@UwO3I1TdoPKfALj=+0~%Rf=u9~riA!c zap=9?6infH1WBkuN1gsCH(P&`4q%{w@Jc&s=!Y9~9KI0i9q|H1bHU1?Po}VJD zF|{2f*7G2PYBCE@VflxuvwFirKxFJZo!z`94p~9IP1MG2qP_dL*3aG@sU$QRFvKOl z({$A6d|A#<4+`gmNl*l}8$TG@#Bwb8_apf*kCH|2#PYAVZ=15rA8AMj$l$1+*mixz zn=wPLKN>I5&#a%Ov!`)NWfYaVl>2%$%!oQTN?nXwHb5APU?0SnFz!fQP9JMA&1XXK z!qa3Wzq*|xs0~Y|jkq+d+ZCyGup0A)4~-|ri<_{0L(q)`Cb#5j1Y1TQ(G^SjY!j6< z^B&Lgdg@GG{^~J!v)2|V&#Ax>7`p4m&t@7IFy&xiA`!7Nx~%I13c}{HN}6jcjL=7F zG^W9udYj(FCIDQQ8}PcoTfyyn6;Rg>_2{l2+)e#uAm~ zq}MkWon1OG<@4^OuWUUrvPYH@Ie7uXiT44HZ}Eg!lcoF&q(rkXf-?oW)ZAJU$6O2v z7Rg2zbg#F`gXHy8$>|Q_IT_MbHI}XnAXGow*YLM8z{YABXQE!Y6#`qgxq|$llHT3v z6ghg$H*N;63@%p9>L8!{Ud|v$Fv(CbZF;7wBD~#-JQNdm);wcFOmqMVRSZf~RI7(9 zL+IF-W7ZBVRrSsuPB_+$)eU6#W!uMtHr@dI!n#HV$vFxQV z7))bPHEt~h%0b3(=-jGFs4i>%QCd#spi|*k8p?RPdboN_cHiwM;ykXjGS6vV?iqsL z1qU^|u<&?an=P^p29dUcn*fKuvCOE>ZW=DU-@fwNmpLKRiTp$+i!bDf_4hLAvPZyw z^0=r5-ji@0n^5aE{U{Pcl?bka6E)iTEWgimv76)cJpeKRP3drjU zE?j1=Fw}3UP)I&Fz|IkdRPC@*^Jyn~=bQf*kGj{(INT3ZYOope=?~NtsO+@Y>c#7VdAWnic$=8;xG{8aevPhFplYZd@{$NuMd0%FUQj|fW+du9)n>W4en3< zY2h;+8k)8Rt^P<}>h<7~-U@@W7~uI`WR01FlTHkB)n;P_do=Q|;SP>(#e{{ku- zmfWwm?^A3>jj`MnUEvnrDFt8>Uz8EfryFJVdj(ZlF@D3-_;Sf!>n#s1Od-_iP{cH`$!UiUu!bAhqON6rS&t+wj z#ro|0PP&z3XvZW_){10jBF|U4c3?gXF?irKN_UBcB%GKGZh6kc)Jfy?cXuNDV8~K8 zy-}rjFC1 zqD6e=0&PU4w{u5EZu&I}xuBFET3Fr?=bI?dm-#HrJ?*lvEb1Ggs42!F5vy~ zrG6I>pP3MRO9*x!yp(ithl>q_`pP=1_HFG$Eh3E!Ydnk1F06S58W1`qj_C8$=gR)r z=fyF{`Rj}Ps3=^v^+hO(l1N(FrNK@STJXt_<{en^uI+ZjjSQefDd0y)0R^q`keEn} z@`-&`GP3&oTv~+d@PiELj-sTlDcHWYdGEsDJ0bsO8xP6z2Ts3l9bbE*7k<96s#{Jp zA==s8O{DGh@9em}eGn>$4}}{<+&*I=ZKQIOajvWyR&QS~MG)I>vHTn=$1+)0zGygG zOPGKP1+C+8nM@TIR)DI;-SCl?JzS`n2eEk15Q{J8=|dREsm+f<5P^(q-_`{I<)KL} z{%F!?cO<{GZG>tFHH?x>Vd6IjZTMk;7QnY!>^S52^Ml&1pG~pPILb=lT=ocHqe%;& z{1_v?U0}cq*6tpp4nHRY>9a0BHXr5D%kSFRRoa`rjagwJnN}dJN4OA$`mjB)73ZG- z9io^mUZrNCOeb^L{><#_83^C&b2fiBTan?_zF>4!KN!~?FLdAjnA1*E3s?4EL*p`ZpJu#dPaiiHfUOUG1gCN!m=BQ z$e^~1QiL=GH@aEFN~Ex6+_ae}bHnGkim2%6z+oBUvzzx7D!0Nju#5f-Dmm;`1<<7C zhq~L7lP#SnQMcc?cvx*;=_hox`88OTI)w(eNl<-LexP2QlPC(gVt_f zltfg0Rd9INV3n>*lYgH>9kOg@@o7Q-x`#$WXOPm1Q-h$(I36m}yMkNycoB(&Qrtn` z!|xUcy_W#lx*x;J>;^iyyY8PEWin8F#ZvF)cvz^xH*y|sJsK0?eq%Ca z9*3{Gn^xSpoQfzvr2`0Sr36c22S+@`VT$r>3XR-`H(27y-7l?>!{ezIE;Naq5~5j% z1=LzxYG}a?Qws8ctU;tMR1(5owJ5)s&KSIhkzTaq3Of~pZfmx2c}TE>i0uPp(cf=w zXklK-e;y;b2C|@ei%~hXLD5O-!yUs(WYM+QdG)cjh$|U>S%u^?`)PQCVo2NU!t^+? zu*|PZ&Y-Gw&X#A~*(o0v!^WD3giP=<#RG7OB&&$IXF($YglHLo0w~KIu9aEkux@Bj z`pk8M4kz!9l3vVowTaPp)}dmpzh7TbBe7`Zt6q_tmE3DRmmdkJ@I1rcA-n5FI`U)R z`zdl*z*wE}pusCGyFq zo$`n|xqs_JMD)7)_c@_XK8UMBVXP=FDJ<%$ON+W zD$j0WreNVjNKgvCJ#8Ugh|gS7j|rnTEyk$$ybg_8U&P@B($hMFe|IYs81alDE4xn{ zZd1m8R!ple+kZJ*;QfU@W$xv9zg*R38VBnR3YPZc-Ly5YEHQD zIH@=Lk|~N3_Tj;6MAxH0NVk{G$5@)@RSa8EdB4GZ2Po}U9F0mehf{8X934NPqqL`m zc6+2mV{_%wcLUA~hBfYHN1GAMi+O>83?XwH3+?SZQWf>$pZpzV0To3)!gay8K_@%>Mf6pJtd%AaQj;*+i zQHROGr{OjW)^lBBR-Nj&<J6E)!`-#~q7(Zs0!YS`0z!!g65EvJMOYX8~Z4>!b zKOepfEd<{%IWieDKP20e_VfYStVa#9ft9#n?zzs}fn|3owFC+4CW^_(d!y1*2nII7 zyiG-u#FGlOj=CwQqh@zaokJj4|LZ(+LfL?J(jYbXh-Ng7=X4a-Kv}buHvSw(?!8hC z&F(%APUkI_Ws>6FH4|P)WLHd9_X$@#vll;*o%G=a6Rn9!bQIa2n>}K=sk|zi>fVEf zswPGBfvSOw$g?(JMy|gFmPf!OqRTQ&n~{0}<;BjZgSgTM9vPPEwARS8mIQDMffe3m zUII@{b!d|{{of}$!s zp1I`E<69j4h*Q(QFL0#{4Mhb8h!0x&Ls94T*3X(GIRd}4Y1f)@`iH5wld1{I<{TR)brDz5xNZA)j?*@5Xb_*j+qSg#=*eT6g^1AibuvBF74$rF2!6=IOh z<+RWgWw&Qgp(>0&w7+F5-Sv4BrwDaBrj3CwyfGx-a8*jM+Ht1pDqIk=0ONDRHj zurncZBA>l<8SAm|RdVM;9c3V8h7S*4q|nx@dF|%_0vPcxoDoe0izX_Op^L$bC-{0t zHF@Am5sus;@@=1s_{Ll1OdLhD^s)-`giVG*4gywUZWv;Ucw;}KxJ4_fCV$c!eFkyw zgCK_J@qz+Bm_4A{%qeMoux3QYLHG%VtPvLN(S+Qrz{s#~MF26ik0s-`CR|a4E-HoR z?tX=Pz_i{hnU?Pu#uB#+1rg7n4eD64_>OB>e{@u2nvCz4M7Fus6h)I#2UKM|_*BKt z9assIyzsrP_{#LwFHdz2f1GlO^+l~SXnP<8f)*MFG{0wJ*mvxXF#$ESaQ1oj(Lp~v zU(05pci)V+dY1ELnS6g3e=*+aal^dj851nL;e+n4_M9CYbR?Ee*0W*)Y>xgt$DsR9&&&+* zh>-RdQP#?LqC^`HuTA}M_)lYJJx`wmb11znW*ELfx;iLQ3R^~eoUExlpz;JMWRne= z7~{A}bKD)Esry?`QDGXINsHB@t5k1}vba!pY6x?^XqvBWe=@KcAYC159joQ*z{vg@ zcFiqp@JLU=yQ+3A4t`uk`kHfgEjCCJG40#V#A-YHeZmNZsBSKd0~3zK=-cS6qdh)< zNG`eufPuG_rk?f$PIBg?h3@|abR&!0koyGja1(~MA!=p2U2`4zxH@_fHmS~sM|nrm zt}Nye{Dy8k3lcV^tpn=tWl-Do-( zE6By4nFZCRjOKWUII){=7_b$odejv`XUn_b($q*{A=qUo6%7o`h^p53F8cB9XvU+_ zP&z!sCa@~S&@7Ln{lYxk@k>%DvPSoD@P?gRVZjhNfJ}If_X$WcVfTUhx4ryTv{EcNOFC3RS&kuiNP(T`=&d-}V=QlzNCjV- zivY8lcO9H9{yqC707hoIgf;#U>|!py7ULwA+88RIUAHWUEr;Mx?Q$W>xX)V_exsy- z9Uq$#F7wmCEs}GTZ-1<{pj_gTvt2LRk`v5R8L%_|;FjR`w%xuCr)i=qYu>_ffFnF% zT>}!oywr57S>radyD;Sy&9wSh^g(bZuk+nxDwc?|m1^|Bx0 z(BzljZbg1AT6i1mVA!%wF353tJTM*~+Qa3#->a7jDBRW5`|&aigkL{5kUg@+nJ*>C z2%;$mELxeTAQ{q}xJB>^7k;f6h)3lXrjuow3=Kj9WJLW(DIReCk<3H{K{z303NpLd zXu!&_`SwXnE-*%>1aO0o;e>y%%b_uVF(qwD_3dme*`HW}h_*h`rBRK zph+Kcfz&r5E*qKO#lzlb`5P>9iz}`I0J`jTU~qM31}Tf@6B2jmaof^jitGq&HWQv_ zEa{mZxWpp0#X?ucWrCT0f9ve~fS9rr8D7nyg7VnmlNf7= z3o$YbPje#7Omq3p7++jy0Gg;k337!BT_(+W-{$4z^-3_*;|DJV{-P$~tVhw< zO~KoSZQ6W-?`~vludWeqgah08pQ}X`+9~matc8BH4q|3+WRx(H=IC%ZAu<;)2Jr>I zEwvuQ-zO9k(dI?p@B1(|2cJ(6twvL_O8TkpseczP-1%&Z@GQ{-fj$Be>WOt&-Out+ zQ=$(w2)`<=49E{d3n=bYSdw;M!zNxv`NLo4stP<=xy9z5Oq~0R@S&dc2-kS!b2#q} z^(R+n|8q+59g?dnDGcj^Vk1C7I)3IdJS=X}y3}xF2ORgPE zC;u{~*N5(tM}y-JB2H}7;jgLY4t-tnL!taH=xBBv5h~LG1y(_AI^O(}DXcPXFzjq4 zKKpl2lRmT3X$zX0h<+}OcrA#!&soErcNq~R=9D!igw&8>>1bc%?T92GEoq)(pcCt{ zX(*q$GdpVQ?j7Yrjv$jFfV@>M%>?4ZJ9eP<7a*9ba%`PbtSCX3MvraVwr%?!+qP}n zwr$(@*tTt(b7zu2nMtPJx*jT>bU*C+YS&uPm05UcrB-xVNnRnP8E&0Hie2Vc(_t>v z8%F8fR7UA8;+wE8PQv2+2mLVde?n^$@ON1hSyfWh@0pYYm7@E3 zjgV_AX4D{0>U~lo^e=eWr(e~=rd7hSR}on~XY{zbNxgLFgrB9?r)|dBBH@n2+lQwy zCuSue%Au{kp0UQTLHZ5->7lm-fbN{-KtQEwm;|*^Vf1-io}d1C5*-JBHlR@6&vRs4jG66x%b_Qv?TYE zW0j`p+r=|u6{GTWH$_90kpM)#g|7G@Y8is*XSw0zEE;O^WuSXpm!`l+!CzDT+hLn5xUW2c~6ttrfh0%Q5cGTc;%d=8?ELyO=f*h zRh7iWQfrVeX$pP4nu0q!L`x}R>u465mh7HwRTBS3H{0E`3t{&2;!G!Mx2h!U6!1SSlGSA3~#J19$>n^llPAK;rSzvBwBGoYdK$U6D$jRaYufnc=BRwYBg=jjz_Iz+Mc=EOicE z7Rj&#xv?wyjmej*TKxCPA0pK4rKYS5!Jvl;I~=(sE@Y#tsY^B&Z{Ouz{mXG=b}Iv@ zj8#T=gFFE>xkS+5n7>w$u?f|$+Ox%M+6sv|c_K7nHd%n1xUntV6Yk(sRYaakl^?8@ z86XU1+(V~OE;bHqCqxajZZfkN7>-Kdq{qRxBiQ2^3F8%XQh zN9<__?LEz9g~fHcvFiS)fajm(22F_Il&`KrjUQ_F*%f^LMxj2qHVJcFP3;h$zgFJ& zbHMbRL*~&0c!HHdWhv^cuY_d~i1v>KR*3*czmW~1hP`B^R|r9z^gQe!3dGlijk7mq zS_&`io(c-WeZQZ_mpI{wN2YNXKP@HP%}T5JnSyyK|5w+Y{?otsQ~XM+M5;^}RUmk( zr?{@pY#q~F@6tu}Yb(cN%0@pNEosos!aeRx4liW(*cREV*qaJv9_(2;VZQVP&;jGG zXXB4>Hh9PrfOYGNZ@Fl46b{#5gHGcXGa&u+?pXB$*sZ03@pE}Z4jPf15kJ}oWTvQ6 zbdKf{MNn>kafT1v=0c};Na(px8h7LtVJCxBN2u)!`Qp?fowR}d&fiC&8MX-vG^rtX zphFQ8e=_2v*4rJXp&vhND1C>tQblX!fyz77j<@=Pe6=R!wYAdDe*gk6OpYGf)|5f^ z`q^6s;wz+y-TrIn%=>W;{B#_KdG4;YjQx5U{Lzd>N)Og_l-%?*`}hxtHv94>5$~_J zFyL<{Ul7k<_djCPNTID0$VY zbXo3nD_l+{5MpgDzG#d74r4^V2cy)Kc(E|rfN!b6?OL`H^%*?o)nXlrLrV)Yho?BX zs4WdFHsS`YgtaOGl0Jk{uVv}FsuAU~i~bZrRvGgk+>X*tU>)R^v^++a><`x7U?QG( zuAE8B&iIpZidav%G6EpI=)&;%mQTKeZTYR^L=*x~)mF|%GI&_Q!&u}C(K3mB#XvV1 zvI49AA`->}e)X_Mj@b3vZeACpiRMlc=U)pPZ#R#gcIp=7%f*jSR~*?Y+;=#=5xZe5 z#(wlEXm%;le;w^qB->(0vAhc~#O)649y=kgs^a?M+e)hi!mZ^)yTzzukWSWKdXz+$ zqgw2b+siXs^oJPB;=Q z2vIz(;;|eTU>=VgQoM$N(@bU3uQ`T|Il`!m=XfaP&X`Qp1T=%~2_S{67%Q^BAW<{~ zyJ8<_G^?uaU9d#KT0Jk{y790}R0=V#exVu{WrQ^XjD4jkk|qOJy?%Pw;|nd=-KA>P zD&I^`b;f@MYW^X}9l*33*Z><5vvcs%4r2VAhM$B}rRcMc(a7z|`zN`Jl9rpjHPehl_Uo>&niIO^yYKPX5s@k6UypN}ufNeZ_^VS8jvbVqDNT^HqPob<3-C zMw-sos5=)c27z^r&mt1{spHNGXidO0ktkJ8F$N9CE0Za+1!2 zkZ?QEHN)++VTQHX9h+d528YXPWYAa3@2B%X8<#%KH}|*7pCD=2G-`jD$h`(?_7}^V z{Xs_)m;AWMTcm!)6(nBe!3NCS8B`L)5A!cWZhOcq^im8?|8_m#wuw38ZSXVS1?rj60Rw#@(rtj)4A>(0%%3% zTsB_Ccie)27(bQ|Y}1rc3ZW;2s32^_j2Un8wiHqV-)vsaHPQqWH%IoV$=pu7<%O## z?h9R72HbA2$W+jMolmGB*NtP3_LEWQN}87aApp=!J<1t`%lzWjZFcm$RAAb7_sqw9dbMwx<{0Mbuxyb)vfGYtlD;>zfhJK5oTB!OvCAg zRjJOX3d!gteOrpEwXX*}(QH0wZ>Gm{(w5 zfpf}YLDpqeLoG!Y*bJbGPq%I1F>bq#f;T8v!=(qt@6>G1rH$%_tMnURwswZlmD(jS zSckqwH5VGq8zJFLSIPxi9*33_(J{KjJGD!_kgOZxjS9`@h(9aJ={mUAl>2Sf4YpmW z`K8bn8*}~67t{{-c&E6Nrt;X$=Z6A)O%pB|C26mmVR@+Fr2vS?4fs>WxMtS#Bpvo6 zS$PQQQKxeT&zw_Xh`_KUL9c3_8upv;r4wfQiVTmVcfvzRr6 z0r+#p`#52I;k!TWiK(mr?$6q?HsyvbkB!G*RvV1OB^BhGY=DC#oB(Mydd1a7lFL@! zc9(QUNEnJR(pRrRIRs4E-Fp1_AgNeBeON69JRpjQAbThXE^Xq^fu1c`ojBM1bc~to zm)f_RkH(1$CIuOs#y$j|G%}{G4fgRSmeTkw&39rD;Q+3*c(|`>LmT!qk`08e^c|MZ zCfI~aP9S;iavv*z_o@|eWa!vnj7w$d;NFl^EU38Q3F*#MgwAtI4s&;=*bNd_S?uE& zgnK--Rgnt@^jK96NQ1NBJytjmb2!{pZ0jR@fs*Gbr?&}80NM!}kcx*iAI9-k6xtld zez8O-hT?gI_9%20bKMh0sLQKR{hq@PKWTTYkin|ZS=Lvlv>u6DUPI4wfNyktz3Q`@ zlP|MBw<|kb^lNCGkq}2KLr{TCP8X5}PQad9^Kfwh0kLP1|6n64ZTC0sFl+~IqggNA zofZx;0&YIn#d@AitLq*om@HKdQv59%8*R z!3Gi&*^HB~c zm?(!l%PMSG7fm$*<17lw^8w~womsRV{qe@beSb>$6G}dvd%uuQnVDA+#%_*%eYW?O z>dG+3C(+fGI_NHX-AR)tlbchqy9M;1BB=G6i<8`nh>Nwy>u?VOgaI%g@T6h>mInT7ls&!x|3~L3jIoj9e z5S%hS;6d`)JfQw7|Cpn8ZK?ym{$I2fH!KQgzCDcbyfdE48v-z?wubYw%OsLmkJoIW z$;?v$PMLV3tFMRRI1wK4iRaE?4PV;nw7JA5R5I!axHQd58h~P)GTp@A%w`w@i1l%a zij8w;Yo#19k<3mz`VgH+>t-QjWfZ={kBmffCMnJoyI<8Ob?~-O z4mp)sd*Y?=PR)}3NV3sk!p?fKUw5V8?5OT+62YZM zsdz?9{%Va$uPwrp$L^^^5@zN*>Q>t}a#H>*U1_b@Orm708=J~_Ap;PnxV(y@f|;6* z5=e(^{j<^U;X?jYr1nEDUraxx5~P{2eAi9qUWJ64TN6LERxeM6`;7D~F{;WGp*fz; zCvTMI4y(VrwzxdeNfJ^ETl&#TlPD9Q0X&}44oZf9MpEHDs7Z7XNa%295lVs)MZ%W` zbANG}>)<$zZkLMT!9U$CyWlfD;yu}$OWq)QD%moiyzNgLuN$O+i)H{y!=T&6R+lI7 z{`_31JGD3XB2RT^4}4z;%im@?KE_=H`*{B}K{DmLkn`ELXKqNrs9Y1fa%KZo+nC6H z4Eiu&j1L!ndr9(@xfuv9>CgYjY8CP$bt7~vCE*5arO}j>+1h3&QDXI9#9ggOUmO2! zj|8>AqZR$TD+EII>D@pLnzmu%%9QLa)l^5wJYR6&kb?L^MZ~r-=297>s+NCLqFx1E z!2qt7#&p^MD0$c9;GUx(x&q~Av4U@*s&XULCloahfVy{Xey_A;uyg6?WZ`+n;0Y9$ z>B+oB(y>q*sxrbt^^iKG^}j|#Pz5p|dDj-g%J zf`!y(ZVsg|H3KdUYuT*HkkfIR(Oofx;PKMe#$9HL?@HuD9%1$ zS4bFAVde8o4i5pH7n3;!7VrU5n9M>}{mc3qXYvgC2Y)&)dW=^1DyY?$!+e_P@Bq?= zsQqS=NR=$E80L1l{N<^!<{k&Bqo0a6RJ zp%nJtzkIm{Hd{bi{aT3K?HvOF~{;VyQ(XJB_v0Wz3q9J zql}{Ux_lqQi-(s0Qs&u|I&EZtn_;L&pl&~pg!i&?r zzd0uadgXcnojU`4eL7Q|!=1ZgqQVhU|2!Y_lZGW&)h(uTLXO%2^Fq=8O!JeRck5a| z=CSD0lP}FX*)JffmIbMBF~FUd^vNaquWej~Fa!4PbKk!DgE|Q>=_Y~VQQ|BA;_}_6 z3yqRkX03M*<$*cZ!rU*vAf8o&E(G{34}0)+oL7`)=HA)FdIcPjT_(?}4#-a~||TWuC*rcpO+t(%l*H zI`uoul!P-BMD7opC?9;>mdnR2ckJQI5Cd)K0q9|TbE!p62{Xg`m)N!K_vbD_7R_(T z3X1*AC+_0bqy4^=8B}A~WvCg0QGNIX!^F0%G6SCylnB)2fH8e}7vVcO3Sit{1t;sN zjQwKkC{QoVQD}$oOE-mNv8%yhqJ-)KWZS8{DdnxL%MN02Ix-j)(x2^evu%oOh(S+C zxlAn69#iKHF@w>EH}G4_TiCw3!d_jDU)4nvh2~zf36;MhuV@!SLaii7eR<7gcm`0( zj6Mu!a-&K&$tD2g#sJ`S$)E*=R-d`(MOV3$C9*iVUh3&0HD!o@!3$Bm`h86_v?d#^ zHb8mF(7dqbFD=5VE!u7Od(Kf_?n4}!MX8Ci8}=}`t_LX^M{J)Gs=VPLtBs0uYd*VL zPp9oe(P3PZ9`1%U5}r|06s(f(;C&b^rpmz(GWBVNTzB)oStya4JJbU8FW)FZc$H(TmO64rOAn_WuTWS52O9oY0hwCwiv1=IVfygz^IvAoJC@w2n=~+JK*Nv zXs7appu-R!!VHP^d2(F=9`1x{ul(4$f4~FPLnz{ub7rSlfZ@T4evHFs99B?rrv$gR zb_hM?;TK=aol8w}7kTi9!HUJR5Aa;>hXVmUp$~<@sI^V{_4Fgqim;(%X{&0-)TaYI z{`fzb5VfQEL%S@JOTowx2UBYripoWcXrC{KL)x0KWe#sX-;WSs-{#RaqML}>(56+F z`IitEJ8v-$>3c%wI_NVx=KjM*-@-|teZMpXLb0fBieFVf)C}p&>hq5`9AECA;h}cT!$$R&H$nATak1N8HnId+GrGRM5rP`m}2Vq2f~6q$PYv#w|~*Js9BL-j;+36heraG zPPfFW``EJ5(a{E-Nap7QqoWaqP|4(FoNfGZto9c4WIJ4}5eM2JAWI^8-4r1HO!rxx z|Af%jQNBbPi&{k&Fo--gBwl+5g)mT*RM@92ET(rCLIv}k|AGq;3ltTB*`d)5(1HsAP;|=b|`? zjVq|^VtEE7$BK>mlBShHkE((>Ld*c0^6^js>}m)};0dv8S$LT1nVWaE%>yDn*Gm?4 zgs|y;sGq2&y-b{=i!*l(G!_j(8n>=nR$)0vWo6Kl1Kjd?ZmNC2-Y$6EeC6wp402w<5`x&e1lpe-=E$U|p2=TCj&)9iTm` zofO3bxsPd*4Xtu*bW-a-a7Kd$Yg;jk`lTh6R(&`Gic*@)SI-YPC+ri z@JHU?wxQ<&0)<>_pxgQ&qaT4I^A1hvSFtU3#|<5N99eGEKQg)gGzzQ3X-;Wv(e9Me zsf0=#{(PBy$;%k;jM!~EoQbOZ(H-1A`oKLw)*s;}+Qcap*auQ@Qh(wto0qXxda*vT z^KGM3Wlz@voJ$Fd4%NEN}>K}4AZSE?1Tzr(x zmWz;x)8+uxr8`=gtgMjfd2T8U_S2CDF?4(flyF@*rL`dfj`V<wwqK|$alsj5gIEMQukYkjO>z##x%`qKW5lo*Ln5qog&Lmgp`SJv3La3(9(CeSX|(CTedMab!gBY=PT zWnN`_fwO(l{av-SHL)}O(ehwM7k8EQq9#Yr-`eyO6_%m!vqlqO@COEP1_IcYrCs-H z`?S*Z&EoXSB4m1V=j7r5nu@W>3E)#b5g21fp*J&9BLIML1pILSUcBFh2uZ@%H#LC@ zh2c6f5PYxd%xc`lVXG$n#dl4`*1IXLc zqazRr`Q3+3_+i%q=nCzutPJFM{Kd@Z8!Wz(#ht3Yq6N_I_69IQ7+V7l0cHZ%?(PPl zmm8DF0^I(^LIL2;>H)?`1hRJq`~;Z76`<&2Pl`kDr~BD_Vm|<{|JPy`2;f@rOW+F! z(Dj5r1g@|429MFKLo3<_8a6*aQPlYu+92QAmLWSF9FbB`;AWmagX^c zh>$n#hd_>A90EDEhT@o?5%g2%i=o8YbQ0JP)R=r<1}9Io_NuseRI z6s|Si=esQ=?6hro<_4Z$4lbO)^fzD&p?>v(EZyZGU z^aVab;}?JS8^LAsH}8nw7ed3;kAR#+f1Y7?yiRji^e?^%!SJ=tm5gs5DcZ031f)jK zUwqB@Ot{~%gQH-Y;rlr3e~V!_ntiRhKG^?Ee(YwA11oGGG9kj5{!)O?kDucs6pr(g z90c2cH3{tOLY#fThqx2&z~O87UwEykk{0$W1@{TLMycn{dZ{hJ&+cre={!2Rh= z%#pYgP)Z)=1XtQ8xE%V3H`J@x zia>JL6l_gm@5c3q!B9N-n{m6&!e2^7_mO{* z)G?x4Pv56_p2kvV{I~0P)}E~!X#Jp_ULUb7fkviTFh~ziSJaQ~22Ia6lWhWW06$gS z{_3KC!9;Fqeva?rNy@UD0+%EYQfu{CQ1Z|>M~^bKz*Q{KE>TFe7(F2&3j8`7+1A7= ziN&C}Uj801)n~3MSm;|CcRSHC2P1KG=u#AQd)jina;O*PnfG)%(GaVapiNp7kr*dW`& znN_32QbURZNcpNV|kIcIgG4`uhsj%!-e z#KlW{2uK!|x=f`=KoUV8=30YcO!@_PY@7ZSNCn9kg6@_-@!F%b4Pj&ku@|RDou?0; zrYIBZ!P&kDl4I8zcaLV`=cn}bJAkbKIbmB#SEZwM!O1>AHnfnMv=G7hdMFm^^%SEG z=-2=Ct_|Iyw-@W(jW;^CzTFh((lI3@8Yx!;OnrFluxoL^`C|JqzVl_iyB2#hmuP*Zy zW%UT=YptKKhGcw)rs3L6m8)Q3jcMbRUM~Xh2Q5` z%S88IqC8uwq-b@Cxf4_v&p!sRfj;TBz`Tr zW@=)-z6w_Z>ywLf^I2SX{*4lrlhv&l@eWc?>caoFWch~L%!$<$zP`$ZlOj~T-{G5b z%w(r-=TjFc{8)IKlGc=?X$cfKP~dE@YRz;n=&uLtjJMFWeU&dnWjCz}cW3udZ66?3 zAyyHDik$YOI}dD&TV{;yzL;+V)(M{1vScdxSYOl3lC^9|Lg0ymfmW@9_YMHB#Y4xr zE|SS>iyRAkE7({Bz3T(qqlX!Q)=+)G?Cj@xV`LmVTO#u2{YR6qK1IAOPW2SWckoSP zUf4;T`*OnE`(UFJ$~yIS;1{cavXegCxsJf1a>t0z_4$jp<`5MxEFD_8jz)R#hk6w5 zfU9TTz#b8ldCm$)PB`H=9)%y2e=RGJga^w!`PZ!>O~88%6dx}qU?eGCo~=cQJ!CYk z`*YQQTqpqfkIX@SYjTr^^KJ(vz#LkRKB;G(H46E-E%5GJ%&TugCX?K$*e&uGy2p~y zyKgIle_@1jNZu)z+v;2~w!wRBVJoK*^^z3Hg1oaBg815{QBUuBw3Ib=z5`RJ`+DTNF zGUllaP}nL)<(s%0D~XsqC#DA{HY~t9&3<@Z*+#H@H)L=UE?gHVr2io-CNgD}A@?-a zJog<6aT!+jH>su+(0paoQyy~S7w62)*`9M4{!}}2`y%5nfz~)a-pEn?n$w;WD)KcT zgkBf{Zm76#jIx#6@mIR#!{J+dC3|b=#`#Tkd+RUq_(%iW^TkT;6mJ- zhz*|mJkxO>>EN)ehpdZ)t_wwOJc8u<*FP}$icj-M1r?Oz0392sUW^EIhF?$Xq6^b$ zh=0Ysu5ev8AG4f5SpWu131$**YLQ12?!h{5ca$?XBF279G}QRYZd{A7L2ix+1zkvf zY(ViZ9l?Hw>NVWY0*Q*MEAJ@Lv;Lq^HCowNZbV`AF zqV|reE#C>445`7awLa7FZ~wIpxh_itl$8hjw`Xp28^)K$@E))JCG#DsHfKC!Z^p~_DAlveOgJI z)!mpyw)G@k%OX0s6*{N(M5Oyvr2x^j`r&C(0DzN{@j5CTn$EV;P{eL`F?W@wlWd>I7c{;UewS= z%5Z9n%bxkuqFQ`^5(9_PwaUpdJblihH(zQ?g6i5;I}TPcLFMs2Qlot5C|b$Eq1u?s zJEyODUn^?!T+d6*adnHun#ya3eUScBYNaIhCo6OXs|-!~NBv1F3ERB4Iv=KAP*bDU zJs!l^zstPODP-KM5DLN+CQydi}7hz_`v9Apry1u)7ap$QRJrcv$lb~%}_ht z8Czh;(9+xXR9PWva{0FjTO(Liyb#4JqWf-bo$U<+yP7vcNR6Gs;2HFbHe0(n;FFlJ zHC!D;#6cgGEsyz)7-Z`Sg+Wr~kVMh0g{eJHn}Ku%#-U74e7PdQ^GPKfD3&Nn6`ET0 zIGR3X$VoO5G#KezrGp2q=Ej^hKa$RFOpbq2r~I@09Xw2m70KAvRrKK5YWfK2E8gd~ za+3F`;X3_iOmDiJQ_X1d>8_?}L*ifory&Jsb9-MhB~+Kvb|NoseaVj#bkQYC&sP*Ly9! zspbX=r2s9!Mben_g%@}7=7`8g0dA2LQ~ej?z_vflypp7FVOx)>d?V0`&v&>vR_+Y9 zvR^;dlfr@43^EtD*)DU_U+FnBeinkFlgpXG=+GJOBJD5w49!W5nq!9#LoLzcbJ#AVr*)Ec@!!=S`Ly;RXM- z4N@80u_D={JziTRY5e!IfA(k1uTqH-&nHsLm~mN^e)-y+Y5>cN@ZWP%zH)(y!OM51 zEeB7bBBQv(>1Y}HrwtaObLFPAGG0v}DlTLC=6wt5-b1+w>WPi?=^1357(jFF9lfFJ z6tt>#Vp~6;B~yn$QeCV!r|TnaUe$JFSyDspAjUj%)Fd3V zVmz(SA|C&4p4p7H_g9>Y1$u_MrSjj9+Z^>EVZ2z>ScyM$sjLihsy2}qzIw>Ag|&Vs zJsm2qdz&3&JRuuG`$W>#xm22hLR!cRpX8!pXc_Fl^yg^#mM=s{ny{^h!{Du>R60-G4gt}^#n-zTamji5HKt6n*qV;4-&qE88B z*<_$*)PAEbUFC0}S;j39IXGTjxPHn zq?ERx4l>*_$47aBL58X$Ei2CYrW%;Lfhz5?j4YFYD-NqWR55v&d`4V5xM0WXw&E2| z)6R!}M~TH!J2j>l(0LppSft{CG(THNp`I6;y`hdhw9h#f&${g$3kM4AS=>al8L({V zqg=2nqX`uVw#5v(AHr)QBdKj_MWcaXa1I^^Me~jS#PT=3D&SZnn{ihRDZbE9z=!q^RNKDha zfos#8w|~NVNFfCd>t22|8DyA}b7XB;8g+6(K-$R*gF4n2`7TTiox<{+Ig1SIgs^{f zRiy{v7=Kw^&ersiq~u->jGiP6&z&k@MsAOnBH!)CaY5aUG=NHKQg)Bzs!BW95U;Eh zJ@?*}LKc3KTGVuwyZZMrJ-oehCB@rf7ZKTSn0NCal4FlZNH(}?$AFeDW}nd{63?%P z1t7K{BoDv&CMplrq8FzCRpygNqL*be>KM{J+zFthR?YLi$+j>}+C)uzpm(xUkS4Msi-_AfhQ4Hef&rQXInx0)ug$ zN@poM-FS!1qe=};y6u*^OSbLToS92+=F_Ms_OuF8$%7P(oEJi-X*beGZK z?$Usx0J7<(G}V#IX-oE6#NK5jn5wEnHIpy2u~Oc~7RBYk^w#v}x4Os{R4EBHc8~!> z1>cf^s^Glko8i8Hc6$xkvH%oBij5@{#sduqMS94hJkeONTXJMHC>%y^aWv!g-Xc`v zW0Ne1c%LtVWzkk1;$%72ihjLBg_i?ffgglG+xR?&cKL*`^{0%$)MmD#fAk9=K+p9j z@QDGb`+v{?ICInIPYbTrrEY5Rs%C`OW8fee8tx+JHCp-#h!7=b->K*&W`r-R5AVU` znb6!I>^6URXAH))?FmZCw>s3b@DheM`RDHEYH<9S%JI>?e*83|a zq7)x1^i5@F6WHgT*H@P!9fxm~smzMp>H1OnRh4c^)a=O}%Jvwpz-VFa@H(uDEw$=R zsGM~taSu5h^SD25FOcfDI_LiwIr#?Jqb;S8H)Kq#*>Q4)o36T)AVFnrM>ba`r*N ze5Cu#0lPSj3vM7iO8?NJ8*kP?&6@RI?hUr4oQHfXFnpp7e`i|R?**CTR+wR>77n-X zxsnJtxJ>YEmH5sA$52g*(mx*Bx9Y81H|e%T$q76Tsv)OTkFkvUQ3{yLPCF2~efRXy zo`i?iX3HvcPH2Jni!Pt)cb`2lF=Kmd*@SfhfyE?+uZ)RiQRwQ9tC5m7o=US<$`Ho084h)#fdGqE*fMhGuIp^pt3)vdcb`0D z<4PZlQj!0HKJU`^NdGM46Kq_!^SihNXrgey#PbnslZ{z6&ne%Kl3|vZtvqBivS~4 zh`?`e);=qh?3S1@4}vd~a3-)J_%q4->_vobj8c72;ceWZs(6P{CC+avH0MGY0xM9q zAwL}zU-^h)Iu-S*j=piN^PvT`mxocari8j7trHIw`c)9GUYcta7>G%n@XE;VaPv5r z6flwIW=5ZHxe@skk7s@qb56@~XObZzOO2nnkFGwm%ox`r|1r9&{U!ZegO}5l*E8ue z`D-J;Z60lugx2tF(y1r72hwNC#`aG0#IX|*C|NtHzrr48BQegW*k#mGxo=bFgUc6% zj2zzA$1&ijmM$G{b2phKc>BFd`P491i&GZxR8XnfMkvd>GMzjZc{G;{y%h0^Q;JmLS}uY;*@{sWd-FCVuLz7!$kfZf4W3i zOF~Z?Wgrs5A4=Myd^E_mU^ZRF+n8J((~8FY;RH(^yh^6@0p?rzK(5A@tDxCKNv2-U zSd5yAl<`x70C1U^D8jE_-gB;>W<@zX{a1$>Q+< ziD8GI(ypUR`vrZT5!2p0j&3%)=c<$SDLG2NkvE0$J~6FNIA=*Dw~@#X+r4Zq;Np=| zrIrTG*;aHBvG`|SRJ`NP8{H!9c`WOU$|7^O0H+gL_=aiImA^e}`< z39N+FhYm~T@$z8rDE~g9LfG=??lWs9{OhDxo=L2(COG+W2wfFHHP_=Vo9qFs(&?T! zxH;?)9*6Mx;%TVlsP!w~b!N+?}T!R3ec zr5a&vZ>_gK;3%Ww*o!Zm#Lk<00YFO|E+6%YkT5vIy(=%Iqv8|N5RnD$xf-f4ZyEy7 zd!Cdv$994vvxsH#hvjoxfPp;OHJz{e7 zbM0C^tV$|eZdu_>x#4%;oz=71G_Q+%YYHBpW19I*m;c!ET0pl$-Uz5GyA#r555L>% zphe=mU`kAJdOgwY@?|j#d~aYxn8z8J9tT!S2kXvxg?#(DHs53s9;NwA!Y5>d4e4&q zA=rJ`-V!T2D%ZI>oki0&f5e9hboqzmCyUrKZ#|wa;bfnKK#M2AdKMD2I)2~zU|DhS zIFW**jUfl>Msh@H0EOzM9xxyJ7b6;{awF2cE=a+G+RFFKK0tIB2AgvaLUy;Hxw(-CMnkC9^-1Z9KSiG*t9_6IhxoGXu)bh( zeu|Lk2_K=<$Q#(3;JC-nh$s_-S+;^)C`8I%+M^IiE{beQ43yV~22vE}a62`EDX4v5 ze7_2x+gZr#Im1}Y07p&ciKRi&=QfDhaSNuH>y(|}Dl{s+y6wQ$B<4_+$AC-i5hO%f zJ&qx~*x*?d{h7vK#jlqXnA|n$@zQmXvGo+5LB^ZL_XA?Zq)vU0X#uHRS^(f>&((mh zQl~g^I3Z@t1@Kqxj~@We5kIy`MZ)osiU7mRfflRLQ7OhC}p$62{KS254fZNq+(;{H`3 zxmZbwS~XREGSTj6yc+=b8kkJH;ah<(pRCM2h#y6@9}(k8%Z5Wu#F4+KKsk@g1Ee`G z;-kw&jZUs5v5y58hm3N!-|h^U+vGc#0cBe``GrIGN@76}s%=r%j(gt%3{AP|M)%ci zLZFJMl3MF$xB0rNW%px33G7cFvw(fKY>D%3p}aKh6W(v(%Xj;f#k(bd7axXMlT$A+zUy ztBxv7+ArL6a-t#(d2kP2=3Ajd96t-`Zh2*j%40n7ttYZW1BCaR2rYv8x>3WA;7!Zq zvk}Qa=i3_-99$)~I@h_#t7`3&Dnmr6IeVaFRGi*hd_`lApNyIIm2GJrOwUOl5XY5flTh1$6r?S{vqDc`xURIc z&6&d3XK6)~USN^6&=2T(4;FbeaqssAhi+d1Eb;s&icJ4Nk)|@amxoVLLK$Gf=p)1G(pOKao#dt_ zNThajl_tHlnG;HGlqlQ7^Spm8YS|wW?E0%)gjwljQ>gy$^wvvW1(@FOm^=N=f>-EM zE2+}n*(y0+b4>Yel)e%Oe`A(OA?2_d;+p^9t7|3R&H`BXVrt(l&(CveYZ*}{G_e$ zI<&Y8jEF9hm2t7jrccumYb3*2a$tzBUUpja`M&@}K)b)ce_bey>Z_KK9JVp-JK6J6 z+RH{M$_L&O)T_-$@hI+D|1s;EX;U53@#EC3Fv!pQ+Bw5NjJCsAeYS7g=qY)W^$m&y z_t_Vw+UTgKiTP7_7bJGCW|KvC7)*=CN;ENV6V1=r*fPxLflh54#AKeyZCIZKb zN56eZ$mVgW`yxgWag;K`a{26scLOI6-?N9i5l^Qt zK&?lu-DfoVs(>JQ*1f7MGof0dlCwTi=-R(SuD7q2T*)DodrW&#%RR|+($6+YsoFcz z*O_-2S3rp}kFD>eHt=#SyYP3;W4x9zcKXzYJo7 zerIN@)ar%0YpkT2%4wM>2{lge6|qpn!tj@kHjs>zv6>U}c!a%+D8kZOZiaqjVoAx3 zw;p~TU71Eu^0bvQ`KRoqUrLVaNd6)_s*Jtbbi_k*io5^anITl3YUKftl45_W{#RV| z-7aa1#Xc${%0Q&^rNhWz-WSRp^8CjjTlkz{v+%VyT0g`iJr&1pUO8up9H^)!u!E4a znC<5e2G*dWh<=b0$-{2i0OU+sElW+V!9Ek6jPMVBkmo9kVV z+r3nR5NHOOvCuDci6N zd`+?E(@vWs&YoFj8Lw;AKVI$a44GAHUnxo0hZqbrXYnzkKhmwsdZlja@2`g2f0LaH!~=@0M}nlKXF=C*It54tSwqZQP(d zx`|9kL;r%=wK6dmFHLJMT?_Qm#8){8W6+Q&Y062*YcW?ntr`)z>^zF1Pw#hhFk(Z6 zGO4exsuttw424gm%nuekFHmZadDXrh*jf|Zs|{l_O8+^RO3T0nOF%U~Gdhj4+?uu4 z<{i*yix-wqzOVBTOZuYrKxaXr)Qc^zWO}yPMfmx$gGD_4voboIE}D_kXK3qI>1p+V za`UlPpRpa=et-AXp^TQoJ!3|Fc*)JLbBhbM0n~De()*$>;|dqpc<<4+8Jsdc#pP0m zPOh>gCwmhM6-8a6l+*Gt-vk;*U z*4c894&~qUDtC8%pPa>psm{fACWUF&lcEd@%QEB{ATRqyQHmx`*YWggdxH$z*+-Mi z-d4+7$!&^02U)X_u)3_r$clrJzHcL+9jN;=+=i`PRCueaK9Gh4ik!gw4B(XP>d+x? zvWRNYwE!A-7(X^Qa-?ih7mVYw9V8J*3dv_?5`j$IP6;>Pf;rT2GV!>{4W%l1@UcTS zj9d{S@{g!u-!Ef|c2-vs#zOy~Nn*t0rzoh(&rL;v+)gsRRtr0gZNKK)C(Xy?&^;@R zJN@ zk(XaYU4+=|%zTn{IA|cl`)Cz#ZgC z)1pTUL#T>Tx2LGoPf$PgKG(pzp?JM^A<$`-J%<~AJ*_45NolQ8P*5uDik}u{F0c(| z@0|cYxq%`3=lrdSr}Oofjh@e8bVdy5x^+nbX#D&Rb|oelP=q_T@(kHZSR?LbLhCTp zOST#yQyv&xoQT2MT*bJ$39_xn(e0b8j~dguHh5gbr29&r8-^YxueT{CV|F5b;=s|> z(@Bg|U&B)6DaQY>JWPD%Z{3DMBZ{&4UQ( zdaoOpH2LTmkQm9!@j#or&yP>a?ym7pRJ{p^-L+SbZdS|Aqr#O}iP*dpB_ouZ=30W0 z2ED%Z{y2ilzQ+*)gcdl7t_&9w@{-*A?-o&V-1^=!C!eN&@_Ub$+@srBKuae3&ry-iP%@B7=+`!k6aQY}f?Ti>1p2{yi` zs?9@*R$cmhSmH@niviYTQ7Yz-9M!=ukwb z6%c&A=oZvE;j$r41G3VQ1s+O?q}pH7hxnvDXrv13E2`vbB&w^dcS>oifj`5!IWeWg zY1iRpys8xCTmLY>4NUxsC8`JgCTzu~nqwgxeczMfT~_G+uCg?Y4%4yROhv)N{;w4r z(A$&kTvyf?r9N+HfeJOZNosfD!qGYWRf14&IzzWRq?wdFkZchiR}7qCcafye%?~E4 zExzp~Ndg>xvVwwGykC2#u>$$RD6brLnsr{Gm(L7?QP@^@!?bteBB5+m2pnxm(e|=F zhseiU&Q~q1VjYYwsaS?WPpGQHj@Y7$(1?3~_&Lp`{eU^bgI7dVY;-ueJ*7@h#ta z;#Cl1tKH&#x(91fi3XddV92+oN0XTQ0=9=ZYG1@%`^+xO)1PN4PbKzIR{1;fDeK=O zE72vj5Che%O`UjmQVRN6Gw@^ivC1O6Mwp9n-VuE%M5~z`<)BM!PEf}*z(T*n4qAUH zb#Se=wZuDg8&XkyQ}jyekXMry9nhdE~|Bt`r>WDgGY+>lg@OTFyK#xXbQu9E?hvK^TS zT2=|m+lftjLymUQ2@`*k(_@kf^derhq~s+7RW4#;KD@iEHNL%uCe@T#!6t_DC8$f1 zmC#kz$jz4dFJ7)uZy%e4|*?#!TTBK+#@KrMP11H0W0Kc0}O@PcpumqBs8dm z@Q{^4De0dlZ>9sGN-PnE1v~8ZpxbxN&-;XIF*U4x*^BZ5v`PHGhRb1FBe(c*VWN#D zUEZzpN0F3PCnnUw?!~cPVVpJ^;Vi#gy!I)RtP`7r{lr#m{qB27Wk%^-3q7;qQg@E^ z{U;;&RYK|GkGO}A_`36*(H(pHjFK4iUQWi2eMdeGt~l!!X~+oko;9zOg5u!F_ydi9 zer`n`*DIRa?dp=7j*{|2O?WZ_L)m0yZ|3D9w#DEriNgLGf8<|E^6CDZ%A*FXs4R#tm~OpT_wH!QgdVQ77N;sX09Gau{OWodQMa3$lvuu;rt* zF!HyUBsRZm4Tgt#J!Z>;hJR5{=+(X_dZ}h{d^sgVz?N!W;qb`Ac zKi1Z20`KrC5oi{yw%4pMt*I#OPEPi49jhEdKbmndmBg5mSsco5)8<5A=rBL?CYGmk z(Q%jNyj(Jz>c-=tv@w_Cn+=sL%Y(y|m9^{u*EOj=RVJQWiA{Sr&4tbk%Qr?_3Ym1gBX%y|A7lr;@7c(1(^^|G% z>SvNj@|0gJ-@;;I&rDL`bRH`@!j%U#*u>O|XZ3=Eh0uBPS-XT`k|Yn-i_ZK_m=FA`gN}M zhtqA6d1f%8UQ!Y-tz+}{z`_v+by|M^8a`=1;s)&fh3VLSxIYMRv7wO@XX}Ug5~KSD zPwcEeR3=te;2!jgx;1%Pd6*3tvc~DV-O58Pe|mr5T}RwZdfLgxnZkdpf4+QW7B>aV zkt4W4^Lk{*g>&!D;sb|bW>!uyPZjk@7FZQoao<V_L@SHvzD35hK1bSnmb1#2`}HD@$vU*~MY9GjEo<3;inKHOwOZ&#!bV2*!y`_- zZ$GoW!K~>Q=u`5wQT}Bci*peL#zAPZ+LP0JK1VUx4!3dpU{6?j4;$ej%tvkf%$n7T zS(hvI4q)a>SXBS7p^)L;7uekbn7fo;leA(fjmed>PZzr@7w@V(H6y36^IJ}FvE(Jf z!~DHyj7!x%x%t$&Ob{2Lbuif=zWSkRBbO|(YyV^{s2cf^wbK;Yp!slAdC+OW!IW~{ zK)h`eRf|)>{oNU~=X+UwB%%RB903x=OXeZ8X|QH;LxIz+vnJxi@(0Vyg*re<;l{-* zEG4esBV1D0$^q4EgzWibaKGx7{3scU-&08DFgN^y{ueE-?(y?QT%m78<`0y!R|FHN)Av*C*+rY;UkP zPfy^BzAyMNY(c}i zn<~?$wE8d$M@3Gn_xxC2;PKE?h0;H^d*Wy|zrL2LiZ0{t;AO8ZmKZ^tplNuY(nQG@ zAF>03SYuTxHWIjW&v*I`?6)9g4~Pc1Ny+8REM1oB_8GtuGx68{*l?MQ9pa~&+D<_H zc&uzXs^T;){XHu2Ml;r6N=3tg>fxGYGQXl!WCFuavQl~LdlAe9b$%$`9JC*Of0=n_ zl>+mdGi($dC|QEWkitaQ7*81;LRbJY5mfaDv%`hq)-=o&CDMyp;~4hEH*H)2Sykv> z)8C+C4OGetUvt0Tvrz#4D9PP-?})aQ^WeR{Qf6&pvboZPew$RrpoGpmluHDlC)t-r zVj9-$fl6C>@jL55$xE?AFMx@ZwBjhWpVA5H=3=s2U7a7&FUz9vxp?|6RjLq0$!>lHw|O&RL&k>2b*{L+In7e`jz&p` zjs?9N?=E_bf;;D}id_ZU)!4$QGyJDnB1XWIH4Dd$fXZ~|>Y#x8Yt_%M`+ zOq+wFs}4HzjFLjA^sn|sD(toMlkd^;UycU7XR!3jHM5;8itZTL6SB@dwLoE5e52N$ z7DoJ~cmw5a&Z77Ty2-lm7_WDD;r3%VjF@-fps-iLKnQA9MS_W7ND<15WeH`lmUXW2 zk8en*VPA$_CVkO2jB7UL&3_@)l(SRFlHcK{V$Qo8DlAJ46BN2l!Z4)29g1AQ6TCRA zDqgY8z9ET01>)Tu>AzFe)AhSokHgUQ^#5Q~0`$;X^zJ_5{emux{jT3n zD|#|E?S>Y7W|ZLUceZPQQ1~D$V#Y%I5y{-ZB2f(!4RJJKN4ywoM*O`(07Ik-j!@LX z3~oO{5NCvzX1a09Y*=tj&P#i`3$Io6fX;3HYi56I-3{#bdy|nn zzSpT_K)D!+pHK;YR&7R@y6>b!p(9fJ5<>NA{Y~DAUr%k94E4Jy|Hf~vGQvu7Yl(pm zK4$o*7T-YrH-}rP7O@#>&30|cRVeZSRC4buWlP9C;NA*+>L9m0IABvgk{#_|xNYwp zZok*?F}keb{;C6vJK&8M8qL3`0Nv#m=8YV;kg~&CEvuRB^HNq?x9WhrY+WgR%iyNSD799@ zxfzM+eoi_cYqhTXJ^e7RVe#w2WRIdNA3K|tFW=J96w~3VsQL+~!@k?&jLuwKilx2B znNT<~H2u)Zndw$2BqQv2LOGrL*8IKE;U>0s)wo*T#CY*Le8uLsZTwK0fP209Mnkf` zR}RmNAAbg`=G5Sq<-OaqQH`1pv|;AN82EY`?=1EKR>IvvgVD(PHH))k7>r-0#ic|O zyA#nHfC9G1{w@w?EE(M+);S*IE(L2$M9vJnZ_oVwzLgt@=!&K`U+P{hW9J`466gty+@v5FGnxN7ro37x6QMvU+9Ii==ep9qsjUh{7KRIWvOTEi zo-a!imAEkypAwM4hN&8cfmY$O^&i|(o1#57SklS+h|VyMgqqC6!V*{lL#XS6i6)7M zbu}xgDqmZj&zctMy^!VQeS=aR0hf_9Le+djhm}tsEI>5#o_n#tJz`|*qD6onxS2|Y z-;W@i`l6*^K9B%~m|w3^r#ZT|2UaR0|W5vkY$>kM>mpEn@#Tk>j)v6_9pHX9On)m5jAg8k@?;^XZ?K!r1uumd^uEuIh& zf1y43Pu*v0O0h2ZnW>0igmrSBZbm(x?B_wKT&Mi!0^L77fh}rc!_PZ@q3%*NT2wc=4A=q}y94k<(+Xk|M9Ey2muejuy5l6+@Np>N{gt!sS_q z;B)=E(;i#*ZM2awrQ`D){LDh#UDTnadFKzeC(QY3hscet%0~LIqGG2YC^Jr2v%ZWR zZtdn>_EX{F%JO>JXd+G7Ugxi-?J8`;9^-vPKi^#GE6yJbJ|=*~B4=8MfeqJE`+tfoCl(`)9J zv_O>2uglMx`do zX#?&f$y)Zyg`j41(r<9xVPLm6lZ=zzqN~l)g#1iI3JI$$bjI2yd)>Eh)JPYC!93MM z=0A4o9KdY8?HmYYWVs8jy87EvD5dD~^K^-<@Vnj(~jCYp=((o7@LB^I&s7*r)?+$f@d z|5NKAR4#)?P=_>maO_b?<=~8#si6~11UU|~+FJ2g@@r-%aLb~v7`Jb{*uKPRWA7v$ zzvwm3Ac-N|WamTt!Wdr&3L>$*m+6Pu%fNyF?!FrYyH1qQYPGD3A4sswC5QFcZmiWB zsc5X%drd)XE};t(_j!z@&r*qlt;AD@xqR%cFc)9F#>1L-W2j##CKr+~H(@qP`j@R1 zgwIkC1g`6a{yL-8TcND`4tRuLCBgbri_lr*>;i>NqX>y9nYNkhITZPr zF~{c=g_d264h2*7{jrq3Sd*7Vs&2`tnR8Gk#vMO9%+FCzjN@dc6%#XR!}c%2hd}NK z1|`JJ*3G#uu;(#3)HU;Fnov(#YS}cn>g{Z3R(ZYiLyM)vetZ*`}1x9 zH+bpQudFBMn{PPt<<-I%!K-4Pd~Myk#R7Hjj{5@w{kJ*W*^*ft3uW?jHnRXWKLU6p zU}Gf`@5YA*>6P7Jl+h;^2Co#C?AsrfLd4iz6sN>>Bcqykv+y<7Za^ck=GLrq9gz|%HVQ;aJ`ND1ltnXbDT0X<n@h=?ak ztiEvRi*%SXqQ51gEpzJGkok=FL&k>db*Xo5e#&(c;UZMv#g0%%(#Sp`yi2AZ%`?77 z$PVuId%Rv+kwNza`%2?D97M1NB=hr>@9K+kK>@4Rc|HqGmES;WRRrJ)xx#52sWKZu z##{eLuUOjg&x*dMrpVa^j3wu#TdapKgV8j}_&3tu)RrFyUNUb{cEFr!yZ6(iXNzzX z4xMMF2_o9onr=3IiK{Fo`0s4O8deMhSlE&R4PoZd&WJU3jmz zmr6wW<#0I|UdSkL8)3%u3WHoLZJTorh5~r>-#<@R9<8*yQwk=}gOy36&%+}LL2k~F z_J@E6z?l+9C?;_dkTbsaPk9fWv5DuN#CKhY4tLLw?7dOI3$7H!tvWA~IPdJ+u~B3}T93ZZAN7ngu?+K~ zVo_sTbEG@)_EB6Ae-FPR)!Tg1KlvvKWsUXmb%2T%`;x(b{We)T+BVrRUI1uG%HH z1^~!Nn4-3nf^cko5PGUT%r)m37R|`F!LjcZ@_FNXCVlmVY}4OgS+0L&(j8YM9^m3) z?D1-dmV|6c70E00xzDql`&Fv6wiHv1_ixAZtT~0*RAtTv9*tZTGOE<0-Yi1e9U`Aa zqib3l31k(!*Y;Se)OC04J{39IMfuA6YVW46<`00tL`EtO^c(zZm5qCev;)P0R{160IR2Ne4KMRxDV{dKVu+hXTON`U5{&@^I%jRGeI2YP>Ib?OrjDZ1hoyVXmBOH$pM7OvHrbrBE&jC@W~~J-1>&Bi4H~$a#FW zb_~?fGy*RnqGzVGN_R*}*AQ(-vzSwF63r3kI3Cyt|1R#;Xo`}x;-XH&kamCwZCX4S zrL(Hb2dXPMI3|s4qie#FL8kmUOO$WpbdOlPyyt=`a8nSH`9B6ExjB=A%?;lq3j!3! zt*Y{bfjtJthd^m%y%;&kpLBw28l%y%%BL~Zs-`}mQK?Wpu-vp&`k<)eTHiQ{M_vL* zo(MjOn^o}trEIGJDxs`=_zzygHi{Ix@lHQW75!8^6ESpKFPKN%LO-XC0po>D+{BK-o}`r~sFk4IO$L)shS1nT=|nIma%M}| zVlKYnq~E{PV5*=iW>6Th_Z=3;-F%%?7<=~n&S1Fg&mAwNOPti&ns2g73;R5LtHtL% zW-;A*=X7!{nKQI2%+YzZhNm>qiUNbXOv1pj&uApvSYN!4@UjiWnY;Q9`MAySgm&mnec8TTR)EkOGOHRSJe}2P&CET<=!_ zM=xZ=*p|Q>CZO~#rKf9Jf>62OSj3hkBf#$ZnRN{cl&V!ubf$G`8u28UIO?Wqv9kz- zuhH4;FH;v~qPv}N*}t7kEK7W)X1-Pb!)OV7Fp)a0ENqN08LLLlzoDD@IN?o4<3Atd z?=JVOmJwfU{q_K|w!&Ikn(;D{a5G1cg)qur9A~DU?#|{DNVgZ{6~$m{i|ckOhTUR7 z^h|I1#X{05R1z_Yx^i+x6I6DJas`dzHlSxe;`LTsuQw1G831ekQuwok2~tM@g`9co zbQ;`p$y}e(Gqs+15<{7c*sb5gxR(}3bly3;-n_3}^Qj&%h(Fg;jssl}gaDGXtWteF zGJmBy_3oiWuLSA)uM)$m*Tgw0i_^RA_*@86I&Hz*3Or0yK3XqYL|xqeVnYe-TX9bg z4?enM3r}G$5IaOQziVCZ_>0QW&Az(a+5WF19u16x{!Z_!gT70wNa5FXM(*0l=i!4L zuC>PbyGpk;!k3(+z@cfSC$Crp6Kjtx5gKxx*lsu0zM8&y27(W)kj_)GJeOSsK;8dK zlZVWhm}Nv>hM$jc{p}rwR7Mf$pz=te(+B1Uamkpiye8%QS z4)r}S*Gl}zf{%OB=P6@Ev@Y+41urm(`l|4npC2f;bl($fKW*u)b!iiev|L4Bc! zn}Dt0nDL!7+|qPZOxNN^5{)cc2A)+Sw-XC9==PvqgT)5C5eFJb{;6FU5i0-ZL(EbK z_r<^{2C|;OY$40y;z}j>c3U^`E~GFp=^*!e07}EpE}@tr*{G+;)}PqHbiYm9aFgL} zo?f?zSn%WtwqKf&R&CL%L;UO$-ZKz}f`CXj{(vI{^Z|41q_T<`6bLJ?ijOaG#1`j( zjbj~f7G6raDI2wXR0_1{z>d9J>lOKvMBrST=vcCHW`?ulzoL;RifVxPZ~82F^&>9= zatQ;r`V%}UW8#eLc$0w_mc#gsRRzvd#aaS54FB0%> zmqS~FfdO8as4Key3{U*NeHohHYEq3rO=zWZ)*F{>V3*h3kSh!WwKfdZ-1N}sk4HwaxO@G zZ!&GN5d}@h-8n>j#$Q|Ht5O}PIgS03>CSW6li==N30;0w043wt3)t#7G1SvPnW<)y zgCOyU^_P4mD%x=5Ech32sLEm-l9lkeav719+Py<|9!B4#l6|AG(pI^Ze6|Dj>VPft z;N!9(z|s!;1`htRITzR#e!Lpbk|xsaoSdFr_vL2Ta_t2^nl$+~#I-zI>2cB12tc=I zzbI<%BrzldrF9Xgmc>c$^R}od>_3L+ z)?8NEAQY!>!PpuCy)3zbZpy%8W&S}VdKr7^J0ca0yqF+)JRp;qvRl0c2)cG*0|hR1 z$U6x#`vWrScST+fctZa-6K0wQ`Cd9pUai2nQ5>Vt;L54sut*)Y$m)fzwHH3KVfyyY zWcU2~9!Y$k8=$HaC((D961f`q*P`?4YjNoQk~`lpGqnN-{zQi{Q@VP0)GN)6l2ymM z!gp7)M%o>N-u}dO!UztVLfj*-c-Z~%BWRwBGKrHbvfTZkDI-W9jL{|Nq%Z7vKKT^9 zXHT~?Xy~aTz;q=$zB?A(*^t2*BDwny5B#mv$pldRg<=*|;$%B;vw(lzaGQBgY|n7b z8O7?h!`(;Y;P;*$?dRt~wZ7xTpzZr_hGbwYF|J=};0^3st6I{^wk#Wz25`diZ35hJC zkd@VAakU$-3u_KM^*x)s~+WrQpyyGF#c@5iy<{T3W zb^DK}l&|NRE~E@LX98O+7g{$`u=#<{hA1#9tsDwD_%ZmmSeNjKBi$Lq%mKul@$Tz!*R|r8OBzp@PD{TW3)wB! zHBjb7t}B1VcDCLT%cL zGyD^#OqLfT-Ga-lCzp!X+MvqJ{njQG6J2wJ{geVpziAcJ7S;{bj@_%IRWdKTdH}@4 z+$ncF59f!uF2}N&e4%vTC(6nG0%|Z_j~hG`2B4^zVu<0|R3`|rRAP00PvwKe343FX z3`dUwV$F<@=yc<9W~n*0CoZqoH3wZtvGM^r!VCqG?v_-vTi3p7$WQ{|pNVN#93?3T zaPW;W+>m;2pKOFv1}4R2UL(H=P`Xb;{{TGUzni`HRh$7DRoDX^m@p8L@W&K26qgU= zDUR$DZh)oLuzoID+dZnhkWKoH)$(h0*JJKKbzzs+j0krc0vzYJ`Bn*qbUxU(&QDd7 z%i;$#;9PL0*KS`&z9KB)hooM@6&L+yCIOVdik-U@RVHmp~S<=T)l&KY7kMwD8)OV1dUd3#6#i|X{n=_d3>#&8B1hm6Vmw83sl2b^LRLQ4)D~si$ zyodFhK`EYfE9x4lF?RA+bx!aaS8zQq@BHHb@uF+SS}Y2qPtbKb%%k=J%d#=e+ISTd zOof2LA@BP>(Ow8$*;}Hs%Iw}i`E0z@P;^(;3DitHA~tLqO6Ix9>Sv2KRkgFcLo}gh z*~x~`xN&j4;>PFd*M29L{|f=>P~M2t5NZ2t*0Ub@m|?tfm#nzz0Hs$s>Y@?8!VdjGPFUAoJ2_-MK&EKD?q0 zRm(mVVHzr1#Qi>Zu>qaGGcPDv2v-h?tP;ZFR;C_S(?~IW4jLtD87tQqu8bd*?7Zg- zn5v}Uz2zpx^VEFU3_iIyil#{eTlQCgyR{dbyB}n5?2OMLajB{Q`){aXSHC?;B)9W|gfNGwr&7A%D*hwniJNJtk^bzOCQVOnG-wOF5$)#6JB z3(f}Gg+fTZm|$4uAEa#$N6@Wlh9m@Urj#@!s}&qVLO!4T`aYaFw zbX z)1P5hdhI=*M`#G_k4jHUtEZ)CAe+!wPS=*jEFO-;=hDN?Bz9Wr9JFBj^4}_|O@ysf z)8G5xgvkTn*i72cw1&59QQiFjy)Wq=L-mBlW01>$MFD9(Qt=HBKW>W?aNav3u1%#& zvABoH&R9|>lK$Vm=?DDJSC?fvX+s`h%@a9g)&W%eLuuN6KNxZcp(1|&hNs#sI+*GZ{*R9uk}5L*@GqN#Qe>fMOC7 z$aAx?#|&4~c;3bDO*H#8;5+SheAwXD4xKyrhQweWx4s`i0o#Cabu00?3@DX)Nlz$LcH-Dh(hDE8nbmvwpZMqBC4GrlS($Kwf52=2m9D9 z6{4g>^d~uD%Ghv?E)+jrd)^&rVVZ^DCl3ny&oBwuemoP?h_kmW4EFidQb0wX@$}R* zEH^VFsk}TSkfB0;)$!Y)FvR|v?Z@^%mCceyE%AXXPfL%dVKO#G6(vr+GJ3l7YmkP9 z2L9;zPtC#Pv;g`Xus6TAOQ|u*Saf5AW`W!Oa|B-Yg~lsk1}@gL_l~ns+vy^#s5?#= z!J<~_9egszy0*nF=2@XON#BT%i5QXzKr4I=_@FNHOkir1)#V<+N3YE8-Q-~11jXe^ ze~7uTWy3l6LPdtVAmN+{tvGljP)PD}=l%T4#WJ^l#2rJ%=F> zg_hVOF64nw0+|edjQWXfC!8W^i9&9X(>9Og6#~vR(7CTEvzN9yQK63{<%cQnVOcY&jJ1( zWK2&NcOL^XOV2N1Q5x*oe4CQR$C6~tw?sbOOcF5NcuXqCd62!*E@pnM?{6rlsFUes zj+k3@8!CJZd+|W1{RPk%2Ebo9$r%5TL4DSgIJp8J5+vP2Sk?P|kY^7q-nU=E8-e*P zP5+_*;ZEhPOHObE)g1*JzzvqwRYnC8JTZ&PV3ON&0W(02)lUBKceMYmYvg?jpqTS` zjoMU@jGXK~TylP9s(YTW$xit!6!yLaK=*_0{m4UT)Y$Uwm={75vo}v}HvOOoDlI(w zSvWQc#r~^95z_9=HfE(kt1vHAujGOXbW~Kr>o#zPd6;I~B)aNM3ld z(JjFjGXIxuNyQ^n0YtD{N|NK7DbBP?U`}YZEY93T3T3W~q?EmPaVwPje&78TMt1YEkB5q^P}s`WL5c*GN>s#k7Q==BdBt9^)}AF6FJf8M zdEiv2P@MZ16ABJD^&zY+P*vQ9KX2i6qx5rbspMzGqZgn0^3KY&XmCaI;zCKBcnMvj zb47&%(SkhcQCMun2Uo4kR6Unlgq*YYRe>@jDw&?2+k(wD@?roI_oRMNP+b*UpF`4^z`95iPfB{UN|NBC(j&OZN3!54TXz_3%uQNiy??*AZ=r23{n_2Qu8(uqzMeB#&!PX z+-8-nEcS_nrthXi@Rif~-BOG{nH|_xPOgF1oZpb|axd#&cb4$l%nEg6q7`5(N<~Gt9-xZU%HO?Vp+O zk*mV7!Cz(2vii!m=c#kVQV`t>nhmX!U_>GUUOMl8L?~&%mML1+R|r0ioZoYFm{HHm zaW%L});sQ+vb4L4>Xwza!tzA~rfTyV==s*wqgFWfeuggjb)y);9Gu4MpcUeo8r*yb z1I@U+0kOd)o;rR|kDj(fBZ=^K;?$fCgyjkVhSl}rHd58}%#+dE^iaAe^!fgyf>eln zMmFN#tnO-hHPBUG_?4P3W4C!FETr5wrazb(FSj=3L9){Hn{Eu09-yOsmXkEv`Gm3s z++B^v-Vf#ESy42=)4R4X`=oE?~l#@ zBM&cIyx$#Jz|Oo|nOBnCTQIS2M963&W|`dpc=sI29Ku_YcR=C53onT+w@eCqq2ZryC{ihTfH4>rT7BuK1_e;$u_G7pOkd>}k`h!+9)z$e+Qpfhw05=5!mn#TlT=x)gltjapGAO7^x4SVa{5CJ{3IhA~fJHpExaG>dP4!;lw*H?62|3Zj zHm**bL+0m?S*_tf9VKGoyj9T~v^Hh9t6WL&{Z+RWG+eogf8 zr9UEJ=xGd+0i=u_wwI$=*r{tKaOEQ%dqlf<&eHno%~FSm>>D_E%u^UFvZU}zEn+A< zP%g{=b>eUniPz085La+q^w`F#q{-fX1O;P69=8O-$)9Lxtzzg*~`n}@(ho+*GQ$FDY(Vi+A}dEtW~Lb+v%&1@bL zsNTJY!+yTdyeechGZI#oR+pZPY#~j{e_t2)UUH4kv&*WJ!t^K+c8 zf6!b@Y#3@h+e?AsP=hi}kprdJPkPW6p+E37xEqVEVbT8IW3*7La(#wjERopUy3(~v zlj;FYHSRb<-Jg^_D8?1X)=dWlYMVGA_->15XStb4RLq9RV|%oo(6uyjmV$bI+p`o# zAVlgl#Q~~t^C;DU7dRcFIYA-u?hT`2K{fmb= zxom)Ydr+H}1wh!4wIA(RFzBpiFI}qPE#pZOO3@*KZUjouAFD`ZCs7v3dHJKHJHbY+ z8zviX`oCA}Q>neKuF$7^U(X_;lL+Z_UeJ@~yN^7wLw6|rx#(L{q@2fCO z)){0kavSvV$AbO4VO2wBO85m%KRj0H(tx_U6cEhxN+Q7P!@Bqx7}tFOsW$TphHmyvc`$=9 zH3}iWP)z?k=9#$b;8J+!W!$B_#V8kaAXwxPhSeQW)|I0vc7DlbgSHq+c44lszTr~MrnD_!|oUQl_>Mx`WCUY_5o+ZRu zg9;ev!ZkCA1w)i*IPeDwzf_bAM=Y1R^J^R83*g9)^V!CV1S%EG)|en5>YZUnDdV0! zF`J0Ouxm<}>L|WWA5=n`Co6>Qr9^E`;bMOsLqFZ&nKiaMWKKd+Lt2wk6B>WbV>rb) zy$h;AZA}~7?D0W0SD%x}qrLg;@izRD30!f6zk-^Y9&WLdyJU=N{d)HH#2h3uyg632 z5r_SrLK%e)fjMbd60NV=HsA>t!TaR$EjS2{#^oH}$q^rSyZ_h(#RN0U&`n0`m7JTz z^*sNFI(XFrYb_U$J(Gs=mcGQRNVrmpuo%JoOw0tEr?k4ZDIq(?Vd%%q0S9TiF4e3r z#^WjWrB?~tpIaOzk2PwL|A1rPW?a0j2zs$@74l zos8vZ>jM!NfJffNGjygZ@CX-#z{r@wMQkL9jdSJjG?16WlMbc=(($kG6zm z4|Ye_2AMyx%j@G3RhYNQb2}|s+w_llpDIEXjIg4{UUs<~MgDRfL_c~?qejf5a=rod+6{8_kvrFVzx?yr-VgX>ogVaUtzMP1i3 znMD*>o2-&@6pfXHJkEw2%={O_(tPrL1VpN zz0~q1-{%$g#k~GBTzlK}b&(<%MY}`t^K+!eYD<1=t!NN4ryBgX;LI9Yn=H07ZGXft zAqeXZi6E|te`q|xq%rPjW1np1(iN@MhQ#M0Y>OAUTo2uQs6|M?vnuH!S#*i0@T-&v zQ**{B(L-oJBw2+digp~3LHfSjsaX1WcRsgyGc!Qh#FTB|yv=YkW}VOUApf@fN-`3y z&%?7djp7lFk5K2orY@m`x_ws;X?ztW8t3V8kV<64S!i}(uNUNFpRY(No=9;w0w=ir z!`gHBD-UyX1PWun0cCKhWIzTv>M<@a8YE}weQ?iO1kYJ<^}LJsVxg$vvn1PSl0V;( zw5F_$HLsrbP%TaP&D&i>`kaIfXu!Hx`D*Yg%6i@FLY?geE^~{9MRr?ObJw}v?aYhP zzb!fUG1{hCpB+QJaYSaT))wNIzsQo7(kU{h1#6W@1)2F-v%t3S*RP!Id%-jre8LimRlN;Jv{`r zy!}(P6Vk-Vd5~2nW}?^$r<3zgK<@RITcGl9UCE6eoAXj1t^VYSb*u&;ACPY-=&uFg2hXyI-?YPYTkynb_X6~G<5^i?RTaSmTdR=YBaw|LdJeh>wa8%K>e7 zz{{R|Qx-wOJw9+;eD<=bEM5z*X#}HTDqk6zaNJKm-ID`P+eLQr9Rc5OEcCb8caiRE zze=-MGwG`HS0s1CJV3g#ggcOHg?kC4y~EEpMgkj#>xA2A7;hZP#KE#;?p<|e?j+!o z*D_!iczrV$78TJ85{`^F2Z6`-yQbz-77hdlVh6nnErZ!dQ5?W%QPY=9EArqTuHg7( zuK8&S9rD~hU!#;1F{X5)9P43z0j3aQm>qymwhP0MCs8ekPmtTxC-CAkj%W7CtugYA zG-fm^k07FilS(m%;zHV_6YlNwVRT9SLNx|Yoe((CKhlHZXGUypnv($`*=X$3alj+D z$Q0e+YB%KtRN%b3I86o0V+J>2bRQjBiqhmkHf!=d-S%v8nh?v&x>#}v^yl@e#h1e{ z1&cE6bX-9fqbE2|r5azC2<_Xd_Ucxq251fL5CoB^*s%8x;ShERFFD40hYRlcNS*y- z^(iv)t#Zx`%RJydtp2Kn7d0F6I%3s!M{}!#pEx5G`LKkGhEIpkO^! zRzsZ}0uA7gKP^0KQ&;EqDPu(*UJiQp#hgk2+o~qWY5S%y1aQukTraPe5{o-fczHJJ zK1%Q6uaif}r+8Hdpm}uWEbdF2I*};?U|EgF`gL_t?pAZrX4NTYlB!AKr9xUC4GBpJ z7YTtAodo3EY44l6!jvQ+^tHcPQ=baCW82d>#>E%m4Lr#A2r!EEbqPB(z$v6h1g77u zy9Y|6TX`wwj{^Gvwk)BpC&)@KSmvK)4_3k^F#(s!rHI2h7%igc0m zfZH(9{SCDf6aeL4p<)bjXECjlqfK8LKM=PGaeq@YyO?7U;nb8%t0VynAiX_kR8ZuV zd=FY{n$IfOR2?xewIHmzdVP<^a2dhmNpQ*k!KSX-_|^6IGw}EIw-#n8+^sb>_rWcG z6&IE3;?=UhFWkUJzTcRszI?8!`uK#4>8#b{(Iz4H5%yCd(8`{tpyXu^ zwlxLcRP3tkj$;l{nTzmDN#?Rxfo{FlZW(4Mquh*+ zl9ZpiU#A+~*ek!K&ClPdY<4zY(p9!%C(w_brJ?(Sey}7z^n^CFs414L97s?^Uh!o* z^QdaiB{wx}cv7-8BNG;zyhg*>qv}h8IDpQ))yRr@X|tsDvOH|NNpows%36308+yGU z!uJ!|KDfU&_saoNKs;R@qXak-;l^_xX^b|3YPz9p;dUXRa*T)vxBeFW0%rXD#rxly zNCtX_|H~7}z{K!hF(e~C0}C@V`+u+h(-XoZiKn#zyaQ~}ssNBM zrM60u0boHx(;<@ZV?qFNc5_SvmgE3{=0%2glmf@0@Ebm$$$@co(r09__qO}vOIpIm zz0e3_U+tY6S(_g}l!9>)9p*~-HVmf$G?1FK=((C|TKYkbVdGC0^?}HNboOFY12XHu z_aU1Z8<+wDXaX$xZveouh%2-BHB}^pG+@#!YU;D7f`ivB{@Np`Dz3(v0EmTAR!9Q* zWxxQK!?HZ{-Cpp|Vfjje`4xO*{>tXI_VJxVC0rC-a84vtP5G_>=z+EYW@lCZHhtsS zDA`T{_$qozHroMh{w@RX8y+7V^o>qk-Q3I=onKrHfVeamIn;ed=5{1EGxgzI89}>! zIst6}{$fT^jeS{X(W>cn(UpF0lJ6$$T+#j#ys$!~^08e1 zp!~4|1NyCuP7Z(6ZGV`UcmVK3vW$a4-jo|kSZpOKT z^E~DAG=cq?EWxiJCn6~?nIZor<%TZ=db8%lCq;ux?Jf9SLQ0PFO(bN4^NWs#?Ry|q z5RYqTU)%YKZf07@y%$&OogN3hIvW4#ZD;}Y-xk>U#cK>^8QVIT-I~_MNn_z(iK*eo z2>*rlVIugpZUpH7Py^r>1(+M0ig=f8>}`4&o%$I4Vg=E)1+N3{8yK3wIJ2|{^!ns~ zaACCv2GYvX^6%OCb^2oaYj6yfrkUB^HU4#7hX1Xs4Q2!gcK0*+HFC6D^NR#9tvj0Z zavhZsDBzL<;Ft%B*hsi`Ed7W1_cyEI_nhR|!nD-P0IujgH~hyoExfxq`_vcoM>Yuc z4o(?UxGu3feEavUD!aIf$M0lfh1NvH7xrHF#Mc=SRDA;wFsce$iuy{+>i5Cyi$mVU zcEP*Tr|Nf0$JU4T=J&q4d|?U|#K{HB?*8W(sAu}X*R0pN4{rg$L>g4uTG{^NcX?V@ zJ^`PpuAOBJv2SD?l-}Nf@m|Ql9SInVjI1y7S$g?|Kel%XNY4bwUwcOZ;7<*04*+x> z(n2oI@c|&c^w|?#%`Xk>G2iR8ZYt#D?!{*>_`I&#^f06q^il(>uP4MWkx>88@c4#c?C~|J zo_Aw!_s@53zb{MP*C*nqXiFOU+F)jUU@A)A*iVZTu1gHx=j`#PZn?>?2l1CPWOsIg z?;+nS0|3MmKn4NLjQ{kTpj8iZQ1YAjiEPS%!UJP=(jlbzD+dBzT_{7toG*+KCnDQ=gqTfa<(qjd_9yjQfVp=7Q-e{wu8VOIn`!TY^_r2@(+=8 zN!gNRQiUYqQ5tRMGod@`i~I8c)m~j#g|gRGF|*|^5z$Ho8EbqFGRqDH?vonW#|mXU zr2hL_CcE{u^=XvO?#Lc9<$u!vTsWK4E#LgI<(E0h;V}1C2P=_vaE;S!w z#bCteBDe53x_8vhR_5_b@tHYk(O1##MY&M}wsf`A%9ioBBfZa12Bxc@1JIPqk+!Qg zYAJV7!$pj0FBZ{GE$pmxxyu&Sm6sAr%S|Yk=dFab^|jlt+XK%Y3%)jjJ(NSzHV%gH zu?%A<-FC8`dn=1Ny|;%tZ@Nl8($}Ewkn*vbz8ln$0aCz2&nM0=Y9dKURFpVu#EiE zCadJ@H=ZE<6v6#S#=P=i0a6!v28W5aq+;}qM4m=*bL;-)YJ{>p`$o*3l%F=E-k&2hC<|^0z$=i7{uU(SwgHD3`?S%vU^Wb~O-jEjR13t82xbL@KIco(CV{*!ER zp4b!Uyf39^3Xao*$H~zGd@ZJ`Rmir;76NTgUw|D2G<-wro-s?jrWNkA)VfrDz}zq7 zh19#{!t0`8OW)VfBBDtu^ZC>c<#btp@P@UvU0c`TC+tm;OK5dR=06jo*oH5IU)wt%n^{2+F94&KPw%Z}yh z18y$Py8wNu&lqI(tJ+|GV*fq$LNH>+!be4EDgVJQGO3fgk=++@PdU8%CD(RCgZl5c zf%0XM)L=#r6G{U!t3R~IN08vyC7)KLODttw+ZT*!_gS;#0X$Xs~LEQsy9vLC5Gc9xC`=?tLP`gKD>a&ZT@c0ne1o*4}LDY zG~W9fm#+&QA=$>oA|EA5vp5m*uEH{brG!0$)7d2)ilKQ0)#{G@-DPOq>-?<9bIBQA zy2kwhw#?Oti_d9QHWWa(@L17~Yd0I3sJIdg3AsmwO2}hv_Bop0lS?C0PHfXEZ`yg2dYXXQjO-0<*z?Kj zHa=h0bw}V_WD!EU+X-Snf&cTXV45vz{+FpS?Vp`$(Kgpy@bc)`3qIE~MTAPW?a0yC z55$Mm7F>{0KmFHdnxHu38$O;WNpG1b+~S9(`Fm@amgg;^S?A$-5&6 z*_P)jB)un8AIwzC;-I(GHQKoXUjCsf`vh|cnp#=!rnac&dEj=C{U6PEZ@dWm#&|^2 zIJ|i^+oU0O+mw*<;=^l9ssu-NNyzKF(=o#%EG|o;^`|uJ9W$6vms{=Z-c-9(WYk?_ zHDv-AP`Hfq3Y5d8fWBS=MbO^ogH@4I)sa5wAfGdyf$BXjNyK}3RR>)3-I8jvl!HpP z71?;msguf^wkA|5c&K93r2GgYa4v7czZSvI*pvn-JBi?X>6XwM_KNLJaFuq|MTUEl zRt>AiF>B(bt7p=Sw1nAQ*iaWsV~dQK0*;}rR?eo6qA~mEoE_ab_7-O}?r6KJBjVY~ zPXht%PrF2gnWY~cwYpMzXuITJcxrR7Y`oM^%+Y6UzUKLvZP0zzhPI9K;7x zR{8Z^*g6RnEGV8AX^gQDs|$g_=$BkCYVYqTt@Y&&ZK$o`eBsy1)E=V8y!_6;_@ztg7eThMsn(6t z{zgfzX3y5k-vK?HMB8?;mukymFYU<7_qc-#y1KOu0|@*bh~-(K6w+4X?(K<3Gq2c% z0=w?^bm?qFg~5jeN$NyuS`w=7)qV8YEzX2*a}c0vT;S%_s2rMGR5&U02g}jc#?IR0 z_6){$F%IU9C_#afhp;54dm*QP9`;KRPM94-vpQn9DgZcclCT35liaf^~n?>*BbQ=v4?ULMpE9<(|!qY z3uiZ6&;l)5jztxkzhI{66==fDdT=G(S9x3TP+vC3EDO*G1JMS|$hIB1DIS{y;B5P4 z+(uC@p*WY+F9H9ucD=|g?TzxDjs*(Yy?X7wb*+TDxpGHPnMEUN*)q!Fh0iA|AAx3#KM(Dvx`!;~Y*;{Qm!*(dka^f^I=VXoWbYCpbb5L@9 zdY&r1=U$(Q3{#&SJZJ&Jjh}S;!ICPvJqKc9(AilcDbxUEBjh3bOilRDZURcBqcs6Ebh2}P= zw?lKo)7$Zdy40#yc;BT{MZj>%JTJS;|JG1!2PQgbHT75Y&`i z&Q~P&x(KKuu_am}cr!j}9}K(D%dyY>VJyAhDe*fYq&-nM>OOiY!1XGTP9M3DUDJfI z3>U3AnlXWtQ@BwtwI`>eB8hs?MTezLejS%0_Ztct&th6al8Tj{3Qap$BJyuEW<`P6 zdO}6#oBHQ0E*P7_?W1KPLp+fV-&>meYySKLI!o&V<;dtl2bnrpN7qKsPax)E>H}q1 zHyg_LQ)WTpP-Bt~g4?(ZPwJcjB7Cx9Xew(p|L-A8M z&2nX&Ef@-`fPrGa6$UD1E=zP?9Y61qJ31e|aLsy`^lzvTlK&jmNvh^|K6s+~UZ?2~ zA05;H`#zZz@a{k*+$<@wW~ZBQ5C6(LryMm=FN?T{Jg9aPbxFMqa`<3z-5EEIXLNuc z{~L41W=yXPa&SHw(Vpse2Hk2Wub4M}2{#UZ&`_%wj6vw3U;cF<2EoLcu`dSOp$!5I#gp)e zj>O*W15Hrn746bFGu}|1@9o#!$;|%;tiQaWJacF6FA8grohJ&F;&O{x=)V<%@teZz zw4$nKg?m@%?)@a!Md2w+ZkMxuhZ^4gq zBYyLvGaKXUp@G~IwFc3X{vW8$#@_cFhwDW(FmodGaAH+ZxE5BCfBFkPxPemoIhINd zrUK__na|Bavp-Y{id#T6&1yWaCpH5V{hQI zLZ>3;WJC{>*1j%uw?I?XMTaN?Fa2QqMI@3T)aZT_i3Hfc@{P<`cz@ehr}jO2E8Ifr z5IWaap6%#sNh(f5uZD5`rxCarjfJ8>6mz%g->)ttdGw=}#K$;p8-gFKVumrE(!rt` zSEee-3;rPg#%M{i#+6|}_{NJC5}s$gvdEGMu~rP z=5*J-gz(?kqEZ)x?=$8obAgPT}e(glk?v3dN&9)gW@-L?7T9lDPE49Vt|l zFL!8Yt|oqr0dw>X!Mt%Jsh@T;6wx0qwoiVhgOSWc-<}zA4P_!Y(Jn9cyF=(#Rq2rw zRus%u9-$a8uV^ALukrqAA=eh^F|Cn`?uk1LHY?+gmb-_mg|{*rr9c=49jAKOB(B@H z8Ubg<)xG&t9+dkoR`@t9m@>Nvfq2jAe?i zz(S1mthGXYh^M5EM`bhksktBWu#eROfp~4!kKG2u6+G>HgWQY1jZh0+cy1eSDOS>{ z=|QRfsO@L;7RhlKFM)x}Pwho%a~O4265h;^zc7Zj$KlPVamE~478Ss%OFJKo+c-|w zMinH?A|sf&FZ%h(X}w=2_GL4Hcd?#rT90kzI&83IOhp<0u2&dAI{BFo&s&uqj2at& zCT`!I51i9fMV_wjRu8|(nQyaZ?v3O}*WeE@11+Y#5#USq=1Z7lw0!+&ABAd>F>EWs z36G(nhIih>`4FQ09rO$wO0#06mc?!c5PEamyV`f>5w&v82KSzqT2k+8Ivv6|nW?RB z;>L|L@eqAnE$qwmNMR3 zmC^{vMjYStw^88D7@IeJf{6JIyZU%3`W^B#(TD%4;6o(jcU(am;>6@9k6nHYX# z>(}+>4C-wobjaIv#O@aDLC@yZ`ED{>Ly=myFl3pXb{08w;1+DYmVV8?8NH2LUaNEW ztw^ks7u1(z5ocVjgn%3j&*uzz9VNy9t<1?#2mjF=Nb{V)8=(UpsBIktmor`>+~bvu zX+ooPl=iIk0>Huri2a zm4X+VWN1zN^ZIgc8o2hh$Tge+fuO{UWqIpKcIPDH2G<>y3A5v+Pz9JAJ-|JCR1U0B z?%OhRdbu{Q?`P@iNoBh->Onk-MbTzdiuN6|yOzlG7NV|27p<}r+~U0h!H(GYBI6#g zk*;W)hX7*9psL%@m1g^l*0@hd1Ix*5JbfG~0?d$0grQRya%2YC-H=PlQ zzNt<$RBjm$cHRq2HDZX=1zN%&xyLOgV*=?GF~Y1A7UkCG<=xC`_ZJ;UNY!&P{_onc z`N4aKzOA{-!BEXfB#HC6YRGcwR`M(Lv8(hL8@!5(j1~0mNW1y-HOj*2#4jzfH<~Qa z54z24m*6#TeQ3^?o?$E9qFF5i9igZPu~C6Wt;;qM_3aI$^q#qrXs<>dK|bjBdFG94 za2zbVUkh@!bsh5~^6ffif^N>6?9dg1KpR{1ks{YS=lv~joG5hQdy z&kikJd#cM=q7Da=&IPu0SP^pX%(&eBq|MEqfZf;5^rC{DzO)ZxCtzYOu^KgustmTO zTk|nk#8yM6I4%*4MD@a(P(sb&3`QhPYL%s&WEXUb>c3|RJhdeuzkK2Z_ZRtxLtZ0eTgCoTGo62lJ`!XLF@};w> zvbVINGfXtnF!z>B(z|=xu~>DND9to56Xw6{gvHV9YMj^5QoHU^>{VLCnbHW*oc;n^ zZm;q$ylfjSUk`@a)bPK|S2g3i*)Va8weDqt9bPi%|d}`j@j%CR&bv0d#}`Qd`hwf3?HOvSraBPF?35DH&iwetFA_B6o+?{ zQwA^j@%b80Dh!V+8{w>VA_dYFJ2aZVH*}`VTa>QY1Ugn%SAkfC{PAYzqE>uZWx>Yz ztht`ZdJIyoFVN}B+x=X+F;Du@U$bk)8_rXrcLt-?1uOZqtWh(zL|>+#)0AlwcxkNU z5zYz_CS8CbRyD@rM=BrpR+U(?NN1nH*H6gIU{Vqz^oAKRmRz(*b`Nqa2w~ZeiQ6M9 zF|@%%a3J02vkKBwm7l-g5p`vnXfG&)e)_8wS$P>x_ecF#9ku_1XN?1QumJprJb@vNMe@9 z@zHxgI50=HdBHP=NFuuTlM+q4#6}oL6Ma$i_PkzELKwK?NHbGf%#>ihwo)~VV8~@s z1jTYQ4I7)vGEHWr$&KxR()aTznl2=w=2o^N>PsbJ${yVs0 z8}nQ>lSMi>+gUEIWqcp=y1(jlPIQ{0VBJG<7~FEI5ic?zLU9Pv#A$T99$ zjqSx|;&Hq&jNv0?&;##;0D7hjh#=lg3Od{aB7%M<)fjUE*`JYufe{*)%>@Z~p6I&V z=FA)P7VA|v)fX?l)qh0>h0caPN(aI_$bNYL1}E)Y(j%P`qf**J{m()4X4$S@GfB-9 zEDrB-XQC0?MGT}N^7sYe3*BSG*5hZ-T@uX@2Mq2CrEA@k#6J@;&*)7tLoZ0d9ZiT# zQ&%r{eHR*0pDhi!_nIeOpdkKJ<8RUYovFoK2MBcka)7edvy)NGU(Z@wuj|{pYnAX^ zzPRElY&Cbu=jf)kBrET*5bU**5r{*7B4-^7+N7$k9-gmo!*NeAwhbxwj!z)2fZG1{ z#!y~BK1>h4&RU+Oy>4_U376KpI6Y@P?Yxk>F5kk`HMgoQp|6u5w8W+3pc8fll&gy8L{YxZ4EdJ|=H4<;!Fh%VRh z>UbH^=~e2)R^QVZp1DZ`=D@;-%4L=C7M264JFflGxA`94rm__0+@!@U)~65+_Rl{C zorK=?wAw0&!6o@V4+8s}f0CAEQ%mkUJ!RMlc$B;FK44E? z%TptE;NSCbTZ2{M9cxXpDb%RM_5(7`jwl@z9B97NvCX%I+yLmXVj*$2jfr>|1U>|A zE|}e)8#5u!JF}l@*qLIjygu&FY;+9HO>^y}ZH*W2m^^P0?ts5TDcg_)d5_#!<=)q$ zo=FW}ATj2m9S}IoHMjJ0Vv=P}hEE#Ckys*(pMUP85|!|#D99B5Ad@24I`RF);VI8v z_BRSBZWWqb^h}s-&7EQjZWF~Zr5rL(xnS2U4CgL~e!4M9Pieq2niDha5!o<&!AH_wud_p1k!x6^$u@>mUUx_@0z zHl-g|#nl%aQ7OhmQK3LdaoaY3H?65do5^B`XGv?!T^d+#`=Nu9*ce6jmJ@S8lmW@ozX!^uIS`$s|3dOVWv6YPB)Jk@Sh|>PsZ+6|p(ayW@t_mQU zrSBWgADyUxQ_NEo0U$!KHj;ai3+3bH7;8RWwsj8D%#nD%GxYI zlrA3eg!bEN9d3lG9l|IFj#>lg-e^Z%*uBS3{wl5WnRK~HaQbgPp@K$U%mb=}$;lc> z-iHqF;O?pYi&BD)2wae%76`Xz-!6+v8_FGwt(`#WyrgfXF z!ftMXU3Qj*Ytjwq&znwVK=j%P(~hh!O3P+GGIS~PHg|fgyi(f(b=g?}F2~1dDYmtl zL5%>eAchXVR+V_R8Z`C&5hb2F>#gh)~E)1X>iYoHCova6CXZZ>iaSl!;YKBQ~2phwo#h#z03MV=0V^?-V;63QMZn?BA-BQm4ROJM(5@lCXB6fZ7O96&jU4?`kyu z(KEL;g6}}_V5MVAijR`*%?(zL<7AHVr*F(xsWr%^{BY0OIl!6_5TUez`$gKhlW{5< z9o)-q4BPj6-vfccPE)Cw=9t5__DWa|Mfw}6D$z-va=aciQEw&AH{Bm7M>-**i+3g# zZ8~3F3rGV*UNKycG)y?UiQ=@}HY=_MXC|=VoO*T~wdRL5CCJpim@ud6y zYQr+#wGy&dT!r8Z?j{egGfBiUDNOa1s{7^y(-~k-9v5YRFRsfzk(jsN>zFO>e9~hg zcP<$d`c~iq^-(+BaN6(HMwUxQu#1CfAqeikI~J|Yu^TaHl!nJH8CDPhRZ4&)%~2V? zTyt~#X1Vb+o1;W~WNev4pm~s{F74w*rbpk=A31uxWfij7=mRw8ugMnUd%R0+iCems zeYMChX`mDn=_`LOhlwJ)hA&D0JN2TKt~m(&E(XOzUtEeL9CatZ9Mr!%W|lSJK8(-& z-|sjTAu$F=*vH7F!lBN>+4BPAf$-1shsN!X z?x)7#Ch?a3;}fL}>+D)0n7+rRq)>h$Dz2}bvKtJ5CMn#U5`aB2zW`;m>G#LDD#?}E z2+NJMao4KFF+}xJ6In8@`W4iFo-RNP)D#owXH);D7m*3g1|e12?Vqi@IK2cXj(9B>VEgUzy;fzavOj5 z)Sfn8XiEn$uP~>yVJh!XikU`#ja9ipfwE4bm&-^1g_Efk=v0^feWOfwSw!C!G?3E? z3u3y?R;x=uPaw+G4jcz{tz%1M)4kXeRYxYz3@d60yQHX5&c=#vy)s{MG{e3{u^#QO zl9U|gMrK$ug14y);h1(6wNdTe1>z_tImV=p%zRJLF;a72Phjic67MVNnt~OBk97Q3 zVa{cB$R4@nPFd$k(-jlg>>5)9M4c5fY6pD#@mXs20piZ$_T2oOsyD%1lIg?4+fD}b z8N7S!ATY!;uEbh2s4ZT=%m=DV88`4QhYnL`akCrmc5!Rv!YfsIC!e9*;G#dIn8)ZL z+9uB8U(&~ckLn8@+!z<8P6MUXDZN(1)~43o8BiT$j-4S{1FD+gwY#T}R8=j_;8pq= zk4IgR)*2sQI#LR=O?GF%5I*so0tJAuOb$K9gd?2!E<@bnlD560x&<7a%ZQyU=r*lA zYlouKK~9;|E_(zW=QuR z;18z)F6fE)I+tYhw=5pE9*}2cw6JWN3%L65 zdOu_Lx!T3v!7& zw&#l#xd}n&>%Lv09z-uaRPGcC`TsF;_!) zzSovP+cbs;5*`2=>E2a1@>*tMLenGj9-Ith(epX^?vjFw)A-7SQX%3mwy9;gwlqnS zUm$s-X*}G2eItw`SCX_wXDqg}JhL}wwYu9(ImO?@92sj#Aj5rm*x_`IPYuZ)WB}o~ z-cS?=qntb+?Bf>dbMOhYzqZD|P%zREOLPr6isbMH$bpnHifoD@u%{Dv1KHsDczPaF z_YyJOE*ZyimGU(7H&@ob;b4x2zQ_w~B`s~FYQck_Uyj)SnrvxP%50$^@b&&kAN*-P zG02cgWrgm+rk!&@{Zl~sZMHI+T|m;e#OSuqwIY$*b4UqLE1UL>z&S7LH^aZFYG&(I(=okK><6!ebQLxH6-=lN z&!Y;!Ol^*e{Ge?XP7l-%0KD}q^BGmm3G%N{sDHKK09qr8BUsnB_b@@~3X+UkCG0!v ziO-ij&NO4k2eIcEo!d`R9Hm~$PaU>LW+e`^9uk(}*!@CskKCf1<^(!{_RzZ^=?Y2F z_3UJ1VP}5F7-Bza-p#4NT=VgsZ`)zGW0`wpg4Hc&iW*BVYk6ypWKWajqH`u6^8Pg6 z%`Uh(jTk=3xl^Si@LI`LRF!FNi08ox8DwcFOs}Crq1(*i^|E3++H}VS|68dxyOxa=bowwck`MC!&rjBEA>Oy0hddAw@hGN zH**_KOkMh%2ze-D9k+_7Si_5PKMLG`D0+b!?y!SgoJg*4x|!}|3})Q|btIvEJ?G5T z7Ka|xYGp95#yOTp#QZma;R@M7#WxVGq%=5!5z~YjYWa<)C?SApAlo7^Tk4EVR4`!w zRTBxTM^xeP1bX)`Xw@@K6tRG&tCx3arnUN2g(t(9_;F^5@|D$BTyIMyTt%UUnw_P$ zKF=PI=cXAxJaEv}DU^bj!Lej)FSl=_>+X&c>MUX>EPKh5nEDOXX@tR&6PF zJ2dlm%wNx8;ATudm<;DAwgw&A(!6>8G~xZ0%wVNVnAWr;dHCzPC7Lb_>Yx}^iJHZC zh?qNjt*tY|h1%|k=;8J=!bJF_*bTnYSFhNEo&ej#rmAF8;{o9k!2S^K^HT2%PTh6?O_~3)XdNVVFo}ZY<1sFWE8#{@@9fCcw;L;aoU!7OQuQ+)pE`I6T9&2H}d+gOuZOv&_ z0+{Zc*jD69lLPmmC&=(>&5ijQyS7Pw#F{IC9n8|PVj|%4%W&iZv&b+GdSsIVh0aOi zc*xzgRRF^%;hMu|fOZbo2+hXyIyu+xA1Ea0o^?l}r<34;vo~jwLNLw0U$29mI_TSO zI3HNz#lWMG_GP-5XBn^pYlu}5i}xth_0IbQFK?mUp&=Uvjj}qJ#`;%kn}q*q?5m^V zdbTw~AZT!cOXCiWCO~j#T!J9Fv zPl-zy0(bA_o#!nS3Is64)calZYXrwDY1yMyF~lH%yaiot#P zGDYLiwTK-#>vA#o4S!K)b!WN*r!Nt^uJMo4#su<`!hQ0RV2VoGJ$CLv5+Jed#3+Y@>_J^B$$qUg=Ro1({6~ht2cB}mQac$k})WNw5CnjRy zOdtS$60^(O=_yI}nWQi1gGTFK0kH+a4aY|);8VihT%uGVGoBmzm+I#rc(sVjn zj}=2N3=o;zMhYI&S?4UOS=K`;MTq(aoT^6DxAa#vJs6|aJu=P~HUS@xTR z!D&99h9Hk<>&b6xQc<^j@LRp}-E)xG%_L@O@-Tc9r`kFSXdY~~zTrwT`1X!!6X9nw zHbf$+zAV>wxTWqJ|AC*PaBs$cH_(0kF0q3KFX)dIzG_nhEt;EaLw_}8ej-t4?&+`5J$~*tEl0zsM z;gNKY?$b{|fb!K4xM1GkFcbmL6cSW-#!HG!;Ni6t(`UIuTRZhKX}CKSQm{@DF9lqJB?dbC1Y78I4M zls`Fp={6~iM4K=uKN(;Cp5x&gLskW*<)-Tno=cy$V0z{PJ!RkCwFmkNZNl&TH_@56xNeR1!F+CaLGV;AFB$N3Qar}U zwThX}9NZ(`nkpiFuRl#O90xmq>In_HA>*1EN;2Y_- zswC12d$%=hOnkwJ`trw~ip^^4=_cs9>ySaoftSaJTe2wt{u-M31-(E>i*0$O#-XVN z+Y|5ndMtoDIeMk9=t^3 z;|iGDyoICRZCh<1zv)oqh@cJ_=%Uc^qKJgeF&aA!0Yf%Zj{|}Q&{9;6(-VezU}2(& z{Ifk$Hv1lVNE&p0nN36-S-O=Go@q_R47WKH>c(tnRsI!UrkB6($}C)irq^8dkr-Z2 z+|c2$x2YtP&++w7yg*I7b27XkcJ|vZBsD_!&o660&9thu$rV3@hZ_GhdbE2q)E1z2 zU0!r05tr$~be;^3(?j<&<-REu*6CPG^_k&P5chnC-)k%>yyz=bYoK;zHfJPPE#v{V z=M}mA19cn0`;rG=;oiXl9HWL7CHn~M$pMa;3koIFh{U16+YH#>%((O#5%SvZr+5YM zJ`%_Q^8E5$@`-&g975$EKURuxlm)$pOW?jS$Unt)WnhehJ(dm_-KQw~o>@SV{+p0{&_<5` z0C;Kw9X*XtFuT)A!Vnrz9IdCipqrR1>2IR?C)u%YIqJe6jhGB+vNNnTa{Vy-hDh~P zkucO^&y@OF9vg!TU&QA_xX`4pV63ye!0)2&AG^Vh=#{?SeQc(Rkt)X;6u#dCiKpu9 zR*qK($Y8(j(p9}FKy{XvoGMO8i72*iV_T+z56xZ^l!XzFTSLOPe*4MipAN|Z{rfH~ zszpvIuiRSm6$%xX1M_bNYI14Eo9b^rNQNFQ#{q-wG8H^|@=$f}^)$gw5&4Elkt1SL zb0q>FMs|LFN7No_2wUX4FxTaJp z`|*+C87^rbQxIup%nTtN#F~68QzAy7y+=#hlBJ??%QH~3>&4AW`4~ms>TS&QBfF%T zx`XCUh9g`rTsY({vO}`eOkk{{tcuH2A4l~GNp?CZdnrhBduwxTcbu~FC6ZCra&dwU zaV(Ll9D zo3^l?eHY|d@_y^7VbnMK;Hb0$3ri#JWgP{a0c5J0PUXYzgWa;EksXp1jcbNX2!&Bz zLD`;W%<0=g26nkwgR>Me#7JBg(-0d+yW41n=8RT5zuD>H2!eA#?}knOL?IH6^3N-e z=pS5N-fR81aE|j zj0#6ROXiNS-2#hK+W>bCw)b`7GZQDeFxUQT2~=#(o`s$;UVlYbpBXT`vTZ z&R@TJ3}1@AVP{HnIf=~fp(8zB$ZdX;9{t2?>C`TfmCpTHhsUtK!j*rgzy^szzdsk} zrpd$_b-gz7qX~xb*TEu%un*op6r~B5Y=dS|ckr@a4in-Xh-riE^aXLyFl4 z>=V4FFC+8E2TPgXxe?Em7Hq(~XbQdhBrh`w*2VbTVP;;?D>b{+u^`<3ffy7nqftD0 z>cUp`J0{Y!Hc9yDu-rG8T+*INIYZ66nAhrE>>lZ-qT~wVHCrk{U$K$J0d+Zbmk>9F zZ&bn#m8ZXe@2BdgqwT&0#2VOEC3h?987Gsu`9yvj*0jPe7CLHD`s!3<>^|;u4pOpE z{yaF5>b{j4?e$JWAzq$(+jVCXNm`A0VR&%*9X(iZw2bB>AM+pir3KmO$+pK^HXlP% zQN9Ycxb%bo4AO(iTKAs7QpnB-ZQ}mO%;J2rEKUp~(9AgKykF3G$$0UmK>Z4Z~=S^2GVR5aJ^d zpyKo>2Y!~afTGNHUO!L6_n^wuxLU7v)mCZKASr>w zcd<^3JBB^njGWp)H#|NRJ?;MGIQO#d*8-5NBN*MxF;Q`#>k=m z3OVu3i+kbIrC79g$18Nb#$bk#qhfz5BVMX%br{xVH^fhGhKPxhPW=0~ge>k3z2D%% z$j{FV+@b@?<-)7Sp9yZYC=n2G%Zp~xv*Gr5eOApk?`be_YJ1213~5sLL9wJDrW?hd z_wxAq)Ko+Z<6*~HX$tUxYu9hJjiT%Gx$1Hzan|2RmgSp58nA9JJZzjwY({bvD{XXb z`TZzk4%w)7O}%8_Q8w9-wPq{JrZ3%AnZIVQ7NRK%Ny>2Plgbl^hi~}x!%f=t)m1b3 z1lk+6%qg2v^2)ec<*C|mXUYAn?yU31pgJZhXe$Vu)l+%X^y|J@B=|O#AMs{2O-!MUN0RMvtLfOIA*vSa$0AR2)Hc^9uSvlCafoz;WMpSlD zCzzS71A{KtTi&PgRu>2a%03I|vvBygX8o+WJqukR_fr7^LC?bW-vaim{dM{WVE9~P z^DK;?#Xr@e&&J=jzrFwM;VduI6e&x;9}$8=HTJu_Sa5DQh9H7dIzfbWTw z1Z3{u2m`$3=KP0|1i;P7!}phk1n}Hy1$&w;JILy3NhF{~w#Lxsz8zstyJL;&I(MLB z={V0j^-!koXB)ZfLW}hlXJ&Rb`DfHuSX%#_;uo9Up~l3xSiH`%-lul?< z%qfp70gc`4fa(|{6p;whZI(6!``=&|Bfd%z9>j`x0~8_rdRe4*Mz167tT_-?M4-&0 z+ccf97htZhB5Z)YENS3tdzH|bkP*{w%^|(UF*Fb4?(#1NvJt3%*~O?QbfqvQ>8KCO z!zJTci(>h*NRHP^Sf+~2H9DRsqP|)shzt6JpUSrbhxjgY0uM={;J&cq=DF z?UJWN`Zq!d1o5WX?fdE8Kb5Ao;#-CK2=qaBk`*aXW?@xWxBl_8=VR3o0nKbRB%?I> zokZ~Hu|)firz;`on1Q2oVt%`P^DkDkLhaouu3AGEs9&T@puP~MECjNP69O5wE*QCeYX!Gad^!@~-ZytN;>|BSpL)%4g@p%rRf- z#e52fF@z;2%$Nk+D#}~Rta))C>EfsoRk%`R1?_LCMk&tXd>j)oZMkN^h-4)JhAVv}0NVe20`Ndn|; z1F8ZHk%wQ1N$p9aS7o1Pm5qz(OvebsHFV<5Bp0k-wv}|z4fNe3CUKlv2@Y!P&pJ5U zr+5)li)4jHTmUDT!Dbo^_ckg%lmA;4*{(i5=BAqZ^{KNPPjq^UpE2C6aesDs<%qsq~W48m&+saA` z842gOW#gF-C_R{!r|U&yP#&?SbRGd}0+OjyYJt9w<)aP>17-KXY&q%9mGY8d$ zSZAA@YIHR{yp*Gp(n=VvQa($H!haSZ(Su5PV z&^0bZXAQSBwr8W6LXR`<2qi}ynef$e)CLN#g2MC*d*_$rn7kuk60PG2Em>5SD5y*t z2faI=ZMi&oNz*TG8;QQ+OLPl*6#0p<7c%Ha2!eRH_z`XjOOilV$D9 zXQkCbQdKK#{AA{}h_x_y0dpKN4T7U+OiLe*6T&dnQk9RVA4n;*b826xV8AVWzGM?g ziBX?_Z}uy=*8HYSJAV&YJ{M)9tZ50#I~b^Hx_4aqxJ2tUMR9gmdo-x8GiEOLsG#%O z(?q^&&~C_y{%C2tDrN>l_1DAQW5!)Z4dI%G^xE45lh_r_RXSQ_+KRN1SqaXVfkxNZ z?-Io*X&1_ZidH&GIoTW)sUxE=FILcV+0&?n?i0WTC}~9_U|dOsK^XNO{(>oT-dgXY zi)Ln>CEB!JF?UYD8?dQG;Y+<}3Y!KIk{DxIzu!u>9@@hVX-s|88uxIW@BU)81gwb9xs#g!R2LZE0kXtKtt{sion!&>O5P-cqMM(aeuu zb8`s^<(%y%@_^#IBG>~Zo-7lHJLib6#02BKP5MlN<6b`^pxl#CP{CM+7C389q+aH5(HEzIDIfe3 zd>W_l{6ih%&|b2FI;-NAktMvp8r0-F=VZKS!{>WNE?dF9cG^!=ZogCArj|_t5DD`RQLo6(NOwOw>@KP6ybvWNK#9&qyYzKaiBBuj!9hZ zyR0cp|MY%tgk>&zge?9L4LQhN4}_g6GiF4^A7QFCK1F>lAp2YF5hd#_dEB~wqxpxp zD|O3!iCb_~b4HN?!cHL`S&x{;hLkKtBQkU1#J!(Bq1I_a9Cn0_8}1vyz0j=sX5i2i z)2Az22aa>KjS`i|$>d;Di357_?2rUhGPxm`+^`V*BN`{IMy6bxc=Biu)m9~OqN zcX3uNk+^y2G8fsYc!WaF`a>w@w|30&*0U#<4pBW9FVn3_o5+keVDaq5kT;6K9hvhF zrvc-MhmVijgZB1mY!Yf2wPZu%{jobD*Bu!%Q8kgXV19n$tbAXwYtxUkX||%4Z-3wJ zoTw-0f1FI@raX@fg3q2-=O4}=CP+Mukfiojfle=vXAfgPs$q7_y&%`z3B!m>KZbvZ z%{Yn6IF(C(IxM=0_VI{T|2S6DvzfqHV4!_CtVxOZP~&>7>3;lBNjrXnEgn~8mfzj% zP+ifTNk>pQnT;aLVZFBKF12;#_Kwx3C;A%Y@x?~Ir}jYjtBm8!i z+|;z;oo|)RVC-8&ChyMH6aA}l_}&R4fQQT7Mdv45xY@7Oji?sw>QPnw=kN6v(bHl@ zhHQ1Jm#fdbpz!Z%(CRrFqao_i${wMw)LI9esyHK@s^yK74O)qDl6`iC?Mw`c-S2 zepKt%2>pR;$%>EYi`&9?atE^Eo-yqisBv;FIa?e{nVzh!tPqt zo6Fj~o>{W&cWJNOlr?E3=U5vpKGCy^jB$s3V|vsae;E=9`$HZ-q=X^V%+F`7)~uwu zE{MP`IF*-Z7&y^7|NGa)N)RiOyCBn6%dAh+<1R&W37*MvR$_&diVANFw7qA0vcHaQ zsc6>n%6^KicT%KZYGWypeaX}1#9t!;@_|&n=`8?MRS{;@{6?cJ-v|#Kmea2x#Td(4 zOKrN8W!N_!%c*(Yg|rtj9LBd07Q3gkhu44?dKOEbYErz~D36IU-_ip-?AH-Q%5Y_Hu)vdX9GmRWM_6#l;(XR|;(Dr6+xUPb%D_Vr|xAqgmJ*MkT ztUDXkHxb%$m?VLE&hVR*hUWz3=McoDC$8ejy_o~`0YN4YpU2HxrJma~KNnC-@qwk6 z@*U#sg$sC|)zdeLzRCMXje8B9c}dI?!a&)a*RI|XQce@=u3k8KGm{%y9c%O-$sa}( z{$Gns*ttz*CrM})*v=)3^-7nvfEzbWi2{qc3fMwz@H91;Q#V0&^vyq()`9I#@3oi0 z@6&smH0GNIn|}uWxT?4OjJ!LiCjBF!@QN>}70UZPS*h@%Vht!$E$hr@98-2XKJY9YZNQ0;>|msg{%a z7}{WZwU<6^6vYjW0{vty$pU)eS)LlzA30hmeZ1GpC(8~6ml|DfKK5Op(4 zp?&+JnLcS7!BGh?6Qyt$Y*0r!u+55Ll1-0gbvp$UFLPBblR#e&6KI>j?X7X*YM`8A zMN&Pkb)8TGOkLd1DYIcOIsl=F7$*Gf1GvbwP8k)9i9e*uhK|?rLwawczVe=Qv9f2U z$uXzxO6D$Lv0+CO*xcT41}84|mDBBpN)kkZ>mXzI6^ffTb`3)bbz?0L)LsMU0b@>u zRJ@p2;U8t#cGA{)4%R_;6xStIOWNi7br?@%JtbF%QE#b#sVgP6N5v|C& zP=T>l8jr{o-YdyhGcLbEYP9LX1zKL1iN(@j(M)Pmdg#xtPtN7~7p7`L#cU zh4V2dHum=;^|lG1Kwr>c4ajJrL-Z2Zb~7u=L6)JmQ#AL)GJ||#h4q#s?@d9M{Bc5B_`J%J?h!S|j030lZZH;{;*S%+O6wm;CSo3tq3_{Ivc zH@)e*1*8%rW@C*XXzDBET#!!ffhh7OA3T$r8BK^Dl}+@(tvV**TjT^&bdQfOli!cK z|96SG|D7pV(7c zhf4_XKSQ3zk%!ut!ps1iZ{Pka74!d*iy<46;p`N}^F6-2q2LGt#2`?H0yf>!At7$* z)Cu^td@ltyUf}n^S&_EAu|tD~Qm=S?)%RMwFKQ%Ur8?P@NG-U$qKWf|$MjTw`db8L zE`&nX@Quc|*9nwJ8_IC;SiZC~zD@=H7Bc}@ftjGLM{9z~4q~fz-hB}=t0IYT-s#up zOqpmp-G&5IOcihE!^K_`f5T2u`+ey=_p;}+FAb?Es`e_@*a#xh2uAn_-05dOMex&S zNyOfsiLjrE3cqdxyl($4OJ`V4{Gp+tl34xPwT@Wzy0L~>lQp=MPEDV^lx{K?UPE;k zRjAn9Y1&1DYgfpp#ALExs$|oELU9Fe2!Oeztb9m${V1BQzmC}{Obz*eci$0a=m2wd Vc-n4GZVuo(4pdrNNhJ{Ke*waU?@9mw literal 0 HcmV?d00001 diff --git a/r_tutorials/w06-R_tutorial.rmd b/r_tutorials/w06-R_tutorial.rmd new file mode 100644 index 0000000..1f031d1 --- /dev/null +++ b/r_tutorials/w06-R_tutorial.rmd @@ -0,0 +1,114 @@ +--- +title: "Week 6 R Lecture" +author: "Jeremy Foote and Aaron Shaw" +date: "October 20, 2020" +output: + html_document: + toc: yes + toc_depth: 3 + toc_float: + collapsed: true + smooth_scroll: true + theme: readable + pdf_document: + toc: yes + toc_depth: '3' + header-includes: + - \newcommand{\lt}{<} + - \newcommand{\gt}{>} +--- + +```{r setup, include=FALSE} +knitr::opts_chunk$set(echo = TRUE) +``` + +## Analyzing categorical data in R + +The goal of this script is to help you think about analyzing categorical data, including proportions, tables, chi-squared tests, and simulation. + +### Estimating proportions + +If a survey of 50 randomly sampled Chicagoans found that 45% of them thought that Giordano's made the best deep dish pizza, what would be the 95% confidence interval for the true proportion of Chicagoans who prefer Giordano's? + +Can we reject the hypothesis that 50% of Chicagoans prefer Giordano's? + + +```{r} +est = .45 +sample_size = 50 +SE = sqrt(est*(1-est)/sample_size) + +conf_int = c(est - 1.96 * SE, est + 1.96 * SE) +conf_int +``` + +What if we had the same result but had sampled 500 people? + + +```{r} +est = .45 +sample_size = 500 +SE = sqrt(est*(1-est)/sample_size) + +conf_int = c(est - 1.96 * SE, est + 1.96 * SE) +conf_int +``` + +### Tabular Data + +The Iris dataset is composed of measurements of flower dimensions. It comes packaged with R and is often used in examples. Here we make a table of how often each species in the dataset has a sepal width greater than 3. + +```{r} +data(iris) +table(iris$Species, iris$Sepal.Width > 3) + +``` + + +As described in the most recent *OpenIntro* reading, the chi-squared ($\chi^2$) test is a test of whether the frequencies we see in a table differ from what we would expect if there was no difference between the groups. Base-R provides a `chisq.test()` function to conduct these tests. Here's an example of the test evaluating whether the frequency of Iris sepal widths greater than 3 differs by species (the null hypothesis is that there is no difference in the frequencies and the groups are drawn from the same population). It's always good to examine the contingency table before conducting the test! + +```{r} +table(iris$Species, iris$Sepal.Width > 3) + +chisq.test(table(iris$Species, iris$Sepal.Width > 3)) +``` + +While we know little about the dataset, sample, or measurement, the incredibly low p-value means that the observed frequencies likely did not come from the same distribution and that the frequency of sepal widths more/less than 3 differs by species. + +Note that you can create tables to conduct $\chi^2$ tests by hand. The `chisq.test()` function will need the object to be of "class" `table` or `matrix` in order to run the test. + +## BONUS: Using simulation to test hypotheses and calculate "exact" p-values + +Section 6.1.4 of *OpenIntro* mentions a simulation-based approach to inference when assumptions of $\chi^2$ tests aren't met. You possess all the tools to implement something like this in R, so let's do it. Basically, the idea is that we use simulated data to construct a null distribution of outcomes and then compare our observed test statistic against the null to estimate an "exact" p-value corresponding to the proportion of samples drawn from the null would produce statistics at least as large as our observed data. + +The earlier (3rd) edition of Chapter 6 of the *OpenIntro* textbook uses an example of a medical practitioner who has 3 complications out of 62 procedures, while the typical rate is 10%. We can use that information to create an example here based on the proportions. The null hypothesis is that this practitioner's true rate is also 10%, so we're trying to figure out how rare it would be to have 3 or fewer complications, if the true rate is 10%. + +```{r} +# We write a function that we are going to replicate +simulation <- function(rate = .1, n = 62){ + # Draw n random numbers from a uniform distribution from 0 to 1 + draws = runif(n) + # If rate = .4, on average, .4 of the draws will be less than .4 + # So, we consider those draws where the value is less than `rate` as complications + complication_count = sum(draws < rate) + # Then, we return the total count + return(complication_count) +} + +# The replicate function runs a function many times +simulated_complications <- replicate(5000, simulation()) + +``` + +We can look at our simulated complications + +```{r} +hist(simulated_complications) +``` + +And determine how many of them are as extreme or more extreme than the value we saw. This is the "exact" p-value. +```{r} + +sum(simulated_complications <= 3)/length(simulated_complications) +``` + -- 2.39.2